You are on page 1of 69

G.R. No. 167146             October 31, 2006 the National Internal Revenue Code of 19778 (Tax Code of 1977).

al Revenue Code of 19778 (Tax Code of 1977). The dispositive


COMMISSIONER OF INTERNAL REVENUE, petitioner, portion of this decision reads:
vs. PHILIPPINE GLOBAL COMMUNICATION, INC., respondent.
WHEREFORE, premises considered, judgment is hereby rendered in favor
This is a Petition for Review on Certiorari, under Rule 45 of the Rules of Court,
of the petitioner. Accordingly, respondent’s Final Decision dated October 8,
seeking to set aside the en banc Decision of the Court of Tax Appeals (CTA) in CTA
2002 is hereby REVERSED and SET ASIDE and respondent is hereby
EB No. 37 dated 22 February 2005,1 ordering the petitioner to withdraw and cancel
ORDERED to WITHDRAW and CANCEL Assessment Notice No. 000688-80-
Assessment Notice No. 000688-80-7333 issued against respondent Philippine Global
7333 issued against the petitioner for its 1990 income tax deficiency
Communication, Inc. for its 1990 income tax deficiency. The CTA, in its assailed en
because respondent’s right to collect the same has prescribed. 9
banc Decision, affirmed the Decision of the First Division of the CTA dated 9 June
20042 and its Resolution dated 22 September 2004 in C.T.A. Case No. 6568.
The CIR moved for reconsideration of the aforesaid Decision but was denied by the
CTA in a Resolution dated 22 September 2004.10 Thereafter, the CIR filed a Petition
Respondent, a corporation engaged in telecommunications, filed its Annual Income
for Review with the CTA en banc, questioning the aforesaid Decision and Resolution.
Tax Return for taxable year 1990 on 15 April 1991. On 13 April 1992, the
In its en banc Decision, the CTA affirmed the Decision and Resolution in CTA Case
Commissioner of Internal Revenue (CIR) issued Letter of Authority No. 0002307,
No. 6568. The dispositive part reads:
authorizing the appropriate Bureau of Internal Revenue (BIR) officials to examine
the books of account and other accounting records of respondent, in connection
with the investigation of respondent’s 1990 income tax liability. On 22 April 1992, WHEREFORE, premises considered, the Petition for Review is hereby DISMISSED for
the BIR sent a letter to respondent requesting the latter to present for examination lack of merit. Accordingly, the assailed Decision and Resolution in CTA Case No.
certain records and documents, but respondent failed to present any document. On 6568 are hereby AFFIRMED in toto.11
21 April 1994, respondent received a Preliminary Assessment Notice dated 13 April
1994 for deficiency income tax in the amount of P118,271,672.00, inclusive of Hence, this Petition for Review on Certiorari raising the following grounds:
surcharge, interest, and compromise penalty, arising from deductions that were
disallowed for failure to pay the withholding tax and interest expenses that were
likewise disallowed. On the following day, 22 April 1994, respondent received a THE COURT OF TAX APPEALS, SITTING EN BANC, COMMITTED REVERSIBLE ERROR
Formal Assessment Notice with Assessment Notice No. 000688-80-7333, dated 14 IN AFFIRMING THE ASSAILED DECISION AND RESOLUTION IN CTA CASE NO. 6568
April 1994, for deficiency income tax in the total amount of P118,271,672.00.3 DECLARING THAT THE RIGHT OF THE GOVERNMENT TO COLLECT THE DEFICIENCY
INCOME TAX FROM RESPONDENT FOR THE YEAR 1990 HAS PRESCRIBED

On 6 May 1994, respondent, through its counsel Ponce Enrile Cayetano Reyes and
Manalastas Law Offices, filed a formal protest letter against Assessment Notice No. A. THE PRESCRIPTIVE PERIOD WAS INTERUPTED WHEN RESPONDENT FILED TWO
000688-80-7333. Respondent filed another protest letter on 23 May 1994, through LETTERS OF PROTEST DISPUTING IN DETAIL THE DEFICIENCY ASSESSMENT IN
another counsel Siguion Reyna Montecillo & Ongsiako Law Offices. In both letters, QUESTION AND REQUESTING THE CANCELLATION OF SAID ASSESSMENT. THE
respondent requested for the cancellation of the tax assessment, which they alleged TWO LETTERS OF PROTEST ARE, BY NATURE, REQUESTS FOR REINVESTIGATION
was invalid for lack of factual and legal basis. 4 OF THE DISPUTED ASSESSMENT.

On 16 October 2002, more than eight years after the assessment was presumably B. THE REQUESTS FOR REINVESTIGATION OF RESPONDENT WERE GRANTED BY
issued, the Ponce Enrile Cayetano Reyes and Manalastas Law Offices received from THE BUREAU OF INTERNAL REVENUE.12
the CIR a Final Decision dated 8 October 2002 denying the respondent’s protest
against Assessment Notice No. 000688-80-7333, and affirming the said This Court finds no merit in this Petition.
assessment in toto.5
The main issue in this case is whether or not CIR’s right to collect respondent’s
On 15 November 2002, respondent filed a Petition for Review with the CTA. After alleged deficiency income tax is barred by prescription under Section 269(c) of the
due notice and hearing, the CTA rendered a Decision in favor of respondent on 9 Tax Code of 1977, which reads:
June 2004.6 The CTA ruled on the primary issue of prescription and found it
unnecessary to decide the issues on the validity and propriety of the assessment. It
decided that the protest letters filed by the respondent cannot constitute a request Section 269. Exceptions as to the period of limitation of assessment and collection
for reinvestigation, hence, they cannot toll the running of the prescriptive period to of taxes. – x x x
collect the assessed deficiency income tax. 7 Thus, since more than three years had
lapsed from the time Assessment Notice No. 000688-80-7333 was issued in 1994, xxxx
the CIR’s right to collect the same has prescribed in conformity with Section 269 of

1
c. Any internal revenue tax which has been assessed within the period of limitation dissenting opinion, identified the potential loss to the taxpayer if the assessment
above-prescribed may be collected by distraint or levy or by a proceeding in court and collection of taxes are not promptly made.
within three years following the assessment of the tax.
Prescription in the assessment and in the collection of taxes is provided by the
The law prescribed a period of three years from the date the return was actually Legislature for the benefit of both the Government and the taxpayer; for the
filed or from the last date prescribed by law for the filing of such return, whichever Government for the purpose of expediting the collection of taxes, so that the agency
came later, within which the BIR may assess a national internal revenue charged with the assessment and collection may not tarry too long or indefinitely to
tax.13 However, the law increased the prescriptive period to assess or to begin a the prejudice of the interests of the Government, which needs taxes to run it; and
court proceeding for the collection without an assessment to ten years when a false for the taxpayer so that within a reasonable time after filing his return, he may
or fraudulent return was filed with the intent of evading the tax or when no return know the amount of the assessment he is required to pay, whether or not such
was filed at all.14 In such cases, the ten-year period began to run only from the date assessment is well founded and reasonable so that he may either pay the amount of
of discovery by the BIR of the falsity, fraud or omission. the assessment or contest its validity in court x x x. It would surely be prejudicial to
the interest of the taxpayer for the Government collecting agency to unduly delay
the assessment and the collection because by the time the collecting agency finally
If the BIR issued this assessment within the three-year period or the ten-year
gets around to making the assessment or making the collection, the taxpayer may
period, whichever was applicable, the law provided another three years after the
then have lost his papers and books to support his claim and contest that of the
assessment for the collection of the tax due thereon through the administrative
Government, and what is more, the tax is in the meantime accumulating interest
process of distraint and/or levy or through judicial proceedings. 15 The three-year
which the taxpayer eventually has to pay .
period for collection of the assessed tax began to run on the date the assessment
notice had been released, mailed or sent by the BIR.16
In Republic of the Philippines v. Ablaza,19 this Court emphatically explained that the
statute of limitations of actions for the collection of taxes is justified by the need to
The assessment, in this case, was presumably issued on 14 April 1994 since the
protect law-abiding citizens from possible harassment:
respondent did not dispute the CIR’s claim. Therefore, the BIR had until 13 April
1997. However, as there was no Warrant of Distraint and/or Levy served on the
respondents nor any judicial proceedings initiated by the BIR, the earliest attempt The law prescribing a limitation of actions for the collection of the income tax is
of the BIR to collect the tax due based on this assessment was when it filed its beneficial both to the Government and to its citizens; to the Government because
Answer in CTA Case No. 6568 on 9 January 2003, which was several years beyond tax officers would be obliged to act promptly in the making of assessment, and to
the three-year prescriptive period. Thus, the CIR is now prescribed from collecting citizens because after the lapse of the period of prescription citizens would have a
the assessed tax. feeling of security against unscrupulous tax agents who will always find an excuse
to inspect the books of taxpayers, not to determine the latter’s real liability, but to
take advantage of every opportunity to molest, peaceful, law-abiding citizens.
The provisions on prescription in the assessment and collection of national internal
Without such legal defense taxpayers would furthermore be under obligation to
revenue taxes became law upon the recommendation of the tax commissioner of
always keep their books and keep them open for inspection subject to harassment
the Philippines. The report submitted by the tax commission clearly states that
by unscrupulous tax agents. The law on prescription being a remedial measure
these provisions on prescription should be enacted to benefit and protect taxpayers:
should be interpreted in a way conducive to bringing about the beneficient purpose
of affording protection to the taxpayer within the contemplation of the Commission
Under the former law, the right of the Government to collect the tax does not which recommended the approval of the law.
prescribe. However, in fairness to the taxpayer, the Government should be
estopped from collecting the tax where it failed to make the necessary investigation
And again in the recent case Bank of the Philippine Islands v. Commissioner of
and assessment within 5 years after the filing of the return and where it failed to
Internal Revenue,20 this Court, in confirming these earlier rulings, pronounced that:
collect the tax within 5 years from the date of assessment thereof. Just as the
government is interested in the stability of its collections, so also are the taxpayers
entitled to an assurance that they will not be subjected to further investigation for Though the statute of limitations on assessment and collection of national internal
tax purposes after the expiration of a reasonable period of time. (Vol. II, Report of revenue taxes benefits both the Government and the taxpayer, it principally intends
the Tax Commission of the Philippines, pp. 321-322).17 to afford protection to the taxpayer against unreasonable investigation. The
indefinite extension of the period for assessment is unreasonable because it
deprives the said taxpayer of the assurance that he will no longer be subjected to
In a number of cases, this Court has also clarified that the statute of limitations on
further investigation for taxes after the expiration of a reasonable period of time.
the collection of taxes should benefit both the Government and the taxpayers. In
these cases, the Court further illustrated the harmful effects that the delay in the
assessment and collection of taxes inflicts upon taxpayers. In Collector of Internal Thus, in Commissioner of Internal Revenue v. B.F. Goodrich, 21 this Court affirmed
Revenue v. Suyoc Consolidated Mining Company,18 Justice Montemayor, in his that the law on prescription should be liberally construed in order to protect

2
taxpayers and that, as a corollary, the exceptions to the law on prescription should records or newly discovered or additional evidence. A re-evaluation of existing
be strictly construed. records which results from a request for reconsideration does not toll the running of
the prescription period for the collection of an assessed tax. Section 271 distinctly
limits the suspension of the running of the statute of limitations to instances when
The Tax Code of 1977, as amended, provides instances when the running of the
reinvestigation is requested by a taxpayer and is granted by the CIR. The Court
statute of limitations on the assessment and collection of national internal revenue
provided a clear-cut rationale in the case of Bank of the Philippine Islands v.
taxes could be suspended, even in the absence of a waiver, under Section 271
Commissioner of Internal Revenue22 explaining why a request for reinvestigation,
thereof which reads:
and not a request for reconsideration, interrupts the running of the statute of
limitations on the collection of the assessed tax:
Section 224. Suspension of running of statute. – The running of the statute of
limitation provided in Sections 268 and 269 on the making of assessments and the
Undoubtedly, a reinvestigation, which entails the reception and evaluation of
beginning of distraint or levy or a proceeding in court for collection in respect of any
additional evidence, will take more time than a reconsideration of a tax assessment,
deficiency, shall be suspended for the period during which the Commissioner is
which will be limited to the evidence already at hand; this justifies why the former
prohibited from making the assessment or beginning distraint or levy or a
can suspend the running of the statute of limitations on collection of the assessed
proceeding in court and for sixty days thereafter; when the taxpayer requests
tax, while the latter cannot.
for a reinvestigation which is granted by the Commissioner; when the
taxpayer cannot be located in the address given by him in the return filed upon
which a tax is being assessed or collected x x x. (Emphasis supplied.) In the present case, the separate letters of protest dated 6 May 1994 and 23 May
1994 are requests for reconsideration. The CIR’s allegation that there was a request
for reinvestigation is inconceivable since respondent consistently and categorically
Among the exceptions provided by the aforecited section, and invoked by the CIR as
refused to submit new evidence and cooperate in any reinvestigation proceedings.
a ground for this petition, is the instance when the taxpayer requests for a
This much was admitted in the Decision dated 8 October 2002 issued by then CIR
reinvestigation which is granted by the Commissioner. However, this exception does
Guillermo Payarno, Jr.
not apply to this case since the respondent never requested for a reinvestigation.
More importantly, the CIR could not have conducted a reinvestigation where, as
admitted by the CIR in its Petition, the respondent refused to submit any new In the said conference-hearing, Revenue Officer Alameda basically testified that
evidence. Philcom, despite repeated demands, failed to submit documentary evidences in
support of its claimed deductible expenses. Hence, except for the item of interest
expense which was disallowed for being not ordinary and necessary, the rest of the
Revenue Regulations No. 12-85, the Procedure Governing Administrative Protests of
claimed expenses were disallowed for non-withholding. In the same token, Revenue
Assessment of the Bureau of Internal Revenue, issued on 27 November 1985,
Officer Escober testified that upon his assignment to conduct the re-investigation,
defines the two types of protest, the request for reconsideration and the request for
he immediately requested the taxpayer to present various accounting records for
reinvestigation, and distinguishes one from the other in this manner:
the year 1990, in addition to other documents in relation to the disallowed items
(p.171). This was followed by other requests for submission of documents (pp.199
Section 6. Protest. - The taxpayer may protest administratively an assessment by &217) but these were not heeded by the taxpayer. Essentially, he stated that
filing a written request for reconsideration or reinvestigation specifying the following Philcom did not cooperate in his reinvestigation of the case.
particulars:
In response to the testimonies of the Revenue Officers, Philcom thru Atty. Consunji,
For the purpose of protest herein— emphasized that it was denied due process because of the issuance of the Pre-
Assessment Notice and the Assessment Notice on successive dates. x x x Counsel
(a) Request for reconsideration-- refers to a plea for a re-evaluation of an for the taxpayer even questioned the propriety of the conference-hearing inasmuch
assessment on the basis of existing records without need of additional evidence. It as the only question to resolved (sic) is the legality of the issuance of the
may involve both a question of fact or of law or both. assessment. On the disallowed items, Philcom thru counsel manifested that it has
no intention to present documents and/or evidences allegedly because of the
pending legal question on the validity of the assessment. 23
(b) Request for reinvestigation—refers to a plea for re-evaluation of an assessment
on the basis of newly-discovered evidence or additional evidence that a taxpayer
intends to present in the investigation. It may also involve a question of fact or law Prior to the issuance of Revenue Regulations No. 12-85, which distinguishes a
or both. request for reconsideration and a request for reinvestigation, there have been cases
wherein these two terms were used interchangeably. But upon closer examination,
these cases all involved a reinvestigation that was requested by the taxpayer and
The main difference between these two types of protests lies in the records or granted by the BIR.
evidence to be examined by internal revenue officers, whether these are existing

3
In Collector of Internal Revenue v. Suyoc Consolidated Mining Company,24 the Court The second reinvestigation was asked on 16 January 1956, and lasted until it was
weighed the considerable time spent by the BIR to actually conduct the decided on 22 April 1960, or a period of 4 years, 3 months, and 6 days, during
reinvestigations requested by the taxpayer in deciding that the prescription period which the limitation period was interrupted.
was suspended during this time.
The Court reiterated the ruling in Republic v. Lopez in the case of Commissioner of
Because of such requests, several reinvestigations were made and a hearing was Internal Revenue v. Sison,27 "that where a taxpayer demands a reinvestigation, the
even held by the Conference Staff organized in the collection office to consider time employed in reinvestigating should be deducted from the total period of
claims of such nature which, as the record shows, lasted for several months. After limitation." Finally, in Republic v. Arcache,28 the Court enumerated the reasons why
inducing petitioner to delay collection as he in fact did, it is most unfair for the taxpayer is barred from invoking the defense of prescription, one of which was
respondent to now take advantage of such desistance to elude his deficiency income that, "In the first place, it appears obvious that the delay in the collection of his
tax liability to the prejudice of the Government invoking the technical ground of 1946 tax liability was due to his own repeated requests for reinvestigation and
prescription. similarly repeated requests for extension of time to pay."

Although the Court used the term "requests for reconsideration" in reference to the In this case, the BIR admitted that there was no new or additional evidence
letters sent by the taxpayer in the case of Querol v. Collector of Internal presented. Considering that the BIR issued its Preliminary Assessment Notice on 13
Revenue,25 it took into account the reinvestigation conducted soon after these April 1994 and its Formal Assessment Notice on 14 April 1994, just one day before
letters were received and the revised assessment that resulted from the the three-year prescription period for issuing the assessment expired on 15 April
reinvestigations. 1994, it had ample time to make a factually and legally well-founded assessment.
Added to the fact that the Final Decision that the CIR issued on 8 October 2002
merely affirmed its earlier findings, whatever examination that the BIR may have
It is true that the Collector revised the original assessment on February 9, 1955;
conducted cannot possibly outlast the entire three-year prescriptive period provided
and appellant avers that this revision was invalid in that it was not made within the
by law to collect the assessed tax, not to mention the eight years it actually took
five-year prescriptive period provided by law (Collector vs. Pineda, 112 Phil. 321).
the BIR to decide the respondent’s protest. The factual and legal issues involved in
But that fact is that the revised assessment was merely a result of petitioner
the assessment are relatively simple, that is, whether certain income tax deductions
Querol’s requests for reconsideration of the original assessment, contained in his
should be disallowed, mostly for failure to pay withholding taxes. Thus, there is no
letters of December 14, 1951 and May 25, 1953. The records of the Bureau of
reason to suspend the running of the statute of limitations in this case.
Internal Revenue show that after receiving the letters, the Bureau conducted a
reinvestigation of petitioner’s tax liabilities, and, in fact, sent a tax examiner to San
Fernando, La Union, for that purpose; that because of the examiner’s report, the The distinction between a request for reconsideration and a request for
Bureau revised the original assessment, x x x. In other words, the reconsideration reinvestigation is significant. It bears repetition that a request for reconsideration,
was granted in part, and the original assessment was altered. Consequently, the unlike a request for reinvestigation, cannot suspend the statute of limitations on the
period between the petition for reconsideration and the revised assessment should collection of an assessed tax. If both types of protest can effectively interrupt the
be subtracted from the total prescriptive period (Republic vs. Ablaza, 108 Phil running of the statute of limitations, an erroneous assessment may never prescribe.
1105). If the taxpayer fails to file a protest, then the erroneous assessment would become
final and unappealable.29 On the other hand, if the taxpayer does file the protest on
a patently erroneous assessment, the statute of limitations would automatically be
The Court, in Republic v. Lopez,26 even gave a detailed accounting of the time the
suspended and the tax thereon may be collected long after it was assessed.
BIR spent for each reinvestigation in order to deduct it from the five-year period set
Meanwhile the interest on the deficiencies and the surcharges continue to
at that time in the statute of limitations:
accumulate. And for an unrestricted number of years, the taxpayers remain
uncertain and are burdened with the costs of preserving their books and records.
It is now a settled ruled in our jurisdiction that the five-year prescriptive period This is the predicament that the law on the statute of limitations seeks to prevent.
fixed by Section 332(c) of the Internal Revenue Code within which the Government
may sue to collect an assessed tax is to be computed from the last revised
The Court, in sustaining for the first time the suspension of the running of the
assessment resulting from a reinvestigation asked for by the taxpayer and (2) that
statute of limitations in cases where the taxpayer requested for a reinvestigation,
where a taxpayer demands a reinvestigation, the time employed in reinvestigating
gave this justification:
should be deducted from the total period of limitation.

A taxpayer may be prevented from setting up the defense of prescription even if he


The first reinvestigation was granted, and a reduced assessment issued on 29 May
has not previously waived it in writing as when by his repeated requests or positive
1954, from which date the Government had five years for bringing an action to
acts the Government has been, for good reasons, persuaded to postpone collection
collect.
to make him feel that the demand was not unreasonable or that no
harassment or injustice is meant by the Government.

4
This case has no precedent in this jurisdiction for it is the first time that such has prescription period to, among other instances, protests wherein the taxpayer
risen, but there are several precedents that may be invoked in American requests for a reinvestigation. In this case, where the taxpayer merely filed two
jurisprudence. As Mr. Justice Cardozo has said: "The applicable principle is protest letters requesting for a reconsideration, and where the BIR could not have
fundamental and unquestioned. ‘He who prevents a thing from being done may conducted a reinvestigation because no new or additional evidence was submitted,
not avail himself of the nonperformance which he himself occasioned, for the running of statute of limitations cannot be interrupted. The tax which is the
the law says to him in effect "this is your own act, and therefore you are subject of the Decision issued by the CIR on 8 October 2002 affirming the Formal
not damnified."’ (R.H. Stearns Co. v. U.S., 78 L. ed., 647). (Emphasis supplied.) 30 Assessment issued on 14 April 1994 can no longer be the subject of any proceeding
for its collection. Consequently, the right of the government to collect the alleged
deficiency tax is barred by prescription.
This rationale is not applicable to the present case where the respondent did
nothing to prevent the BIR from collecting the tax. It did not present to the BIR any
new evidence for its re-evaluation. At the earliest opportunity, respondent insisted IN VIEW OF THE FOREGOING, the instant Petition is DENIED. The assailed en
that the assessment was invalid and made clear to the BIR its refusal to produce banc Decision of the CTA in CTA EB No. 37 dated 22 February 2005, cancelling
documents that the BIR requested. On the other hand, the BIR also communicated Assessment Notice No. 000688-80-7333 issued against Philippine Global
to the respondent its unwavering stance that its assessment is correct. Given that Communication, Inc. for its 1990 income tax deficiency for the reason that it is
both parties were at a deadlock, the next logical step would have been for the BIR barred by prescription, is hereby AFFIRMED. No costs.
to issue a Decision denying the respondent’s protest and to initiate proceedings for
the collection of the assessed tax and, thus, allow the respondent, should it so
SO ORDERED.
choose, to contest the assessment before the CTA. Postponing the collection for
eight long years could not possibly make the taxpayer feel that the demand was not
unreasonable or that no harassment or injustice is meant by the Government. There CIR v. PHILIPPINE GLOBAL COMMUNICATION, GR NO. 167146, 2006-10-31
was no legal, or even a moral, obligation preventing the CIR from collecting the
assessed tax. In a similar case, Cordero v. Conda,31 the Court did not suspend the Facts:
running of the prescription period where the acts of the taxpayer did not prevent
Respondent, a corporation engaged in telecommunications, filed its Annual Income
the government from collecting the tax.
Tax Return for taxable year 1990... on 15 April 1991

The government also urges that partial payment is "acknowledgement of the tax On 13 April 1992, the Commissioner of Internal Revenue (CIR) issued Letter of
obligation", hence a "waiver on the defense of prescription." But partial payment Authority No. 0002307, authorizing the appropriate Bureau of Internal
would not prevent the government from suing the taxpayer. Because, by such act of
payment, the government is not thereby "persuaded to postpone collection to make Revenue (BIR) officials to examine the books of account and other accounting
him feel that the demand was not unreasonable or that no harassment or injustice records of respondent, in connection with the investigation of respondent's 1990
is meant." Which, as stated in Collector v. Suyoc Consolidated Mining Co., et al., L- income tax liability.
11527, November 25, 1958, is the underlying reason behind the rule that
On 21 April 1994, respondent received a Preliminary Assessment Notice dated 13
prescriptive period is arrested by the taxpayer’s request for reexamination or
April 1994 for deficiency income tax
reinvestigation – even if "he has not previously waived it [prescription] in writing."
On the following day, 22 April 1994, respondent received a Formal Assessment
The Court reminds us, in the case of Commissioner of Internal Revenue v. Algue, Notice... respondent... filed a formal protest letter against Assessment Notice
Inc., 32 of the need to balance the conflicting interests of the government and the
taxpayers. On 16 October 2002, more than eight years after the assessment was presumably
issued... received from the CIR a Final Decision... denying the respondent's protest

Taxes are the lifeblood of the government and so should be collected without On 15 November 2002, respondent filed a Petition for Review with the CTA. After
unnecessary hindrance. On the other hand, such collection should be made in due notice and hearing, the CTA rendered a Decision in favor of respondent on 9
accordance with law as any arbitrariness will negate the very reason for government June 2004.
itself. It is therefore necessary to reconcile the apparently conflicting interest of the
authorities and the taxpayers so that the real purpose of taxation, which is the Thus, since more than three years had lapsed from the time Assessment Notice No.
promotion of common good, may be achieved. 000688-80-7333 was issued in 1994, the CIR's right to collect the same has
prescribed
Thus, the three-year statute of limitations on the collection of an assessed tax It decided that the protest letters filed by the respondent cannot constitute a
provided under Section 269(c) of the Tax Code of 1977, a law enacted to protect request for reinvestigation, hence, they cannot toll the running of the prescriptive
the interests of the taxpayer, must be given effect. In providing for exceptions to period to collect the assessed deficiency... income tax.[7] Thus, since more than
such rule in Section 271, the law strictly limits the suspension of the running of the

5
three years had lapsed from the time Assessment Notice No. 000688-80-7333 was Pursuant to such reinsurance treaties, Phoenix Assurance Co., Ltd., ceded portions
issued in 1994, the CIR's right to collect the same has prescribed of the premiums it earned from its underwriting business in the Philippines, as
follows:
Issues:

whether or not CIR's right to collect respondent's alleged deficiency income tax is Year Amount Ceded
barred by prescription... when the taxpayer requests for a reinvestigation which is
granted by the Commissioner. 1952 P316,526.75

Ruling: 1953 P246,082.04

the law provided another three years after the assessment for the collection of the 1954 P203,384.69
tax... due

The three-year period for collection of the assessed tax began to run on the date upon which the Commissioner of Internal Revenue, by letter of May 6, 1958,
the assessment notice had been released assessed the following withholding tax:

The assessment, in this case, was presumably issued on 14 April 1994 since the
respondent did not dispute the CIR's claim. Therefore, the BIR had until 13 April Year Withholding Tax
1997. However, as there was no Warrant of Distraint and/or Levy served on the
respondents nor any judicial proceedings... initiated by the BIR, the earliest attempt 1952 P 75,966.42
of the BIR to collect the tax due based on this assessment was when it filed its
1953 59,059.68
Answer in CTA Case No. 6568 on 9 January 2003, which was several years beyond
the three-year prescriptive period. Thus, the CIR is now prescribed from... collecting 1954 48,812.32
the assessed tax.

this case since the... respondent never requested for a reinvestigation Total P183,838.42
G.R. No. L-19727             May 20, 1965 =============
THE COMMISSIONER OF INTERNAL REVENUE, petitioner,
vs. PHOENIX ASSURANCE CO., LTD., respondent.
On April 1, 1951, Phoenix Assurance Co., Ltd. filed its Philippine income tax return
for 1950, claiming therein, among others, a deduction of P37,147.04 as net addition
----------------------------- to marine insurance reserve equivalent to 40% of the gross marine insurance
premiums received during the year. The Commissioner of Internal Revenue
G.R. No. L-19903             May 20, 1965 disallowed P11,772.57 of such claim for deduction and subsequently assessed
PHOENIX ASSURANCE, CO., LTD., petitioner, against Phoenix Assurance Co., Ltd. the sum of P1,884.00 as deficiency income tax.
vs. COMMISSIONER OF INTERNAL REVENUE, respondent. The disallowance resulted from the fixing by the Commissioner of the net addition to
the marine insurance reserve at 100% of the marine insurance premiums received
during the last three months of the year. The Commissioner assumed that "ninety
From a judgment of the Court of Tax Appeals in C.T.A. Cases Nos. 305 and 543, and third, days are approximately the length of time required before shipments
consolidated and jointly heard therein, these two appeals were taken. Since they reach their destination or before claims are received by the insurance companies."
involve the same facts and interrelated issues, the appeals are herein decided
together.
On April 1, 1953, Phoenix Assurance Co., Ltd. filed its Philippine income tax return
for 1952, declaring therein a deduction from gross income of P35,912.25 as part of
Phoenix Assurance Co., Ltd., a foreign insurance corporation organized under the the head office expenses incurred for its Philippine business, computed at 5% on its
laws of Great Britain, is licensed to do business in the Philippines with head office in gross Philippine income.
London. Through its head office, it entered in London into worldwide reinsurance
treaties with various foreign insurance companies. It agree to cede a portion of
premiums received on original insurances underwritten by its head office, On August 30, 1955 it amended its income tax return for 1952 by excluding from its
subsidiaries, and branch offices throughout the world, in consideration for gross income the amount of P316,526.75 representing reinsurance premiums ceded
assumption by the foreign insurance companies of an equivalent portion of the to foreign reinsurers and further eliminating deductions corresponding to the coded
liability from such original insurances.1äwphï1.ñët premiums. The amended return showed an income tax due in the amount of
P2,502.00. The Commissioner of Internal Revenue disallowed P15,826.35 of the

6
claimed deduction for head office expenses and assessed a deficiency tax of Overclaimed Head Office expenses:
P5,667.00 on July 24, 1958.
Amount claimed . . . . . . . . . . . . P29,624.73
On April 30, 1954, Phoenix Assurance Co., Ltd. filed its Philippine income tax return
Amount allowed . . . . . . . . . . . . 19,455.50 10,16.23
for 1953 and claimed therein a deduction from gross income of P33,070.88 as head
office expenses allocable to its Philippine business, equivalent to 5%, of its gross
Philippine income. On August 30, 1955 it amended its 1953 income tax return to
exclude from its gross income the amount of P246,082.04 representing reinsurance Net income per investigation P170,489.41
premiums ceded to foreign reinsurers. At the same time, it requested the refund of
P23,409.00 as overpaid income tax for 1953. To avoid the prescriptive period
provided for in Section 306 of the Tax Code, it filed a petition for review on April 11, Tax due thereon P 39,737.00
1956 in the Court of Tax Appeals praying for such refund. After verification of the
Less: amount already assessed 36,890.00
amended income tax return the Commissioner of Internal Revenue disallowed
P12,304.10 of the deduction representing head office expenses allocable to
Philippine business thereby reducing the refundable amount to P20,180.00.
DEFICIENCY TAX DUE P 2,847.00
===========
On April 29, 1955, Phoenix Assurance Co., Ltd. filed its Philippine income tax return
for 1954 claiming therein, among others, a deduction from gross income of
P99,624.75 as head office expenses allocable to its Philippine business, computed at The above assessment resulted from the disallowance of a portion of the deduction
5% of its gross Philippine income. It also excluded from its gross income the claimed by Phoenix Assurance Co., Ltd. as head office expenses allocable to its
amount of P203,384.69 representing reinsurance premiums ceded to foreign business in the Philippines fixed by the Commissioner at 5% of the net Philippine
reinsurers not doing business in the Philippines. income instead of 5% of the gross Philippine income as claimed in the returns.

On August 1, 1958 the Bureau of Internal Revenue released the following Phoenix Assurance Co., Ltd. protested against the aforesaid assessments for
assessment for deficiency income tax for the years 1952 and 1954 against Phoenix withholding tax and deficiency income tax. However, the Commissioner of Internal
Assurance Co., Ltd.: Revenue denied such protest. Subsequently, Phoenix Assurance Co., Ltd. appealed
to the Court of Tax Appeals. In a decision dated February 14, 1962, the Court of
Tax Appeals allowed in full the decision claimed by Phoenix Assurance Co., Ltd. for
1952 1950 as net addition to marine insurance reserve; determined the allowable head
office expenses allocable to Philippine business to be 5% of the net income in the
Net income per audited return P 12,511.61
Philippines; declared the right of the Commissioner of Internal Revenue to assess
Unallowable deduction & additional income: deficiency income tax for 1952 to have prescribed; absolved Phoenix Assurance Co.,
Ltd. from payment of the statutory penalties for non-filing of withholding tax return;
Overclaimed Head Office expenses: and, rendered the following judgment:

Amount claimed . . . . . . . . . . . . P 35,912.25


WHEREFORE, petitioner Phoenix Assurance Company, Ltd. is hereby
Amount allowed . . . . . . . . . . . . 20,085.90 P 15,826.35 ordered to pay the Commissioner of Internal Revenue the respective
amounts of P75,966.42, P59,059.68 and P48,812.32, as withholding tax
for the years 1952, 1953 and 1954, and P2,847.00 as income tax for 1954,
Net income per investigation P 28,337.96 or the total sum of P186,685.42 within thirty (30) days from the date this
decision becomes final. Upon the other hand, the respondent
Commissioner is ordered to refund to petitioner the sum of P20,180.00 as
overpaid income tax for 1953, which sum is to be deducted from the total
Tax due thereon P 5,667.00
sum of P186,685.42 due as taxes.
===========

1954 If any amount of the tax is not paid within the time prescribed above, there
shall be collected a surcharge of 5% of the tax unpaid, plus interest at the
Net income per audited P160,320.21
rate of 1% a month from the date of delinquency to the date of payment,
Unallowable deduction & additional income: provided that the maximum amount that may be collected as interest shall

7
not exceed the amount corresponding to a period of three (3) years. The Court of Tax Appears that the original return was a complete return containing
Without pronouncement as to costs. "information on various items of income and deduction from which respondent may
intelligently compute and determine the tax liability of petitioner, hence, the
prescriptive period should be counted from the filing of said original return. On the
Phoenix Assurance Co., Ltd. and the Commissioner of Internal Revenue have
other hand, the Commissioner of Internal Revenue maintains that:
appealed to this Court raising the following issues: (1) Whether or not reinsurance
premiums ceded to foreign reinsurers not doing business in the Philippines pursuant
to reinsurance contracts executed abroad are subject to withholding tax; (2) "... the deficiency income tax in question could not possibly be determined, or
Whether or not the right of the Commissioner of Internal Revenue to assess assessed, on the basis of the original return filed on April 1, 1953, for considering
deficiency income tax for the year 1952 against Phoenix Assurance Co., Ltd., has that the declared loss amounted to P199,583.93, the mere disallowance of part of
prescribed; (3) Whether or not the deduction of claimed by the Phoenix Assurance the head office expenses could not probably result in said loss being completely
Co., Ltd.as net addition to reserve for the year 1950 is excessive; (4) Whether or wiped out and Phoenix being liable to deficiency tax. Not until the amended return
not the deductions claimed by Phoenix Assurance Co., Ltd. for head office expenses was filed on August 30, 1955 could the Commissioner assess the deficiency income
allocable to Philippine business for the years 1952, 1953 and 1954 are excessive. tax in question."

The question of whether or not reinsurance premiums ceded to foreign reinsurers Accordingly, he would wish to press for the counting of the prescriptive period from
not doing business in the Philippines pursuant to contracts executed abroad are the filing of the amended return.
income from sources within the Philippines subject to withholding tax under
Sections 53 and 54 of the Tax Code has already been resolved in the affirmative
To our mind, the Commissioner's view should be sustained. The changes and
in British Traders' Insurance Co., Ltd.v. Commisioner of Internal Revenue, L-20501,
alterations embodied in the amended income tax return consisted of the exclusion
April 30, 1965. 1
of reinsurance premiums received from domestic insurance companies by Phoenix
Assurance Co., Ltd.'s London head office, reinsurance premiums ceded to foreign
We come to the issue of prescription. Phoenix Assurance Co., Ltd. filed its income reinsurers not doing business in the Philippines and various items of deduction
tax return for 1952 on April 1, 1953 showing a loss of P199,583.93. It amended attributable to such excluded reinsurance premiums thereby substantially modifying
said return on August 30, 1955 reporting a tax liability of P2,502.00. On July 24, the original return. Furthermore, although the deduction for head office expenses
1958, after examination of the amended return, the Commissioner of Internal allocable to Philippine business, whose disallowance gave rise to the deficiency tax,
Revenue assessed deficiency income tax in the sum of P5,667.00. The Court of Tax was claimed also in the original return, the Commissioner could not have possibly
Appeals found the right of the Commissioner of Internal Revenue barred by determined a deficiency tax thereunder because Phoenix Assurance Co., Ltd.
prescription, the same having been exercised more than five years from the date declared a loss of P199,583.93 therein which would have more than offset such
the original return was filed. On the other hand, the Commissioner of Internal disallowance of P15,826.35. Considering that the deficiency assessment was based
Revenue insists that his right to issue the assessment has not prescribed inasmuch on the amended return which, as aforestated, is substantially different from the
as the same was availed of before the 5-year period provided for in Section 331 of original return, the period of limitation of the right to issue the same should be
the Tax Code expired, counting the running of the period from August 30, 1955, the counted from the filing of the amended income tax return. From August 30, 1955,
date when the amended return was filed. when the amended return was filed, to July 24, 1958, when the deficiency
assessment was issued, less than five years elapsed. The right of the Commissioner
to assess the deficiency tax on such amended return has not prescribed.
Section 331 of the Tax Code, which limits the right of the Commissioner of Internal
Revenue to assess income tax within five years from the Filipino of the income tax
return, states: To strengthen our opinion, we believe that to hold otherwise, we would be paving
the way for taxpayers to evade the payment of taxes by simply reporting in their
original return heavy losses and amending the same more than five years later
SEC. 331. Period of limitation upon assessment and collection. — Except as
when the Commissioner of Internal Revenue has lost his authority to assess the
provided in the succeeding section internal revenue taxes shall be assessed
proper tax thereunder. The object of the Tax Code is to impose taxes for the needs
within five years after the return was filed, and no proceeding in court
of the Government, not to enhance tax avoidance to its prejudice.
without assessment for the collection of such taxes shall be begun after the
expiration of such period. For the purposes of this section, a return filed
before the last day prescribed by law for the filing thereof shall be We next consider Phoenix Assurance Co., Ltd.'s claim for deduction of P37,147.04
considered as filed on such last day: Provided, That this limitation shall not for 1950 representing net addition to reserve computed at 40% of the marine
apply to cases already investigated prior to the approval of this Code. insurance premiums received during the year. Treating said said deduction to be
excessive, the Commissioner of Internal Revenue reduced the same to P25,374.47
which is equivalent to 100% of all marine insurance premiums received during the
The question is: Should the running of the prescriptive period commence from the
last months of the year.
filing of the original or amended return?

8
Paragraph (a) of Section 32 of the Tax Code states: on the net income. What, therefore, needs to be resolved is: Should the 5% be
computed on the gross or net income?
SEC. 32. Special provisions regarding income and deductions of insurance
companies, whether domestic or foreign. — (a) Special deductions allowed to The record shows that the gross income of Phoenix Assurance Co., Ltd. consists of
insurance companies. — In the case of insurance companies, except domestic life income from its Philippine business as well as reinsurance premiums received for its
insurance companies and foreign life insurance companies doing business in the head office in London and reinsurance premiums ceded to foreign reinsurance. Since
Philippines, the net additions, if any, required by law to be made within the year to the items of income not belonging to its Philippine business are not taxable to its
reserve funds and the sums other than dividends paid within the year on policy and Philippine branch, they should be excluded in determining the head office expenses
annuity contracts may be deducted from their gross income: Provided, however, allowable to said Philippine branch. This conclusion finds support in paragraph 2,
That the released reserve be treated as income for the year of release. subsection (a), Section 30 of the Tax Code, quoted hereunder:

Section 186 of the Insurance Law requires the setting up of reserves for liability on (2) Expenses allowable to non-resident alien individuals and foreign corporations. In
marine insurance: the case of a non-resident alien individual or a foreign corporation, the expenses
deductible are the, necessary expenses paid or incurred in carrying on any business
or trade conducted within the Philippines exclusively. (Emphasis supplied.)
SEC. 186. ... Provided, That for marine risks the insuring company shall be required
to charge as the liability for reinsurance fifty per centum of the premiums written in
the policies upon yearly risks, and the full premiums written in the policies upon all Consequently, the deficiency assessments for 1952, 1953 and 1954, resulting from
other marine risks not terminated (Emphasis supplied.) partial disallowance of deduction representing head office expenses, are sustained.

The reserve required for marine insurance is determined on two bases: 50% of Finally, the Commissioner of Internal Revenue assails the dispositive portion of the
premiums under policies on yearly risks and 100% of premiums under policies of Tax Court's decision limiting the maximum amount of interest collectible for
marine risks not terminated during the year. Section 32 (a) of the Tax Code quoted deliquency of an amount corresponding to a period of three years. He contends that
above allows the full amount of such reserve to be deducted from gross income. since such limitation was incorporated into Section 51 of the Tax Code by Republic
Act 2343 which took effect only on June 20, 1959, it must not be applied
retroactively on withholding tax for the years 1952, 1953 and 1954.
It may be noteworthy to observe that the formulas for determining the marine
reserve employed by Phoenix Assurance Co., Ltd. and the Commissioner of Internal
Revenue — 40% of premiums received during the year and 100% of premiums The imposition of interest on unpaid taxes is one of the statutory penalties for tax
received during the last three months of the year, respectively — do not comply delinquency, from the payments of which the Court of Tax Appeals absolved the
with Section 186. Said determination runs short of the requirement. For purposes of Phoenix Assurance Co., Ltd. on the equitable ground that the latter's failure to pay
the Insurance Law, this Court therefore cannot countenance the same. The reserve the withholding tax was due to the Commissioner's opinion that no withholding tax
called for in Section 186 is a safeguard to the general public and should be strictly was due. Consequently, the taxpayer could be held liable for the payment of
followed not only because it is an express provision but also as a matter of public statutory penalties only upon its failure to comply with the Tax Court's judgment
policy. However, for income tax purposes a taxpayer is free to deduct from its gross rendered on February 14. 1962, after Republic Act 2343 took effect. This part of the
income a lesser amount, or not to claim any deduction at all. What is prohibited by ruling of the lower court ought not to be disturbed.
the income tax law is to claim a deduction beyond the amount authorized therein.
WHEREFORE, the decision appealed from is modified, Phoenix Assurance Co., Ltd. is
Phoenix Assurance Co., Ltd.'s claim for deduction of P37,147.04 being less than the hereby ordered to pay the Commissioner, of Internal Revenue the amount of
amount required in Section 186 of the Insurance Law, the same cannot be and is P75,966.42, P59,059.68 and P48,812.32 as withholding tax for the years 1952,
not excessive, and should therefore be fully allowed. * 1953 and 1954, respectively, and the sums of P5,667.00 and P2,847.00 as income
tax for 1952 and 1954 or a total of P192,352.42. The Commissioner of Internal
Revenue is ordered to refund to Phoenix Assurance Co., Ltd. the amount of
We come now to the controversy on the taxpayer's claim for deduction on head
P20,180.00 as overpaid income tax for 1953, which should be deducted from the
office expenses incurred during 1952, 1953, and 1954 allocable to its Philippine
amount of P192,352.42.
business computed at 5% of its gross income in the Philippines The Commissioner
of Internal Revenue redetermined such deduction at 5% on Phoenix Assurance Co.,
Ltd's net income thereby partially disallowing the latter's claim. The parties are If the amount of P192,352.42 or a portion thereof is not paid within thirty (30) days
agreed as to the percentage — 5% — but differ as to the basis of computation. from the date this judgment becomes final, there should be collected a surcharge
Phoenix Assurance Co. Lt. insists that the 5% head office expenses be determined and interest as provided for in Section 51(c) (2) of the Tax Code. No costs. It is so
from the gross income, while the Commissioner wants the computation to be made ordered.

9
G.R. No. L-20601             February 28, 1966 The issues presented in this appeal are: whether or not petitioner herein is liable to
BUTUAN SAWMILL, INC., petitioner, vs. HON. COURT OF TAX APPEALS, ET pay the 5% sales tax as then prescribed by Section 186 of the Tax Code on its sales
AL., respondents. of logs to the Japanese buyers; and whether or not the assessment thereof was
made within the prescriptive period provided by law therefor.1äwphï1.ñët
Appeal from a decision of the Court of Tax Appeals, in its CTA Case No. 965,
ordering petitioner herein, Butuan Sawmill, Inc., to pay respondent Commissioner of On the first issue, petitioner herein insists that the circumstances enumerated in the
Internal Revenue the sum of P36,107.74 as deficiency sales tax and surcharge due above finding, which this Court had, in previous decisions (Cf. footnote [1]),
on its sales of logs to buyers in Japan from January 31, 1951 to June 8, 1953. considered as determinative of the place of transfer of ownership of the logs sold,
for purposes of taxation, are not in themselves evidentiary indications to show that
the parties intended the title of the logs to pass to the Japanese buyers in Japan.
The facts, as found and stated by the lower court in its decision, are in full accord
Thus, it points out that the "FOB" feature of the sales contract was made only to fix
with the evidences presented therein; hence, we quote them hereunder:
its price and not to fix the place of delivery; that the requirement of certification of
quality, quantity, and measurement specifications of the logs by local authorities
. . . that during the period from January 31, 1951 to June 8, 1953, it sold logs to was done to comply with local laws, rules, and regulations, and was not a part of
Japanese firms at prices FOB Vessel Magallanes, Agusan (in some cases FOB Vessel, the sales arrangement; that the payment of freight by the Japanese buyers is not
Nasipit, also in Agusan); that the FOB prices included costs of loading, wharfage an uncommon feature of "FOB" shipments; and that the payment of prices by
stevedoring and other costs in the Philippines; that the quality, quantity and means of irrevocable letters of credit is but a common established business practice
measurement specifications of the logs were certified by the Bureau of Forestry; to secure payment of the price to the seller. It also insists that, even assuming that
that the freight was paid by the Japanese buyers; and the payments of the logs the "FOB" feature of the disputed sales determines the situs of transfer of
were effected by means of irrevocable letters of credit in favor of petitioner and ownership, the same is merely a prima facie presumption which yields to contrary
payable through the Philippine National Bank or any other bank named by it. proof such as that the logs were made deliverable to the "order of the shipper" and
the logs were shipped at the risk of the shipper, which circumstances, if considered,
Upon investigation by the Bureau of Internal Revenue, it was ascertained that no would negate the above implications. Hence, petitioner herein contends that the
sales tax return was filed by the petitioner and neither did it pay the corresponding disputed sales were consummated in Japan, and, therefore, not subject to the
tax on the sales. On the basis of agent Antonio Mole's report dated September 17, taxing jurisdiction of our Government.
1957, respondent, on August 27, 1958, determined against petitioner the sum of
P40,004.01 representing sales tax, surcharge and compromise penalty on its sales The above contentions of petitioner are devoid of merit. In a decided case with
[tax, surcharge and compromise penalty on its sales] of logs from January 1951 to practically identical set of facts obtaining in the case at bar, this Court declared:
June 1953 pursuant to Sections 183, 186 and 209 of the National Internal Revenue
Code (Exhibit "E", p. 14, CTA rec. & p. 14, BIR rec.). And in consequence of a
. . . it is admitted that the agreed price was "F.O.B. Agusan", thus indicating,
reinvestigation, respondent, on November 6, 1958, amended the amount of the
although prima facie, that the parties intended the title to pass to the buyer upon
previous assessment to P38,917.74 (Exh. "F", p. 52, BIR rec.). Subsequent requests
delivery of the logs in Agusan; on board the vessels that took the goods to Japan.
for reconsideration of the amended assessment having been denied (Exh. "G", p.
Moreover, said prima facie proof was bolstered up by the following circumstances,
55, BIR rec.; Exh. "H", pp. 75-76, BIR rec.: Exh. "I", pp. 79-80, BIR rec.; Exh. "J",
namely:
p. 81, BIR rec.), petitioner filed the instant petition for review on November 7,
1960.
1. Irrevocable letters of credit were opened by the Japanese buyers in favor of the
petitioners.
On the bases of the above-quoted findings and circumstances, the lower court
upheld the legality and correctness of the amended assessment of the sales tax and
surcharge, ruling that the sales in question, in the light of our previous decisions 1, 2. Payment of freight charges of every shipment by the Japanese buyers.
were domestic or "local" sales, and, therefore, subject to sales tax under the
provision of section 186 of the Tax Code, as amended by Republic Acts Nos. 558
3. The Japanese buyers chartered the ships that carried the logs they purchased
and 594; and that the assessment thereof was made well within the ten-year period
from the Philippines to Japan.
prescribed by Section 332(a) of the same Code, since petitioner herein omitted to
file its sales tax returns for the years 1951, 1952 and 1953, and this omission was
discovered only on September 17, 1957. The imposition of the compromise penalty 4. The Japanese buyers insured the shipment of logs and collected the insurance
was, however, eliminated therefrom for want of agreement between the taxpayer coverage in case of loss in transit.
and the Collector (now Commissioner) of Internal Revenue. A motion to reconsider
said decision having been denied, petitioner herein interposed the present appeal 5. The petitioner collected the purchase price of every shipment of logs by
before this Court. surrendering the covering letter of credit, bill of lading, which was indorsed in blank,

10
tally sheet, invoice and export entry, to the corresponding bank in Manila of the five-year prescriptive period within which to make an assessment and collection of
Japanese agent bank with whom the Japanese buyers opened letters of credit. the tax in question from the time the return was deemed filed, should be applied to
the case at bar. Since petitioner filed its income tax returns for the years 1951,
1952 and 1953, and the assessment was made in 1957 only, it further contends
6. In case of natural defects in logs shipped to the buyers discovered in Japan,
that the assessment of the sales tax corresponding to the years 1951 and 1952 has
instead of returning such defective logs, accepted them, but were granted a
already prescribed for having been made outside the five-year period prescribed in
corresponding credit based on the contract price.
Section 331 of the Tax Code and should, therefore, be deducted from the
assessment of the deficiency sales tax made by respondent.
7. The logs purchased by the Japanese buyers were measured by a representative
of the Director of Forestry and such measurement was final, thereby making the
The above contention has already been raised and rejected as not meritorious in a
Government of the Philippines a sort of agent of the Japanese buyers.
previous case decided by this Court. Thus, we held that an income tax return cannot
be considered as a return for compensating tax for purposes of computing the
Upon the foregoing facts and authority of Bislig (Bay) Lumber Co., Inc. vs. Collector period of prescription under Section 331 of the Tax Code, and that the taxpayer
of Internal Revenue, G.R. No. L-13186 (January 28, 1961), Misamis Lumber Co., must file a return for the particular tax required by law in order to avail himself of
Inc. vs. Collector of Internal Revenue (56 Off. Gaz. 517) and Western Mindanao the benefits of Section 331 of the Tax Code; otherwise, if he does not file a return,
Lumber Development Co., Inc. vs. Court of Tax Appeals, et al. (G.R. No. L-11710, an assessment may be made within the time stated in Section 332(a) of the same
June 30, 1958), it is clear that said export sales had been consummated in the Code (Bisaya Land Transportation Co., Inc. vs. Collector of Internal Revenue &
Philippines and were, accordingly, subject to sales tax therein." (Taligaman Lumber Collector of Internal Revenue vs. Bisaya Land Transportation Co., Inc., G.R. Nos. L-
Co., Inc. vs. Collector of Internal Revenue, G.R. No. L-15716, March 31, 1962). 12100 & L-11812, May 29, 1959). The principle enunciated in this last cited case is
applicable by analogy to the case at bar.
With respect to petitioner's contention that there are proofs to rebut the prima
facie finding and circumstances that the disputed sales were consummated here in It being undisputed that petitioner failed to file a return for the disputed sales
the Philippines, we find that the allegation is not borne out by the law or the corresponding to the years 1951, 1952 and 1953, and this omission was discovered
evidence. only on September 17, 1957, and that under Section 332(a) of the Tax Code
assessment thereof may be made within ten (10) years from and after the
That the specification in the bill of lading to the effect that the goods are deliverable discovery of the omission to file the return, it is evident that the lower court
to the order of the seller or his agent does not necessarily negate the passing of correctly held that the assessment and collection of the sales tax in question has
title to the goods upon delivery to the carrier is clear from the second part of not yet prescribed.
paragraph 2 of Article 1503 of the Civil Code of the Philippines (which appellant's
counsel improperly omits from his citation): G.R. No. L-20569 August 23, 1974
JOSE B. AZNAR, in his capacity as Administrator of the Estate of the
Where goods are shipped, and by the bill of lading the goods are deliverable to the deceased, Matias H. Aznar, petitioner,
seller or his agent, or to the order of the seller or of his agent, the seller thereby vs. COURT OF TAX APPEALS and COLLECTOR OF INTERNAL
reserves the ownership in the goods. But, if except for the form of the bill of lading, REVENUE, respondents.
the ownership would have passed to the buyer on shipment of the goods, the
sellers's property in the goods shall be deemed to be only for the purpose of Sato, Enad Garcia for petitioner.
securing performance by the buyer of his obligations under the contract.
Office of the Solicitor General Arturo A. Alafriz, Solicitor Alejandro B. Afurong and
Moreover, it has been "a settled rule that in petitions to review decisions of the Special Attorney Librada R. Natividad for respondents.
Court of Tax Appeals, only questions of law may be raised and may be passed upon
by this Court" (Gutierrez vs. Court of Tax Appeals & Collector of Internal Revenue
vs. Gutierrez, G.R. Nos. L-7938 & L-9771, May 21, 1957, cited in Sanchez vs.
Commissioner of Customs, G.R. No. L-8556, September 30, 1957); and it having
been found that there is no proof to substantiate the foregoing contention of ESGUERRA, J.:p
petitioner, the same should also be ruled as devoid of merit.
Petitioner, as administrator of the estate of the deceased, Matias H. Aznar, seeks a
On the second issue, petitioner avers that the filing of its income tax returns, review and nullification of the decision of the Court of Tax Appeals in C.T.A. Case
wherein the proceeds of the disputed sales were declared, is substantial compliance No. 109, modifying the decision of respondent Commissioner of Internal Revenue
with the requirement of filing a sales tax return, and, if there should be deemed a and ordering the petitioner to pay the government the sum of P227,691.77
return filed, Section 331, and not Section 332(a), of the Tax Code providing for a representing deficiency income taxes for the years 1946 to 1951, inclusive, with the
11
condition that if the said amount is not paid within thirty days from the date the
195 6,800. none 43
decision becomes final, there shall be added to the unpaid amount the surcharge of
00 (pp.
5%, plus interest at the rate of 12% per annum from the date of delinquency to the
54-57
date of payment, in accordance with Section 51 of the National Internal Revenue
BIR
Code, plus costs against the petitioner.
rec.).

It is established that the late Matias H. Aznar who died on May 18, 1958,
predecessor in interest of herein petitioner, during his lifetime as a resident of Cebu The Commissioner of Internal Revenue having his doubts on the veracity of the
City, filed his income tax returns on the cash and disbursement basis, reporting reported income of one obviously wealthy, pursuant to the authority granted him by
therein the following: Section 38 of the National Internal Revenue Code, caused B.I.R. Examiner Honorio
Guerrero to ascertain the taxpayer's true income for said years by using the net
worth and expenditures method of tax investigation. The assets and liabilities of the
Year Net Amount of Exhibit taxpayer during the above-mentioned years were ascertained and it was discovered
Incom Tax Paid that from 1946 to 1951, his net worth had increased every year, which increases in
e net worth was very much more than the income reported during said years. The
findings clearly indicated that the taxpayer did not declare correctly the income
194 P12,8 P114.66 pp. reported in his income tax returns for the aforesaid years.
5 22.00 85-88
B.I.R.
Based on the above findings of Examiner Guerrero, respondent Commissioner, in his
rec.
letter dated November 28, 1952, notified the taxpayer (Matias H. Aznar) of the
assessed tax delinquency to the amount of P723,032.66, plus compromise penalty.
194 9,910. 114.66 38-A The taxpayer requested a reinvestigation which was granted for the purpose of
6 94 (pp. verifying the merits of the various objections of the taxpayer to the deficiency
329- income tax assessment of November 28, 1952.
332
B.I.R
rec.) After the reinvestigation, another deficiency assessment to the reduced amount of
P381,096.07 dated February 16, 1955, superseded the previous assessment and
194 10,20 132.00 39 notice thereof was received by Matias H. Aznar on March 2, 1955.
7 0.00 (pp.
75-78 The new deficiency assessment was based on the following computations:
B.I.R
rec.)
1946
194 9,148. 68.90 40
8 34 (pp. Net income per return ........................ P9,910.94
70-73 Add: Under declared income .............. 22,559.94
B.I.R. Net income per investigation............... 32,470.45
rec.)
Deduct: Income tax liability
194 8,990. 59.72 41 per return as assessed ...................................................... 114.66
9 66 (pp. Balance of tax due ........................................................... P3,687.10
64-67 Add: 50% surcharge ........................................................ 1,843.55
B.I.R. DEFICIENCY INCOME TAX ...................................... P5,530.65
rec.)

195 8,364. 28.22 42 1947


0 50 (pp.
59-62, Net income per return ..................................................... P10,200.00
BIR Add: Under declared income ............................................ 90,413.56
rec.) Net income per reinvestigation ....................................... P100,613.56

12
Deduct: Personal and additional exemption ...................... 7,000.00 1951
Amount of income subject to tax ...................................... P93,613.56
Total tax liability ............................................................... P24,753.15
Net income per return ........................................................ P6,800.00
Deduct: Income tax liability per return as assessed ............ 132.00
Add: Under declared income ............................................... 33,355.80
Balance of tax due ........................................................... P24,621.15
Net income per reinvestigation ............................................ P40,155.80
Add: 50%
Deduct: Personal and additional exemptions ........................ 7,200.00
surcharge ........................................................ 12,310.58 DEFICIENCY INCOME
Amount of income subject to tax ......................................... P32,955.80
TAX ...................................... P36,931.73
Total tax liability .................................................................. P7,684.00
Deduct: Income tax liability per return as assessed ............... - o - .
1948 Balance of tax due .............................................................. P7,684.00
Add: 50%
surcharge ........................................................... 3,842.00 DEFICIENCY INCOME
Net income per return ...................................................... P9,148.34
TAX .......................................... P11,526.00
Add: Under declared income ............................................. 15,624.63
Net income per reinvestigation .......................................... P24,772.97
Deduct: Personal and additional exemptions ...................... 7,000.00 SUMMARY
Amount of income subject to tax ....................................... P17,772.97
Total tax liability ............................................................... 2,201.40
Deduct: Income tax liability per return as assessed ............ 68.90 1946
.... P5,530.65
Balance of tax due ........................................................... P2,132.500
Add: 50% surcharge ........................................................ 1,066.25 DEFICIENCY 1947 .... 36,931.73
INCOME TAX ...................................... P3,198.75
1948 .... 3,198.75
1949
1949 .... 45,125.94

Net income per return ....................................................... P9,990.66 1950 .... 278,783.00


Add: Under declared income ............................................. 105,418.53
Net income per reinvestigation .......................................... 114,409.19
1951 .... 11,526.00
Deduct: Personal and additional exemptions ...................... P7,000.00
Amount of income subject to tax ....................................... P107,409.19
Total .... P381,096.07
Total tax liability ............................................................... P30,143.68
Deduct: Income tax liability per return as assessed ............. 59.72
Balance of tax due ............................................................ P30,083.96
In determining the unreported income, the respondent Commissioner of Internal
Add: 50%
Revenue resorted to the networth method which is based on the following
surcharge ......................................................... 15,041.98 DEFICIENCY INCOME
computations:
TAX ....................................... P45,125.94

1945
1950

Real estate inventory ................................ P64,738.00


Net income per return ....................................................... P8,364.50
Other assets ............................................. 37,606.87
Add: Under declared income ............................................. 365,578.76
Total assets ............................................ P102,344.87
Net income per reinvestigation .......................................... P373,943.26
Less: Depreciation allowed ...................... 2,027.00
Deduct: Personal and additional exemptions ...................... 7,800.00
Networth as of Dec. 31, 1945 ................ P100,316.97
Amount of income subject to tax ....................................... P366,143.26
Total tax liability ............................................................... P185,883.00
Deduct: Income tax liability per return as assessed ............. 28.00 1946
Balance of tax due ............................................................ P185,855.00
Add: 50% surcharge ......................................................... 92,928.00 Real estate inventory ................................. P86,944.18
DEFICIENCY INCOME TAX ....................................... P278,783.00 Other assets ............................................. 60,801.65
Total assets ............................................. P147,745.83

13
Less: Depreciation allowed ...................... 4,875.41 Add: Estimated living expenses .................. 7,000.00
Net assets ................................................ P142,870.42 Net income ............................................... P114,409.19
Less: Liabilities .................. P17,000.00
Net Worth as of
1950
Jan. 1, 1946 ................... P100,316.97 P117,316.97
Increase in networth ................................. 25,553.45
Add: Estimated living expenses ................. 6,917.00 Real estate inventory .................................. P412,465.52
Net income .............................................. P32,470.45 Investment in Schools and
other colleges ................................ 193,460.99
October assets .......................................... 310,788.87
1947
Total assets ............................................... P916,715.38
Less; Depreciation allowed ........................ 47,561.99
Real estate inventory .................................. P237,824.18 Net assets ................................................. P869,153.39
Other assets ............................................... 54,495.52 Less: Liabilities .................. P158,343.99
Total assets ............................................... P292,319.70 Networth as of Jan. 1, 1950 ... 344,666.14 P503,010.13
Less: Depreciation allowed ......................... 12,835.72 Increase in networth ................................... P366,143.26
Net assets .................................................. 279,483.98 Add: Estimated living expenses ................... 7,800.00
Less: Liabilities ................... P60,000.00 Net income ................................................. P373,943.26
Networth as of
Jan. 1, 1947 ........................ 125,870.42 P185,870.42
1951
Increase in networth ................................... P93,613.56
Add: Estimated living expenses ................... 7,000.00
Net income ................................................P100,613.56 Real estate inventory ................................... P412,465.52
Investment in schools and other colleges ..... 214,016.21
Other assets ............................................... 320,209.40
1948
Total assets ................................................ P946,691.13
Less: Depreciation allowed ......................... 62,466.90
Real estate inventory .................................. P244,824.18 Net assets .................................................. P884,224.23
Other assets .............................................. 118,720.60 Less: Liabilities ........................................... P140,459.03
Total assets ............................................... P363,544.78 Networth as of
Less: Depreciation allowed ........................ 20,936.03 Jan. 1, 1951 ................ 710,809.40 P851,268.43
Net assets ................................................. P342,608.75 Increase in networth .................................... P32,955.80
Less: Liabilities ................... P105,351.80 Add: Estimated living expenses .................... 7,200.00
Networth as of Net income ................................................. P40,155.80
Jan. 1, 1948 ...................... 219,483.98 P324,835.78
Increase in networth ................................... P17,772.97
(Exh. 45-B, BIR rec. p. 188)
Add: Estimated living expenses ................... 7,000.00
Net income ................................................ P24,772.97
On February 20, 1953, respondent Commissioner of Internal Revenue, thru the City
Treasurer of Cebu, placed the properties of Matias H. Aznar under distraint and levy
1949
to secure payment of the deficiency income tax in question. Matias H. Aznar filed his
petition for review of the case with the Court of Tax Appeals on April 1, 1955, with a
Real estate inventory ................................. P400,515.52 subsequent petition immediately thereafter to restrain respondent from collecting
Investment in schools and other colleges .... 23,105.29 the deficiency tax by summary method, the latter petition being granted on
Other assets ............................................. 70,311.00 February 8, 1956, per C.T.A. resolution, without requiring petitioner to file a bond.
Total assets ............................................... P493,931.81 Upon review, this Court set aside the C.T.A. resolution and required the petitioner to
Less: Depreciation allowed ........................ 32,657.08 deposit with the Court of Tax Appeals the amount demanded by the Commissioner
Net assets ................................................. P461.274.73 of Internal Revenue for the years 1949 to 1951 or furnish a surety bond for not
Less; Liabilities .................. P116,608.59 more than double the amount.
Networth as of
Jan. 1, 1949 ...................... 237,256.95 P353,865.54
On March 5, 1962, in a decision signed by the presiding judge and the two associate
Increase in networth .................................. P107,409.19
judges of the Court of Tax Appeals, the lower court concluded that the tax liability of

14
the late Matias H. Aznar for the year 1946 to 1951, inclusive should be P227,788.64 Income subject to tax ................................................. P45,709.19
minus P96.87 representing the tax credit for 1945, or P227,691.77, computed as Tax due thereon ......................................................... P8,978.57
follows: Less: Tax already assessed ......................................... 59.72
Balance of tax due ....................................................... P8,918.85
Add: 50% surcharge .................................................... 4,459.42
1946
Deficiency income tax ................................................. P13,378.27

Net income per return .............................................. P9,910.94


1950
Add: Under declared income ..................................... 22,559.51
Net income ............................................................ P32,470.45
Less: Personal and additional exemptions .................. 6,917.00 Net income per return .................................................. P6,800.00
Income subject to tax ............................................. P25,553.45 Add: Under declared income ......................................... 33,355.80
Tax due thereon ...................................................... P3,801.76 Net income ................................................................. P40,155.80
Less: Tax already assessed ...................................... 114.66 Less: Personal and additional exemptions ...................... 7,200.00
Balance of tax due .................................................... P3,687.10 Income subject to tax .................................................. P32,955.80
Add: 50% surcharge ................................................. 1,843.55 Tax due thereon ........................................................... P7,684.00
Deficiency income tax ................................................ P5,530.65 Less: Tax already assessed ........................................... -o- .
Balance of tax due ........................................................ P7,684.00
Add: 50% surcharge .................................................... 3,842.00
1947
Deficiency income tax .................................................. P11,526.00

Net income per return ............................................ P10,200.00


1951
Add: Under declared income .................................. 57,551.19
Net income ........................................................... P67,751.19
Less: Personal and additional exemptions ............... 7,000.00 Net income per return ................................................... P8,364.50
Income subject to tax ............................................. P60,751.19 Add: Under declared income ........................................ 246,449.06
Tax due thereon ..................................................... P13,420.38 Net income ............................................................... P254.813.56
Less: Tax already assessed ..................................... P132.00 Less: Personal and additional exemptions .................... 7,800.00
Balance of tax due ................................................... P13,288.38 Income subject to tax ................................................ P247,013.56
Add: 50% surcharge ................................................ 6,644.19 Tax due thereon ........................................................ P117,348.00
Deficiency income tax .............................................. P19,932.57 Less: Tax already assessed ........................................ 28.00
Balance of tax due ..................................................... P117,320.00
Add: 50% surcharge .................................................. 58,660.00
1948
Deficiency income tax ................................................ P175 980.00

Net income per return .............................................. P9,148.34


SUMMARY
Add: Under declared income ..................................... 8,732.10
Net income ............................................................ P17,880.44
Less: Personal and additional exemptions ................. 7,000.00 1946 P5,530.65
Income subject to tax .............................................. P10,880.44
Tax due thereon ...................................................... P1,029.67
1947 19,932.57
Less: Tax already assessed ....................................... 68.90
Balance of tax due .................................................... 960.77
Add: 50% surcharge ................................................. 480.38 1948 1,441.15
Deficiency income tax ............................................... P1,441.15
1949 13,378.27
1949
1950 175,980.00
Net income per return ................................................. P8,990.66
Add: under declared income ......................................... 43,718.53 1951 11,526.00
Net income ............................................................... P52,709.19
Less: Personal and additional exemptions .................... 7,000.00
15
P227,788.64. The ordinary period of prescription of 5 years within which to assess tax liabilities
under Sec. 331 of the NIRC should be applicable to normal circumstances, but
whenever the government is placed at a disadvantage so as to prevent its lawful
I
agents from proper assessment of tax liabilities due to false returns, fraudulent
return intended to evade payment of tax or failure to file returns, the period of ten
The first vital issue to be decided here is whether or not the right of the years provided for in Sec. 332 (a) NIRC, from the time of the discovery of the
Commissioner of Internal Revenue to assess deficiency income taxes of the late falsity, fraud or omission even seems to be inadequate and should be the one
Matias H. Aznar for the years 1946, 1947, and 1948 had already prescribed at the enforced.
time the assessment was made on November 28, 1952.
There being undoubtedly false tax returns in this case, We affirm the conclusion of
Petitioner's contention is that the provision of law applicable to this case is the the respondent Court of Tax Appeals that Sec. 332 (a) of the NIRC should apply and
period of five years limitation upon assessment and collection from the filing of the that the period of ten years within which to assess petitioner's tax liability had not
returns provided for in See. 331 of the National Internal Revenue Code. He argues expired at the time said assessment was made.
that since the 1946 income tax return could be presumed filed before March 1, 1947
and the notice of final and last assessment was received by the taxpayer on March
II
2, 1955, a period of about 8 years had elapsed and the five year period provided by
law (Sec. 331 of the National Internal Revenue Code) had already expired. The
same argument is advanced on the taxpayer's return for 1947, which was filed on As to the alleged errors committed by the Court of Tax Appeals in not deducting
March 1, 1948, and the return for 1948, which was filed on February 28, 1949. from the alleged undeclared income of the taxpayer for 1946 the proceeds from the
Respondents, on the other hand, are of the firm belief that regarding the sale of jewelries valued at P30,000; in not excluding from other schedules of assets
prescriptive period for assessment of tax returns, Section 332 of the National of the taxpayer (a) accounts receivable from customers in the amount of P38,000
Internal Revenue Code should apply because, as in this case, "(a) In the case of a for 1948, P126,816.50 for 1950, and provisions for doubtful accounts in the amount
false or fraudulent return with intent to evade tax or of a failure to file a return, the of P41,810.56 for 1950; (b) over valuation of hospital and dental buildings for 1949
tax may be assessed, or a proceeding in court for the collection of such tax may be in the amount of P32,000 and P6,191.32 respectively; (c) investment in hollow
begun without assessment, at any time within ten years after the discovery of the block business in the amount of P8,603.22 for 1949; (d) over valuation of surplus
falsity, fraud or omission" (Sec. 332 (a) of the NIRC). goods in the amount of P23,000 for the year 1949; (e) various lands and buildings
included in the schedule of assets for the years 1950 and 1951 in the total amount
of P243,717.42 for 1950 and P62,564.00 for 1951, these issues would depend for
Petitioner argues that Sec. 332 of the NIRC does not apply because the taxpayer did
their resolution on determination of questions of facts based on an evaluation of
not file false and fraudulent returns with intent to evade tax, while respondent
evidence, and the general rule is that the findings of fact of the Court of Tax
Commissioner of Internal Revenue insists contrariwise, with respondent Court of
Appeals supported by substantial evidence should not be disturbed upon review of
Tax Appeals concluding that the very "substantial under declarations of income for
its decision (Section 2, Rule 44, Rules of Court).
six consecutive years eloquently demonstrate the falsity or fraudulence of the
income tax returns with an intent to evade the payment of tax."
On the question of the alleged sale of P30,000 worth of jewelries in 1946, which
amount petitioner contends should be deducted from the taxpayer's net worth as of
To our minds we can dispense with these controversial arguments on facts,
December 31, 1946, the record shows that Matias H. Aznar, when interviewed by
although we do not deny that the findings of facts by the Court of Tax Appeals,
B.I.R. Examiner Guerrero, stated that at the beginning of 1945 he had P60,000
supported as they are by very substantial evidence, carry great weight, by resorting
worth of jewelries inherited from his ancestors and were disposed off as follows:
to a proper interpretation of Section 332 of the NIRC. We believe that the proper
1945, P10,000; 1946, P20,000; 1947, P10,000; 1948, P10,000; 1949, P7,000;
and reasonable interpretation of said provision should be that in the three different
(Report of B.I.R. Examiner Guerrero, B.I.R. rec. pp. 90-94).
cases of (1) false return, (2) fraudulent return with intent to evade tax, (3) failure
to file a return, the tax may be assessed, or a proceeding in court for the collection
of such tax may be begun without assessment, at any time within ten years after During the hearing of this case in the Court of Tax Appeals, petitioner's accountant
the discovery of the (1) falsity, (2) fraud, (3) omission. Our stand that the law testified that on January 1, 1945, Matias H. Aznar had jewelries worth P60,000
should be interpreted to mean a separation of the three different situations of false which were acquired by purchase during the Japanese occupation (World War II)
return, fraudulent return with intent to evade tax, and failure to file a return is and sold on various occasions, as follows: 1945, P5,000 and 1946, P30,000. To
strengthened immeasurably by the last portion of the provision which segregates corroborate the testimony of the accountant, Mrs. Ramona Agustines testified that
the situations into three different classes, namely "falsity", "fraud" and "omission". she bought from the wife of Matias H. Aznar in 1946 a diamond ring and a pair of
That there is a difference between "false return" and "fraudulent return" cannot be earrings for P30,000; and in 1947 a wrist watch with diamonds, together with
denied. While the first merely implies deviation from the truth, whether intentional antique jewelries, for P15,000. Matias H. Aznar, on the other hand testified that in
or not, the second implies intentional or deceitful entry with intent to evade the 1945, his wife sold to Sards Parino jewelries for P5,000 and question, Mr. Aznar
taxes due.

16
stated that his transaction with Sards Parino, with respect to the sale of jewelries, reconstructed in 1947, the lower court correctly concluded that the reconstruction of
amounted to P15,000. the property did not render it valueless during the time it was being reconstructed
and consequently it should be listed as an asset as of January 1, 1946, with the
same valuation as in 1945, that is P1,500.
The lower court did not err in finding material inconsistencies in the testimonies of
Matias H. Aznar and his witnesses with respect to the values of the jewelries
allegedly disposed off as stated by the witnesses. Thus, Mr. Aznar stated to the On the question of accounts receivable from customers in the amount of P38,000
B.I.R. examiner that jewelries worth P10,000 were sold in 1945, while his own for 1948, and P123,816.58 for the years 1950 and 1951, which were included in the
accountant testified that the same jewelries were sold for only P5,000. Mr. Aznar assets of Mr. Aznar for those years by the respondent Commissioner of Internal
also testified that Mrs. Agustines purchased from his wife jewelries for P35,000, and Revenue, it is very clear that those figures were taken from the statements (Exhs.
yet Mrs. Agustines herself testified that she bought jewelries for P30,000 and 31 and 32) filed by Mr. Matias H. Aznar with the Philippine National Bank when he
P15,000 on two occasions, or a total of P45,000. was intending to obtain a loan. These statements were under oath and the natural
implication is that the information therein reflected must be the true and accurate
financial condition of the one who executed those statements. To believe the
We do not see any plausible reason to challenge the fundamentally sound basis
petitioner's argument that the late Mr. Aznar included those figures in his sworn
advanced by the Court of Tax Appeals in considering the inconsistencies of the
statement only for the purpose of obtaining a bigger credit from the bank is to cast
witnesses' testimony as material, in the following words:
suspicion on the character of a man who can no longer defend himself. It would be
as if pointing the finger of accusation on the late Mr. Aznar that he intentionally
We do not say that witnesses testifying on the same transaction falsified his sworn statements (Exhs. 31 and 32) to make it appear that there were
should give identical testimonies. Because of the frailties and the non-existent accounts receivable just to increase his assets by fictitious entries so
limitations of the human mind, witnesses' statements are apt to that his credit with the Philippine National Bank could be enhanced. Besides, We do
be inconsistent in certain points, but usually the inconsistencies not lose sight of the fact that those statements (Exhs. 31 and 32) were executed
refer to the minor phases of the transaction. It is the before this tax controversy arose and the disputable presumptions that a person is
insignificance of the detail of an occurrence that fails to impress innocent of crime or wrong; that a person intends the ordinary consequences of his
the human mind. When that same mind, made to recall what voluntary act; that a person takes ordinary care of his concerns; that private
actually happened, the significant point which it failed to take note transaction have been fair and regular; that the ordinary course of business has
is naturally left out. But it is otherwise as regards significant been followed; that things have happened according to the ordinary course of
matters, for they leave indelible imprints upon the human mind. nature and the ordinary habits of life; that the law has been obeyed (Sec. 5, (a),
Hence, testimonial inconsistencies on the minor details of an (c), (d), (p), (q), (z), (ff), Rule 131 of the Rules of Court), together with the
occurrence are dismissed lightly by the courts, while discrepancies conclusive presumption that "whenever a party has, by his own declaration, act, or
on significant points are taken seriously and weigh adversely to omission, intentionally and deliberately led another to believe a particular thing
the party affected thereby. true, and to act upon such belief, he cannot, in any litigation arising out of such
declaration, act or omission, be permitted to falsify it" (Sec. 3 (a), Rule 131, Rules
There is no sound basis for deviating from the lower court's conclusion that: of Court), convincingly indicate that the accounts receivable stated by Mr. Aznar in
"Taxwise in view of the aforesaid inconsistencies, which we deem material and Exhibits 31 and 32 were true, in existence, and accurate to the very amounts
significant, we dismiss as without factual basis petitioner's allegation that jewelries mentioned.
form part of his inventory of assets for the purpose of establishing his net worth at
the beginning of 1946." There is no merit to petitioners argument that those statements were only for the
purpose of obtaining a bigger credit from the bank (impliedly stating that those
As to the accounts receivable from the United States government for the amount of statements were false) and those accounts were allegedly back accounts of students
P38,254.90, representing a claim for goods commandered by the U.S. Army during of the Southwestern Colleges and were worthless, and if collected, would go to the
World War II, and which amount petitioner claimed should be included in his net funds of the school. The statement of the late Mr. Aznar that they were accounts
worth as of January 1, 1946, the Court of Tax Appeals correctly concluded that the receivable from customers should prevail over the mere allegation of petitioner,
uncontradicted evidence showed that "the collectible accounts of Mr. Aznar from the unsupported as they are by convincing evidence. There is no reason to disturb the
U.S. Government in the sum of P38,254.90 should be added to his assets (under lower court's conclusion that the amounts of P38,000 and P123,816.58 were
accounts receivable) as of January 1, 1946. As of December 31, 1947, and accounts receivable from customers and as such must be included as petitioner's
December 31, 1948, the years within which the accounts were paid to him, the assets for the years indicated.
'accounts receivable shall decrease by P31,362.37 and P6,892.53, respectively."
As to the questions of doubtful accounts (bad debts), for the amount of P41,810.56,
Regarding a house in Talisay Cebu, (covered by Tax Declaration No. 8165) which it is clear that said amount is taken from Exhibit 31, the sworn statement of
was listed as an asset during the years 1945 and 1947 to 1951, but which was not financial condition filed by Mr. Matias H. Aznar with the Philippine National Bank.
listed as an asset in 1946 because of a notation in the tax declaration that it was The lower court did not commit any error in again giving much weight to the
17
statement of Mr. Aznar and in concluding that inasmuch as this is an item separate testimony, a copy of a letter of the City Assessor of Cebu City) to prove that the two
and apart from the taxpayer's accounts receivable and non-deductible expense, it buildings were really destroyed by typhoon in 1949 and, therefore, should be
should be reverted to the accounts receivable and, consequently, considered as an eliminated from the petitioner's inventory of assets beginning December 31, 1949.
asset in 1950.
On the issue of investment in the hollow blocks business, We see no compelling
On the alleged over valuation of two buildings (hospital building which respondent reason to alter the lower court's conclusion that "whatever was spent in the hollow
Commissioner of Internal Revenue listed as an asset from 1949-1951 at the basic blocks business is an investment, and being an investment, the same should be
valuation of P130,000, and which petitioner claims to be over valued by P32,000; treated as an asset. With respect to the amount representing the value of the
dentistry building valued by respondent Commissioner of Internal Revenue at building, there is no duplication in the listing as the inventory of real property does
P36,191.34, which petitioner claims to be over valued by P6,191.34), We find no not include the building in question."
sufficient reason to alter the conclusion of respondent Court of Tax Appeals
sustaining the respondent Commissioner of Internal Revenue's valuation of both
Respondent Commissioner of Internal Revenue included in the inventory, under the
properties.
heading of other asset, the amount of P8,663.22, treated as investment in the
hollow block business. Petitioner objects to the inclusion of P1,683.42 which was
Respondent Commissioner of Internal Revenue based his valuation of the hospital spent on the building and in the business and of P674.35 which was spent for labor,
building on the representation of Mr. Matias H. Aznar himself who, in his letter (Exh. fuel, raw materials, office supplies etc., contending that the former amount is a
35) to the Philippine National Bank dated September 5, 1949, stated that the duplication of inventory (included among the list of properties) and the latter is a
hospital building cost him P132,000. However in view of the effect of a typhoon in business expense which should be eliminated from the list of assets.
1949 upon the building, the value allowed was P130,000. Exhibit 35, contrary to
petitioner's contention, should be given probative value because, although it is an
The inclusion of expenses (labor and raw materials) as part of the hollow block
unsigned plain copy, that exhibit was taken by the investigating examiner of the
business is sanctioned in the inventory method of tax verification. It is a sound
B.I.R. from the files of the Southwestern Colleges and formed part of his report of
accounting practice to include raw materials that will be used for future
investigation as a public official. The estimates of an architect and a civil engineer
manufacture. Inclusion of direct labor is also proper, as all these items are to be
who agreed that a value of P84,240 is fair for the hospital building, made years
embodied in a summary of assets (investment by the taxpayer credited to his
after the building was constructed, cannot prevail over the petitioner's own estimate
capital account as reflected in Exhibit 72-A, which is a working sheet with entries
of his property's value.
taken from the journal of the petitioner concerning his hollow blocks business).
There is no evidence to show that there was duplication in the inclusion of the
Respondent Commissioner of Internal Revenue's valuation of P36,191.34 of the building used for hollow blocks business as part of petitioner's investment as this
Dentistry Building is based on the letter of Mr. and Mrs. Matias H. Aznar to the building was not included in the listing of real properties of petitioner (Exh. 45-C p.
Southwestern Colleges, dated December 15, 1950, which is embodied in the 187 B.I.R. rec.).
minutes of the meeting of the Board of Trustees of the Southwestern Colleges held
on May 7, 1951 (Exhibit G-1). In Exhibit 26 A, which is the cash book of the
As to the question of the real value of the surplus goods purchased by Mr. Matias H.
Southwestern Colleges, this building was listed as of the same amount. Petitioner's
Aznar from the U.S. Army, the best evidence, as observed correctly by the lower
estimate of P30,000 for this building, based on Architect Paca's opinion, cannot
court, is the statement of Mr. Matias H. Aznar, himself, as appearing Exh. 35 (copy
stand against the owner's estimate and that which appears in the cash book of the
of a letter dated September 5, 1949 to the Philippine National Bank), to the effect
Southwestern Colleges, if we take into consideration that the owner's (Mr. Matias H.
"as part of my assets I have different merchandise from Warehouse 35, Tacloban,
Aznar) letter was written long before this tax proceeding was initiated, while
Leyte at a total cost of P43,000.00 and valued at no less than P20,000 at present
architect Paca's estimate was made upon petitioner's request solely for the purpose
market value." Petitioner's claim that the goods should be valued at only P20,000 in
of evidence in this tax case.
accordance with an alleged invoice is not supported by evidence since the invoice
was not presented as exhibit. The lower court's act in giving more credence to the
In the inventory of assets of petitioner, respondent Commissioner of Internal statement of Mr. Aznar cannot be questioned in the light of clear indications that it
Revenue included the administrative building valued at P19,200 for the years 1947 was never controverted and it was given at a time long before the tax controversy
and 1948, and P16,700 for the years 1949 to 1951; and a high school building arose.
valued at P48,000 for 1947 and 1948, and P45,000 for 1949, 1950 and 1951. The
reduced valuation for the latter years are due to allowance for partial loss resulting
The last issue on propriety of inclusion in petitioner's assets made by respondent
from the 1949 typhoon. Petitioner did not question the inclusion of these buildings
Commissioner of Internal Revenue concerns several buildings which were included
in the inventory for the years prior to 1950, but objected to their inclusion as assets
in the list of petitioner's assets as of December 31, 1950. Petitioner contends that
as of January 1, 1950, because both buildings were destroyed by a typhoon in
those buildings were conveyed and ceded to Southwestern Colleges on December
November of 1949. There is sufficient evidence (Exh. G-1, affidavit of Jesus S.
15, 1950, in consideration of P100,723.99 to be paid in cash. The value of the
Intan, employee in the office of City Assessor of Cebu City, Exh. 18, Mr. Intan's
different buildings are listed as: hospital building, P130,000; gymnasium, P43,000;

18
dentistry building, P36,191.34; bodega 1, P781.18; bodega 2, P7,250; college of returns by mistake filed by Mr. Matias H. Aznar as those returns were prepared by
law, P10,950; laboratory building, P8,164; home economics, P5,621; morgue, his accountant employees, but there were no proven fraudulent returns with intent
P2,400; science building, P23,600; faculty house, P5,760. It is suggested that the to evade taxes that would justify the imposition of the 50% surcharge authorized by
value of the buildings be eliminated from the real estate inventory and the sum of law as fraud penalty.
P100,723.99 be included as asset as of December 31, 1950.
The lower court based its conclusion that the 50% fraud penalty must be imposed
The lower court could not find any evidence of said alleged transfer of ownership on the following reasoning: .
from the taxpayer to the Southwestern Colleges as of December 15, 1950, an
allegation which if true could easily be proven. What is evident is that those
It appears that Matias H. Aznar declared net income of P9,910.94,
buildings were used by the Southwestern Colleges. It is true that Exhibit G-1 shows
P10,200, P9,148.34, P8,990.66, P8,364.50 and P6,800 for the
that Mr. and Mrs. Matias H. Aznar offered those properties in exchange for shares of
years 1946, 1947, 1948, 1949, 1950 and 1951, respectively.
stocks of the Southwestern Colleges, and Exhibit "G" which is the minutes of the
Using the net worth method of determining the net income of a
meeting of the Board of Trustees of the Southwestern Colleges held on August 6,
taxpayer, we find that he had net incomes of P32,470.45,
1951, shows that Mr. Aznar was amenable to the value fixed by the board of
P67,751.19, P17,880.44, P52,709.11, P254,813.56 and
trustees and that he requested to be paid in cash instead of shares of stock. But
P40,155.80 during the respective years 1946, 1947, 1948, 1949,
those are not sufficient evidence to prove that transfer of ownership actually
1950, and 1951. In consequence, he underdeclared his income by
happened on December 15, 1950. Hence, the lower court did not commit any error
227% for 1946, 564% for 1947, 95%, for 1948, 486% for 1949,
in sustaining the respondent Commissioner of Internal Revenue's act of including
2,946% for 1950 and 490% for 1951. These substantial under
those buildings as part of the assets of petitioner as of December 31, 1950.
declarations of income for six consecutive years eloquently
demonstrate the falsity or fraudulence of the income tax return
Petitioner also contends that properties allegedly ceded to the Southwestern with an intent to evade the payment of tax. Hence, the imposition
Colleges in 1951 for P150,000 worth of shares of stocks, consisting of: land, of the fraud penalty is proper (Perez vs. Court of Tax Appeals,
P22,684; house, P13,700; group of houses, P8,000; building, P12,000; nurses G.R. No. L-10507, May 30, 1958). (Emphasis supplied)
home, P4,100; nurses home, P2,080, should be excluded from the inventory of
assets as of December 31, 1951. The evidence (Exh. H), however, clearly shows
As could be readily seen from the above rationalization of the lower court, no
that said properties were formally conveyed to the Southwestern Colleges only on
distinction has been made between false returns (due to mistake, carelessness or
September 25, 1952. Undoubtedly, petitioner was the owner of those properties
ignorance) and fraudulent returns (with intent to evade taxes). The lower court
prior to September 25, 1952 and said properties should form part of his assets as of
based its conclusion on the petitioner's alleged fraudulent intent to evade taxes on
December 31, 1951.
the substantial difference between the amounts of net income on the face of the
returns as filed by him in the years 1946 to 1951 and the net income as determined
The uncontested portions of the lower court's decision consisting of its conclusions by the inventory method utilized by both respondents for the same years. The lower
that library books valued at P7,041.03, appearing in a journal of the Southwestern court based its conclusion on a presumption that fraud can be deduced from the
Colleges marked as' Exhibit 25-A, being an investment, should be treated as an very substantial disparity of incomes as reported and determined by the inventory
asset beginning December 31, 1950; that the expenses for construction to the method and on the similarity of consecutive disparities for six years. Such a basis
amount of P113,353.70, which were spent for the improvement of the buildings for determining the existence of fraud (intent to evade payment of tax) suffers from
appearing in Exhibit 24 are deemed absorbed in the increased value of the buildings an inherent flaw when applied to this case. It is very apparent here that the
as appraised by respondent Commissioner of Internal Revenue at cost after respondent Commissioner of Internal Revenue, when the inventory method was
improvements were made, and should be taken out as additional assets; that the resorted to in the first assessment, concluded that the correct tax liability of Mr.
amount receivable of P5,776 from a certain Benito Chan should be treated as Aznar amounted to P723,032.66 (Exh. 1, B.I.R. rec. pp. 126-129). After a
petitioner's asset but the amount of P5,776 representing the value of a house and reinvestigation the same respondent, in another assessment dated February 16,
lot given as collateral to secure said loan should not be considered as an asset of 1955, concluded that the tax liability should be reduced to P381,096.07. This is a
petitioner since to do so would result in a glaring duplication of items, are all crystal-clear, indication that even the respondent Commissioner of Internal Revenue
affirmed. There seems to be no controversy as to the rest of the items listed in the with the use of the inventory method can commit a glaring mistake in the
inventory of assets. assessment of petitioner's tax liability. When the respondent Court of Tax Appeals
reviewed this case on appeal, it concluded that petitioner's tax liability should be
only P227,788.64. The lower court in three instances (elimination of two buildings in
III
the list of petitioner's assets beginning December 31, 1949, because they were
destroyed by fire; elimination of expenses for construction in petitioner's assets as
The second issue which appears to be of vital importance in this case centers on the duplication of increased value in buildings, and elimination of value of house and lot
lower court's imposition of the fraud penalty (surcharge of 50% authorized in in petitioner's assets because said property was only given as collateral) supported
Section 72 of the Tax Code). The petitioner insists that there might have been false petitioner's stand on the wrong inclusions in his lists of assets made by the
19
respondent Commissioner of Internal Revenue, resulting in the very substantial The total sum of P151,859.10 should be decreased by P96.87 representing the tax
reduction of petitioner's tax liability by the lower court. The foregoing shows that it credit for 1945, thereby leaving a balance of P151,762.23.
was not only Mr. Matias H. Aznar who committed mistakes in his report of his
income but also the respondent Commissioner of Internal Revenue who committed
WHEREFORE, the decision of the Court of Tax Appeals is modified in so far as the
mistakes in his use of the inventory method to determine the petitioner's tax
imposition of the 50% fraud penalty is concerned, and affirmed in all other respects.
liability. The mistakes committed by the Commissioner of Internal Revenue which
The petitioner is ordered to pay to the Commissioner of Internal Revenue, or his
also involve very substantial amounts were also repeated yearly, and yet we cannot
duly authorized representative, the sum of P151,762.23, representing deficiency
presume therefrom the existence of any taint of official fraud.
income taxes for the years 1946 to 1951, inclusive, within 30 days from the date
this decision becomes final. If the said amount is not paid within said period, there
From the above exposition of facts, we cannot but emphatically reiterate the well shall be added to the unpaid amount the surcharge of 5%, plus interest at the rate
established doctrine that fraud cannot be presumed but must be proven. As a of 12% per annum from the date of delinquency to the date of payment, in
corollary thereto, we can also state that fraudulent intent could not be deduced accordance with Section 51 of the National Internal Revenue Code.
from mistakes however frequent they may be, especially if such mistakes emanate
from erroneous entries or erroneous classification of items in accounting methods
With costs against the petitioner.
utilized for determination of tax liabilities The predecessor of the petitioner
undoubtedly filed his income tax returns for "the years 1946 to 1951 and those tax
returns were prepared for him by his accountant and employees. It also appears G.R. No. 78953             July 31, 1991
that petitioner in his lifetime and during the investigation of his tax liabilities COMMISSIONER OF INTERNAL REVENUE, petitioner,
cooperated readily with the B.I.R. and there is no indication in the record of any act vs. MELCHOR J. JAVIER, JR. and THE COURT OF TAX APPEALS, respondents.
of bad faith committed by him.
Central in this controversy is the issue as to whether or not a taxpayer who merely
The lower court's conclusion regarding the existence of fraudulent intent to evade states as a footnote in his income tax return that a sum of money that he
payment of taxes was based merely on a presumption and not on evidence erroneously received and already spent is the subject of a pending litigation and
establishing a willful filing of false and fraudulent returns so as to warrant the there did not declare it as income is liable to pay the 50% penalty for filing a
imposition of the fraud penalty. The fraud contemplated by law is actual and not fraudulent return.
constructive. It must be intentional fraud, consisting of deception willfully and
deliberately done or resorted to in order to induce another to give up some legal This question is the subject of the petition for review before the Court of the portion
right. Negligence, whether slight or gross, is not equivalent to the fraud with intent of the Decision1 dated July 27, 1983 of the Court of Tax Appeals (CTA) in C.T.A.
to evade the tax contemplated by the law. It must amount to intentional wrong- Case No. 3393, entitled, "Melchor J. Javier, Jr. vs. Ruben B. Ancheta, in his capacity
doing with the sole object of avoiding the tax. It necessarily follows that a mere as Commissioner of Internal Revenue," which orders the deletion of the 50%
mistake cannot be considered as fraudulent intent, and if both petitioner and surcharge from Javier's deficiency income tax assessment on his income for 1977.
respondent Commissioner of Internal Revenue committed mistakes in making
entries in the returns and in the assessment, respectively, under the inventory
method of determining tax liability, it would be unfair to treat the mistakes of the The respondent CTA in a Resolution2 dated May 25, 1987, denied the
petitioner as tainted with fraud and those of the respondent as made in good faith. Commissioner's Motion for Reconsideration3 and Motion for New Trial4 on the
deletion of the 50% surcharge assessment or imposition.

We conclude that the 50% surcharge as fraud penalty authorized under Section 72
of the Tax Code should not be imposed, but eliminated from the income tax The pertinent facts as are accurately stated in the petition of private respondent
deficiency for each year from 1946 to 1951, inclusive. The tax liability of the Javier in the CTA and incorporated in the assailed decision now under review, read
petitioner for each year should, therefore, be: as follows:

1946 P 3,687.10 2. That on or about June 3, 1977, Victoria L. Javier, the wife of the petitioner
1947 13,288.38 (private respondent herein), received from the Prudential Bank and Trust Company
1948 960.77 in Pasay City the amount of US$999,973.70 remitted by her sister, Mrs. Dolores
1949 8,918.85 Ventosa, through some banks in the United States, among which is Mellon Bank,
1950 117,320.00 N.A.
1951 7,684.00
P151,859.10 3. That on or about June 29, 1977, Mellon Bank, N.A. filed a complaint with the
Court of First Instance of Rizal (now Regional Trial Court), (docketed as Civil Case
No. 26899), against the petitioner (private respondent herein), his wife and other

20
defendants, claiming that its remittance of US$1,000,000.00 was a clerical error We note that in the deficiency income tax assessment under consideration,
and should have been US$1,000.00 only, and praying that the excess amount of respondent (petitioner here) further requested petitioner (private respondent here)
US$999,000.00 be returned on the ground that the defendants are trustees of an to pay 50% surcharge as provided for in Section 72 of the Tax Code, in addition to
implied trust for the benefit of Mellon Bank with the clear, immediate, and the deficiency income tax of P4,888,615.00 and interest due thereon. Since
continuing duty to return the said amount from the moment it was received. petitioner (private respondent) filed his income tax return for taxable year 1977,
the 50% surcharge was imposed, in all probability, by respondent (petitioner)
because he considered the return filed false or fraudulent. This additional
4. That on or about November 5, 1977, the City Fiscal of Pasay City filed an
requirement, to our mind, is much less called for because petitioner (private
Information with the then Circuit Criminal Court (docketed as CCC-VII-3369-P.C.)
respondent), as stated earlier, reflected in as 1977 return as footnote that
charging the petitioner (private respondent herein) and his wife with the crime of
"Taxpayer was recipient of some money received from abroad which he presumed
estafa, alleging that they misappropriated, misapplied, and converted to their own
to be gift but turned out to be an error and is now subject of litigation."
personal use and benefit the amount of US$999,000.00 which they received under
an implied trust for the benefit of Mellon Bank and as a result of the mistake in the
remittance by the latter. From this, it can hardly be said that there was actual and intentional fraud,
consisting of deception willfully and deliberately done or resorted to by petitioner
(private respondent) in order to induce the Government to give up some legal right,
5. That on March 15, 1978, the petitioner (private respondent herein) filed his
or the latter, due to a false return, was placed at a disadvantage so as to prevent its
Income Tax Return for the taxable year 1977 showing a gross income of P53,053.38
lawful agents from proper assessment of tax liabilities. (Aznar vs. Court of Tax
and a net income of P48,053.88 and stating in the footnote of the return that
Appeals, L-20569, August 23, 1974, 56 (sic) SCRA 519), because petitioner literally
"Taxpayer was recipient of some money received from abroad which he presumed
"laid his cards on the table" for respondent to examine. Error or mistake of fact or
to be a gift but turned out to be an error and is now subject of litigation."
law is not fraud. (Insular Lumber vs. Collector, L-7100, April 28, 1956.). Besides,
Section 29 is not too plain and simple to understand. Since the question involved in
6. That on or before December 15, 1980, the petitioner (private respondent herein) this case is of first impression in this jurisdiction, under the circumstances, the 50%
received a letter from the acting Commissioner of Internal Revenue dated surcharge imposed in the deficiency assessment should be deleted. 7
November 14, 1980, together with income assessment notices for the years 1976
and 1977, demanding that petitioner (private respondent herein) pay on or before
The Commissioner of Internal Revenue, not satisfied with the respondent CTA's
December 15, 1980 the amount of P1,615.96 and P9,287,297.51 as deficiency
ruling, elevated the matter to us, by the present petition, raising the main issue as
assessments for the years 1976 and 1977 respectively. . . .
to:

7. That on December 15, 1980, the petitioner (private respondent herein) wrote the
WHETHER OR NOT PRIVATE RESPONDENT IS LIABLE FOR THE 50% FRAUD
Bureau of Internal Revenue that he was paying the deficiency income assessment
PENALTY?8
for the year 1976 but denying that he had any undeclared income for the year 1977
and requested that the assessment for 1977 be made to await final court decision
on the case filed against him for filing an allegedly fraudulent return. . . . On the other hand, Javier candidly stated in his Memorandum, 9 that he "did not
appeal the decision which held him liable for the basic deficiency income tax
(excluding the 50% surcharge for fraud)." However, he submitted in the
8. That on November 11, 1981, the petitioner (private respondent herein) received
same memorandum "that the issue may be raised in the case not for the purpose of
from Acting Commissioner of Internal Revenue Romulo Villa a letter dated October
correcting or setting aside the decision which held him liable for deficiency income
8, 1981 stating in reply to his December 15, 1980 letter-protest that "the amount of
tax, but only to show that there is no basis for the imposition of the surcharge."
Mellon Bank's erroneous remittance which you were able to dispose, is definitely
This subsequent disavowal therefore renders moot and academic the posturings
taxable." . . .5
articulated in as Comment10 on the non-taxability of the amount he erroneously
received and the bulk of which he had already disbursed. In any event, an appeal at
The Commissioner also imposed a 50% fraud penalty against Javier. that time (of the filing of the Comments) would have been already too late to be
seasonable. The petitioner, through the office of the Solicitor General, stresses that:
Disagreeing, Javier filed an appeal6 before the respondent Court of Tax Appeals on
December 10, 1981. The record however is not ambivalent, as the record clearly shows that private
respondent is self-convinced, and so acted, that he is the beneficial owner, and of
The respondent CTA, after the proper proceedings, rendered the challenged which reason is liable to tax. Put another way, the studied insinuation that private
decision. We quote the concluding portion: respondent may not be the beneficial owner of the money or income flowing to him
as enhanced by the studied claim that the amount is "subject of litigation" is belied
by the record and clearly exposed as a fraudulent ploy, as witness what transpired
upon receipt of the amount.

21
Here, it will be noted that the excess in the amount erroneously remitted by estafa for using the funds for his own personal benefit? and would the Government
MELLON BANK for the amount of private respondent's wife was $999,000.00 after refund it to him if the courts ordered him to refund the money to the Mellon Bank? 12
opening a dollar account with Prudential Bank in the amount of $999,993.70,
private respondent and his wife, with haste and dispatch, within a span of eleven
Under the then Section 72 of the Tax Code (now Section 248 of the 1988 National
(11) electric days, specifically from June 3 to June 14, 1977, effected a total
Internal Revenue Code), a taxpayer who files a false return is liable to pay the fraud
massive withdrawal from the said dollar account in the sum of $975,000.00 or
penalty of 50% of the tax due from him or of the deficiency tax in case payment
P7,020,000.00. . . .11
has been made on the basis of the return filed before the discovery of the falsity or
fraud.
In reply, the private respondent argues:
We are persuaded considerably by the private respondent's contention that there is
The petitioner contends that the private respondent committed fraud by not no fraud in the filing of the return and agree fully with the Court of Tax Appeals'
declaring the "mistaken remittance" in his income tax return and by merely making interpretation of Javier's notation on his income tax return filed on March 15, 1978
a footnote thereon which read: "Taxpayer was the recipient of some money from thus: "Taxpayer was the recipient of some money from abroad which he presumed
abroad which he presumed to be a gift but turned out to be an error and is now to be a gift but turned out to be an error and is now subject of litigation that it was
subject of litigation." It is respectfully submitted that the said return was not an "error or mistake of fact or law" not constituting fraud, that such notation was
fraudulent. The footnote was practically an invitation to the petitioner to make an practically an invitation for investigation and that Javier had literally "laid his cards
investigation, and to make the proper assessment. on the table."13

The rule in fraud cases is that the proof "must be clear and convincing" (Griffiths v. In Aznar v. Court of Tax Appeals,14 fraud in relation to the filing of income tax return
Comm., 50 F [2d] 782), that is, it must be stronger than the "mere preponderance was discussed in this manner:
of evidence" which would be sufficient to sustain a judgment on the issue of
correctness of the deficiency itself apart from the fraud penalty. (Frank A. Neddas,
. . . The fraud contemplated by law is actual and not constructive. It must be
40 BTA 672). The following circumstances attendant to the case at bar show that in
intentional fraud, consisting of deception willfully and deliberately done or resorted
filing the questioned return, the private respondent was guided, not by that "willful
to in order to induce another to give up some legal right. Negligence, whether slight
and deliberate intent to prevent the Government from making a proper assessment"
or gross, is not equivalent to the fraud with intent to evade the tax contemplated by
which constitute fraud, but by an honest doubt as to whether or not the "mistaken
law. It must amount to intentional wrong-doing with the sole object of avoiding the
remittance" was subject to tax.
tax. It necessarily follows that a mere mistake cannot be considered as fraudulent
intent, and if both petitioner and respondent Commissioner of Internal Revenue
First, this Honorable Court will take judicial notice of the fact that so-called "million committed mistakes in making entries in the returns and in the assessment,
dollar case" was given very, very wide publicity by media; and only one who is not respectively, under the inventory method of determining tax liability, it would be
in his right mind would have entertained the idea that the BIR would not make an unfair to treat the mistakes of the petitioner as tainted with fraud and those of the
assessment if the amount in question was indeed subject to the income tax. respondent as made in good faith.

Second, as the respondent Court ruled, "the question involved in this case is of first Fraud is never imputed and the courts never sustain findings of fraud upon
impression in this jurisdiction" (See p. 15 of Annex "A" of the Petition). Even in the circumstances which, at most, create only suspicion and the mere understatement
United States, the authorities are not unanimous in holding that similar receipts are of a tax is not itself proof of fraud for the purpose of tax evasion.15
subject to the income tax. It should be noted that the decision in the Rutkin case is
a five-to-four decision; and in the very case before this Honorable Court, one out of
A "fraudulent return" is always an attempt to evade a tax, but a merely "false
three Judges of the respondent Court was of the opinion that the amount in
return" may not be, Rick v. U.S., App. D.C., 161 F. 2d 897, 898.16
question is not taxable. Thus, even without the footnote, the failure to declare the
"mistaken remittance" is not fraudulent.
In the case at bar, there was no actual and intentional fraud through willful and
deliberate misleading of the government agency concerned, the Bureau of Internal
Third, when the private respondent filed his income tax return on March 15, 1978
Revenue, headed by the herein petitioner. The government was not induced to give
he was being sued by the Mellon Bank for the return of the money, and was being
up some legal right and place itself at a disadvantage so as to prevent its lawful
prosecuted by the Government for estafa committed allegedly by his failure to
agents from proper assessment of tax liabilities because Javier did not conceal
return the money and by converting it to his personal benefit. The basic tax
anything. Error or mistake of law is not fraud. The petitioner's zealousness to collect
amounted to P4,899,377.00 (See p. 6 of the Petition) and could not have been paid
taxes from the unearned windfall to Javier is highly
without using part of the mistaken remittance. Thus, it was not unreasonable for
commendable.1âwphi1 Unfortunately, the imposition of the fraud penalty in this
the private respondent to simply state in his income tax return that the amount
case is not justified by the extant facts. Javier may be guilty of swindling charges,
received was still under litigation. If he had paid the tax, would that not constitute
22
perhaps even for greed by spending most of the money he received, but the records collection of such tax may be begun without assessment, at any time within ten
lack a clear showing of fraud committed because he did not conceal the fact that he years after the discovery of the falsity fraud, or omission.
had received an amount of money although it was a "subject of litigation." As ruled
by respondent Court of Tax Appeals, the 50% surcharge imposed as fraud penalty
(b) Where before the expiration of the time prescribed in the preceding section for
by the petitioner against the private respondent in the deficiency assessment should
the assessment of the tax, both the Commissioner of Internal Revenue and the
be deleted.
taxpayer have consented in writing to its assessment after such time, the tax may
be assessed at any time prior to the expiration of the period agreed upon. The
WHEREFORE, the petition is DENIED and the decision appealed from the Court of period so agreed upon may be extended by subsequent agreements in writing made
Tax Appeals is AFFIRMED. No costs. before the expiration of the period previously agreed upon.

G.R. No. L-17438             April 30, 1964 (c) Where the assessment of any internal revenue, tax has been made within the
REPUBLIC OF THE PHILIPPINES, plaintiff-appellant, period of limitation above prescribed such tax may be collected by distraint or levy
vs. RITA LIM DE YU, defendant-appellee. or by a proceeding in court, but only if begun (1) within five years after the
assessment of the tax, or (2) prior to the expiration of any period for collection
agreed upon in writing by the Commissioner of Internal Revenue and the taxpayer
Appellee Rita Lim de Yu filed her yearly income tax returns from 1948 through
before the expiration of such five-year period. The period so agreed upon may be
1953. The Bureau of Internal Revenue assessed the taxes due on each return, and
extended by subsequent agreements in writing made before the expiration of the
appellee paid them accordingly. On July 17, 1956 the Bureau issued to appellee
period previously agreed upon.
deficiency income tax assessments for the years 1945 to 1953 in the total amount
of P22,450.50. She protested the assessments and requested a reinvestigation. On
August 30, 1956 she signed a "waiver" of the statute of limitations under the Tax The first issue raised by appellant is whether or not the returns filed by appellee for
Code as condition to the reinvestigation requested. Thereafter, or on July 18, 1958, the years 1948 to 1953 are false and fraudulent. Appellant maintains they are
the Bureau issued to her income tax assessment notices for the years 1948 to 1953 because the yearly net incomes reported in her returns are much less than as
totalling P35,379.63. This last assessment, like the one issued in 1956, covered not computed by the Bureau, and consequently, under par. (a), Section 332 of the Tax
only the basic deficiency income taxes, but also 50% thereof as surcharge. Upon Code, it has ten years from the date of the discovery of the fraud or falsity, i.e.,
appellee's failure to pay, an action for collection was filed against her in the Court of May 25, 1955, within which to assess the taxes or file a suit for collection without
First Instance of Cotabato on May 11, 1959. After trial the suit was dismissed, and assessment. And since, it is further contended, appellee can no longer question
the Government appealed to the Court of Appeals, which forwarded the case to this correctness of the assessment in view of her failure to the Court of Tax Appeals to
Court, the issues involved being purely legal. review the same, she should be ordered to pay the amounts being collected.

Appellant claims that the lower court erred (1) in ruling that the deficiency income But while fraud is alleged in the complaint, the same has not been established. It is
taxes due from appellee for the years 1948, 1949 and 1956 were not assessed on one thing to say that the correctness of the last assessment made by appellant, July
time; and (2) in dismissing the case on the ground that the right of appellant to 18, 1958, may no longer be challenged on the technical ground just stated and
collect the deficiency income tax assessment had already prescribed. quite another thing to say that appellee committed a deliberate fraud in declaring
small incomes for the years in which she filed her returns. Indeed the Bureau itself
appears none too sure as to the amounts of her net incomes for those years. On
Sections 331 and 332 of the Tax Code provide:
three different occasions it arrived at three highly different computation. First, it
accepted appellee's yearly statements of income from 1945 to 1953 and assessed
SEC. 331. Period of limitation upon assessment and collection. — Except as her a total tax P2,732,37, which she paid. Then in 1956 the Bureau came up with a
provided in the succeeding section, internal revenue taxes shall be assessed within different set of figures for the same period, considerably higher than those stated in
five years after the return was filed, and no proceeding in court without assessment the returns, and using such figures as basis assessed her deficiency taxes in the
for the collection of such taxes shall be begun after the expiration of such period. total amount of P22,450.50. Finally, in 1958 the Bureau made another computation
For the purposes of this section a return filed before the last day prescribed by law of appellee's net incomes for years 1948 to 1953, and assessed her deficiency taxes
for the filing thereof shall be considered as filed on such last day: Provided, That in the sum of P35,379.63. Note that the disparity between the 1956 and the 1958
this limitation shall not apply to cases already investigated prior to the approval of assessments is really much greater than what appears at first glance, as the latter
this Code. do not include the taxes corresponding to the years 1945, 1946 and 1947. Attention
may likewise be drawn to the fact that in paragraph 3 of the complaint appellant
SEC. 332. Exceptions as to period of limitation of assessment and collection of seeks to collect appellee the sum of P28.53, plus a surcharge of 50%, unpaid tax for
taxes. — (a) In the case of a false or fraudulent return with intent to evade tax or of the year 1948, notwithstanding the fact admitted in the stipulation, that appellee
a failure to file a return, the tax may be assessed, or a proceeding in court for the filed her return that year and duly paid the said amount.

23
Fraud not having been proven, the period of limitation for assessment or collection WHEREFORE, the appealed decision is modified by ordering appellee to pay
was five years from the filing of the return, according to Section 331 of the tax appellant the sum of P26,182.00 as deficiency income taxes for the years 1951,
code. The right to assess or collect the income taxes for the years 1948 to 1950 had 1952 and 1953, plus 5% surcharge and 1% monthly interest thereon from July 31,
already prescribed, therefore, when the Bureau of Internal Revenue issued the 1958 until payment of the full obligation, with costs.
deficiency income tax assessments on July 17, 1956.
G.R. No. L-17438             April 30, 1964
The tax years 1948 to 1950 cannot be deemed included in the "waiver of the statute REPUBLIC OF THE PHILIPPINES, plaintiff-appellant,
of limitations under the National Internal Revenue Code" executed by appellee on vs. RITA LIM DE YU, defendant-appellee.
August 30, 1956. The five-year period for assessment, counted from the date the
return is filed, may be extended upon agreement of the Commissioner and the
Appellee Rita Lim de Yu filed her yearly income tax returns from 1948 through
taxpayer, but such agreement must be made before, not after, the expiration of the
1953. The Bureau of Internal Revenue assessed the taxes due on each return, and
original period (Section 332 [b], Tax Code). The clear import of the provision is that
appellee paid them accordingly. On July 17, 1956 the Bureau issued to appellee
it does not authorize extension once prescription has attached.
deficiency income tax assessments for the years 1945 to 1953 in the total amount
of P22,450.50. She protested the assessments and requested a reinvestigation. On
The waiver validly covers only the tax years 1951 and 1952, with respect to which August 30, 1956 she signed a "waiver" of the statute of limitations under the Tax
the five-year period had not yet elapsed when the said waiver was executed. With Code as condition to the reinvestigation requested. Thereafter, or on July 18, 1958,
respect to the tax year 1953, as to which the return was filed by appellee on March the Bureau issued to her income tax assessment notices for the years 1948 to 1953
1, 1954, the waiver was not necessary for the effectivity of the assessment made totalling P35,379.63. This last assessment, like the one issued in 1956, covered not
on July 18, 1958, since such assessment was well within the original five-year only the basic deficiency income taxes, but also 50% thereof as surcharge. Upon
period provided by law. After the assessment on July 18, 1958, appellant had five appellee's failure to pay, an action for collection was filed against her in the Court of
years within which to file suit for collection pursuant to Section 332 (c) of the tax First Instance of Cotabato on May 11, 1959. After trial the suit was dismissed, and
code. Appellee's theory that collection could be made only up to the end of the the Government appealed to the Court of Appeals, which forwarded the case to this
period of extension stated in the waiver, namely, December 31, 1958, is without Court, the issues involved being purely legal.
merit. Assessment and collection are two different processes.
Appellant claims that the lower court erred (1) in ruling that the deficiency income
An assessment is not an action or proceeding for the collection of taxes. It is merely taxes due from appellee for the years 1948, 1949 and 1956 were not assessed on
a notice to the effect that the amount therein stated is due as tax and a demand for time; and (2) in dismissing the case on the ground that the right of appellant to
the payment thereof. It is a step preliminary, but essential to warrant distraint, if collect the deficiency income tax assessment had already prescribed.
still feasible, and, also, to establish a cause for "judicial action" as the phrase is,
used in section 316 of the Tax Code ... (Alhambra Cigar and Cigarette
Sections 331 and 332 of the Tax Code provide:
Manufacturing Company v. The Collector of Internal Revenue, L-12026, May 29,
1959).
SEC. 331. Period of limitation upon assessment and collection. — Except as
provided in the succeeding section, internal revenue taxes shall be assessed within
Section 331 gives the Government five years from filing of the return (which is not
five years after the return was filed, and no proceeding in court without assessment
false or fraudulent) within which to assess the tax due. Paragraph (b) of Section
for the collection of such taxes shall be begun after the expiration of such period.
332 allows the extension of this period by means of a written agreement between
For the purposes of this section a return filed before the last day prescribed by law
the taxpayer and the Commissioner of Internal Revenue. On the other hand,
for the filing thereof shall be considered as filed on such last day: Provided, That
paragraph (c) of the same section is concerned with the collection of the tax after
this limitation shall not apply to cases already investigated prior to the approval of
assessment, regardless of whether the assessment was made during the original
this Code.
five-year period or within an agreed period of extension. Collection then may be
effected within five years after assessment or within the "period for collection
agreed upon in writing by the Commissioner of Internal Revenue and the taxpayer SEC. 332. Exceptions as to period of limitation of assessment and collection of
before the expiration of such five-year period." Thus, although under the waiver taxes. — (a) In the case of a false or fraudulent return with intent to evade tax or of
appellee consented to the "assessment and collection" if made not later than a failure to file a return, the tax may be assessed, or a proceeding in court for the
December 31, 1958, such ,expiration date must be deemed to refer only to the collection of such tax may be begun without assessment, at any time within ten
extension of the assessment period. Insofar as collection is concerned, the period years after the discovery of the falsity fraud, or omission.
does not apply, for otherwise the effect of the waiver would be to shorten, not
extend, the legal period for that purpose. Appellant therefore had five years from (b) Where before the expiration of the time prescribed in the preceding
1958 within which to file his action, which was actually filed in 1959. section for the assessment of the tax, both the Commissioner of Internal
Revenue and the taxpayer have consented in writing to its assessment

24
after such time, the tax may be assessed at any time prior to the already prescribed, therefore, when the Bureau of Internal Revenue issued the
expiration of the period agreed upon. The period so agreed upon may be deficiency income tax assessments on July 17, 1956.
extended by subsequent agreements in writing made before the expiration
of the period previously agreed upon.
The tax years 1948 to 1950 cannot be deemed included in the "waiver of the statute
of limitations under the National Internal Revenue Code" executed by appellee on
(c) Where the assessment of any internal revenue, tax has been made August 30, 1956. The five-year period for assessment, counted from the date the
within the period of limitation above prescribed such tax may be collected return is filed, may be extended upon agreement of the Commissioner and the
by distraint or levy or by a proceeding in court, but only if begun (1) within taxpayer, but such agreement must be made before, not after, the expiration of the
five years after the assessment of the tax, or (2) prior to the expiration of original period (Section 332 [b], Tax Code). The clear import of the provision is that
any period for collection agreed upon in writing by the Commissioner of it does not authorize extension once prescription has attached.
Internal Revenue and the taxpayer before the expiration of such five-year
period. The period so agreed upon may be extended by subsequent
The waiver validly covers only the tax years 1951 and 1952, with respect to which
agreements in writing made before the expiration of the period previously
the five-year period had not yet elapsed when the said waiver was executed. With
agreed upon.
respect to the tax year 1953, as to which the return was filed by appellee on March
1, 1954, the waiver was not necessary for the effectivity of the assessment made
The first issue raised by appellant is whether or not the returns filed by appellee for on July 18, 1958, since such assessment was well within the original five-year
the years 1948 to 1953 are false and fraudulent. Appellant maintains they are period provided by law. After the assessment on July 18, 1958, appellant had five
because the yearly net incomes reported in her returns are much less than as years within which to file suit for collection pursuant to Section 332 (c) of the tax
computed by the Bureau, and consequently, under par. (a), Section 332 of the Tax code. Appellee's theory that collection could be made only up to the end of the
Code, it has ten years from the date of the discovery of the fraud or falsity, i.e., period of extension stated in the waiver, namely, December 31, 1958, is without
May 25, 1955, within which to assess the taxes or file a suit for collection without merit. Assessment and collection are two different processes.
assessment. And since, it is further contended, appellee can no longer question
correctness of the assessment in view of her failure to the Court of Tax Appeals to
An assessment is not an action or proceeding for the collection of taxes. It
review the same, she should be ordered to pay the amounts being collected.
is merely a notice to the effect that the amount therein stated is due as tax
and a demand for the payment thereof. It is a step preliminary, but
But while fraud is alleged in the complaint, the same has not been established. It is essential to warrant distraint, if still feasible, and, also, to establish a cause
one thing to say that the correctness of the last assessment made by appellant, July for "judicial action" as the phrase is, used in section 316 of the Tax Code ...
18, 1958, may no longer be challenged on the technical ground just stated and (Alhambra Cigar and Cigarette Manufacturing Company v. The Collector of
quite another thing to say that appellee committed a deliberate fraud in declaring Internal Revenue, L-12026, May 29, 1959).
small incomes for the years in which she filed her returns. Indeed the Bureau itself
appears none too sure as to the amounts of her net incomes for those years. On
Section 331 gives the Government five years from filing of the return (which is not
three different occasions it arrived at three highly different computation. First, it
false or fraudulent) within which to assess the tax due. Paragraph (b) of Section
accepted appellee's yearly statements of income from 1945 to 1953 and assessed
332 allows the extension of this period by means of a written agreement between
her a total tax P2,732,37, which she paid. Then in 1956 the Bureau came up with a
the taxpayer and the Commissioner of Internal Revenue. On the other hand,
different set of figures for the same period, considerably higher than those stated in
paragraph (c) of the same section is concerned with the collection of the tax after
the returns, and using such figures as basis assessed her deficiency taxes in the
assessment, regardless of whether the assessment was made during the original
total amount of P22,450.50. Finally, in 1958 the Bureau made another computation
five-year period or within an agreed period of extension. Collection then may be
of appellee's net incomes for years 1948 to 1953, and assessed her deficiency taxes
effected within five years after assessment or within the "period for collection
in the sum of P35,379.63. Note that the disparity between the 1956 and the 1958
agreed upon in writing by the Commissioner of Internal Revenue and the taxpayer
assessments is really much greater than what appears at first glance, as the latter
before the expiration of such five-year period." Thus, although under the waiver
do not include the taxes corresponding to the years 1945, 1946 and 1947. Attention
appellee consented to the "assessment and collection" if made not later than
may likewise be drawn to the fact that in paragraph 3 of the complaint appellant
December 31, 1958, such ,expiration date must be deemed to refer only to the
seeks to collect appellee the sum of P28.53, plus a surcharge of 50%, unpaid tax for
extension of the assessment period. Insofar as collection is concerned, the period
the year 1948, notwithstanding the fact admitted in the stipulation, that appellee
does not apply, for otherwise the effect of the waiver would be to shorten, not
filed her return that year and duly paid the said amount.
extend, the legal period for that purpose. Appellant therefore had five years from
1958 within which to file his action, which was actually filed in 1959.
Fraud not having been proven, the period of limitation for assessment or collection
was five years from the filing of the return, according to Section 331 of the tax
WHEREFORE, the appealed decision is modified by ordering appellee to pay
code. The right to assess or collect the income taxes for the years 1948 to 1950 had
appellant the sum of P26,182.00 as deficiency income taxes for the years 1951,

25
1952 and 1953, plus 5% surcharge and 1% monthly interest thereon from July 31, amounting to P30,080.00, exclusive of penalties, surcharges and interests, due on
1958 until payment of the full obligation, with costs. the income received by Haig Assadourian for the years 1947 and 1948 pursuant to
the provisions of Section 53 (b) and (c) of the National Internal Revenue Code. In
addition, defendant Jose Razon is sought to be held liable for the payment of the
G.R. No. L-17462             May 29, 1967
said sum of P30,080.00 for being the attorney-in-fact of Haig Assadourian.
REPUBLIC OF THE PHILIPPINE, petitioner,
vs. JOSE RAZON and JAI-ALAI CORPORATION, respondents.
----------------------------- On February 6, 1953, defendant Jose Razon filed his answer to the amended
G.R. No. L-17472             May 29, 1967 complaint. On July 23, 1953, defendant Jai-Alai Corporation filed its answer to the
JAI-ALAI CORPORATION OF THE PHILIPPINES, petitioner, amended complaint, with a cross-claim against its co-defendant Jose Razon, who in
vs. turn filed, on August 1, 1953, his answer to the cross-claim. On April 6, 1955, the
REPUBLIC OF THE PHILIPPINES, COURT OF TAX APPEALS and JOSE Court of First Instance of Manila remanded this case to this Court for final
RAZON, respondents. determination and disposition pursuant to Section 22 of Republic Act No. 1125.

Appeals taken by the Republic of the Philippines — hereinafter referred to as the Anent the first cause of action, the following facts were established: Haig
Republic — and the Jai Alai Corporation of the Philippines — hereinafter referred to Assadourian, a citizen of Egypt, was previously admitted to the Philippines for
as the Jai Alai — from the decision rendered by the Court of Tax Appeals on August permanent residence. (Exh. 2-Jose Razon, p. 42 CTA rec.) He was considered a
4, 1960 in Case No. 15566, ordering the Jai Alai to pay to the Republic the sum of legal resident of the Philippines as of September 13, 1940 (Exh. 1-Jose Razon, pp.
P12,000.00 representing the 12% withholding tax on the amount of P80,000.00 it 40-41 CTA rec.), and resided in the Philippines from 1940 to 1945. (Exh. A & 14-
paid to Haig Assadourian, plus surcharge, and dismissing the complaint, insofar as it Jai-Alai, p. 470 BIR rec.) He had his residence at the University of Santo Tomas,
concerned defendant Jose Razon, as well as the cross-claim of the Jai-Alai against España St., Manila, and was the general manager of the Jai-Alai Corporation. (Exh.
the latter. 1-Jose Razon, p. 40 CTA rec.)

In view of Razon's death on February 11, 1961, Marina Baretto, his widow and With the intention of going to the United States for rest and business, Haig
administratrix of his Intestate Estate (Special Proceeding No. 46754 of the Court of Assadourian filed, on October 19, 1945, an application for a re-entry permit with the
First Instance of Manila) was substituted in his place on July 17 of the same year. Bureau of Immigration, stating in his application among others, that the length of
his proposed absence in the Philippines was indefinite. He was granted the permit to
reenter the Philippines as a non-immigrant, with expiry date on October 19, 1946,
As the two appeals are interrelated and involve common issues, we consider them
which was extended for another year ending on October 19, 1947. (Exh. 2-Jose
jointly in this decision.
Razon, p. 42 CTA rec.)

The case and the facts involved — mostly, if not all — undisputed are stated in the
On March 4, 1946, a certain JACK GEORGE, residing at 529 Aviles, Manila, executed
appealed decision as follows:
a "Guaranty" (Exhs. 3-Jose Razon & 4-Jai-Alai, p. 215 BIR rec.), assuming the tax
responsibility of Haig Assadourian and binding himself "to pay such internal revenue
This is an action instituted by the Republic of the Philippines to collect from taxes, surcharges and interests as may be found to be due from him" (Haig
defendant Jose Razon the amount of P73,522.62, as alleged income tax due from Assadourian) prior to his departure or during his absence from the Philippines.
Haig Assadourian for the year 1946, including surcharges and interests up to
December 31, 1951, and from defendant Jose Razon and Jai-Alai Corporation,
On March 5, 1946, Haig Assadourian filed an application for a tax clearance
jointly and severally, the sum of P30,080.00 exclusive of penalties, surcharges and
certificate (Exhs. A & 14-Jai-Alai, p. 470 BIR rec.) with the Bureau of Internal
interests, as income taxes due from Haig Assadourian, for the years 1947 and
Revenue, stating among others, that he was leaving the Philippines for the United
1948:
States on March 8 or 9, 1946, and that for the purpose, of filing his tax returns,
paying and compromising taxes that may be assessed against him during his
The original complaint was filed with the Court of First Instance of Manila on absence, he has appointed defendant Jose Razon. On the same day, Haig
January 8, 1952 solely against defendant Jose Razon. On January 16, 1953, the Assadourian was issued Tax Clearance Certificate No. V-2257 (Exhs. 9-Jai-Alai & 12-
plaintiff amended its complaint, including the Jai-Alai Corporation as party Jose Razon, p. 94, BIR rec.) on the strength of the guaranty signed by Jack George.
defendant, and asserting two (2) causes of action. Under the first cause of action, (Exh. 9-A Jose Razon, p. 257 BIR rec.) He departed in 1946 and since then he
the plaintiff seeks to recover from defendant Jose Razon as attorney-in-fact of Haig never returned to the Philippines. His last known address is Los Angeles, California,
Assadourian, the amount of P73,522.62 as the latter's income tax liability for the U.S.A. (Exh. K, p. 57 CTA rec.)
year 1946, computed as of December 31, 1951; and, under the second cause of
action the plaintiff seeks to recover jointly and severally from defendants Jose
During his stay in the Philippines, Haig Assadourian did not engage in trade or
Razon and Jai-Alai Corporation as withholding agents, the 12% withholding tax
business. However his wife, Mrs. Valentina Assadourian, was a partner in the
26
business firm known as Alaska Ice Cream Factory (Exh. Z, p. 183 BIR rec.). He had acknowledged full payment of the latter's claim for management percentage fees
no properties, real or personal, except the 100 shares of stock in the Jai-Alai earned for the year's 1946 to 1950 and any or all future claims against the Jai-Alai
Corporation and another 100 shares in the name of his wife in the same Corporation. The payment of percentage fees to Haig Assadourian was in
corporation. (Exh. Y, p. 184 BIR rec.). accordance with certain management contracts, dated July 30, 1939 and November
18, 1940, with the Jai-Alai Corporation, by virtue of which Haig Assadourian
operated the Jai-Alai Fronton, and which contracts were to continue up to the end of
On March 1, 1947, Jose Razon, in behalf of Haig Assadourian, filed an income tax
the year 1950, but were terminated by the indenture. Finally under the terms of the
return for 1946 (Exh. B, p. 469 BIR rec.), showing a net income of P156,194.93 and
indenture, the sum of P200,000.00 was to be paid by the Jai-Alai Corporation in
giving as source thereof his salary and other income from the Alaska Ice Cream
installments as follows: (1) P40,000.00 on or before August 20, 1947; and (2)
Factory. On the same day, Jose Razon, as the "duly named attorney-in-fact" of Haig
P20,000.00 on or before the 20th day of each month, beginning the month of
Assadourian, wrote the Collector of Internal Revenue (Exh. C, p. 468 BIR rec.)
September of 1947 up to and including the month of April, 1948.
requesting approval of an installment plan for the payment of the income tax
liability for 1940, 1941 and 1946.
On the basis of the payments to Haig Assadourian, Examiner Rosales filed for Haig
Assadourian an income tax return for 1947 (Exh. L, p. 456 BIR rec.; Exh. Jai-Alai p.
On March 19, 1947 (Exh. D, pp. 466-467 BIR rec.), the Deputy Collector of Internal
426 BIR rec.), showing the income of P120,000.00 and another income tax return
Revenue approved the installment plan for the payment of the 1940 and 1941
for 1948 (Exh. M, p. 855 BIR rec.; Exh. 3- Jai-Alai, p. 428 BIR rec.), showing an
deficiency income tax of Haig Assadourian, but denied the request for the payment
income of P20,000.00.
of the 1946 income tax by installment, and at the same time enclosed a tax
assessment notice (Exh. B, p. 369 BIR rec.) calling for the payment of P48,731.87.
As the assessment was not paid, the Collector of Internal Revenue, on September On November 10, 1952, the Collector of Internal Revenue, wrote a letter to
18, 1948, addressed another letter to Jose Razon (Exh. F, pp. 464-465 BIR rec.) defendant Jai-Alai Corporation, demanding the payment of taxes corresponding to
again demanding the payment of the 1940, 1941 and 1946 income tax liability of 12% of the amount of P200,000.00 which it allegedly paid to Haig Assadourian and
Haig Assadourian, which, as of October 15, 1948, was in the total amount of which it should have withheld according to Section 53 (b) and (c) of the Tax Code
P64,506.34. (Exh. N, p. 453-454, BIR rec.), enclosing therewith assessment notices Nos. AR-
1060-52/47 and AR-106152/48 for the years 1947 and 1948, respectively, in the
total amount of P30,080.00 (Exhs. O & F, pp. 452 & 450 BIR rec.).
Meanwhile, on May 2, 1950, the Collector of Internal Revenue wrote a letter to Jack
George (Exh. 18-Jose Razon, p. 187 BIR rec.), the guarantor of the tax liabilities of
Haig Assadourian, demanding from him the payment of the 1940, 1941 and 1946 On November 22, 1952, J. Laurea, in behalf of the Jai-Alai Corporation, wrote a
tax liability of Haig Assadourian. Since Jack George refused to pay the tax liability of letter (Exh. S-Jai-Alai, p. 328 BIR rec.) to the Collector of Internal Revenue,
Haig Assadourian, the Collector of Internal Revenue, on January 9, 1951, wrote the requesting the latter to rescind the assessment on the ground that, owing to the
Solicitor General (Exh. 8-Jose Razon, pp. 194-197 BIR rec.), requesting that a civil financial inability of the Jai-Alai Corporation "to pay the purchase price to Mr. Haig
action be filed against Jack George, as guarantor of Haig Assadourian to collect the Assadourian, decided to subrogate its rights to one of the stockholders of the
income tax liabilities for 1940, 1941 and 1946 of the latter in the total amount of Corporation, Senator Vicente Madrigal" and "instead of the Jai-Alai Corporation
P77,142.03 as of November 21, 1950. paying for the price contracted, Senator Madrigal paid the agreed price to the legal
representative of Mr. Assadourian, who was then in the United States."
Subsequently, on July 9, 1951 (Exh. G, p. 463, BIR rec.), August 3, 1951 (Exh. H,
p. 462 BIR rec.), September 25, 1951 (Exh, 1, p. 459 BIR rec.) and December 11, On December 19, 1952, a similar letter of demand (Exh. Q, pp. 447-448 BIR rec.)
1951 (Exh. J, p. 458 BIR rec.), the Collector of Internal Revenue sent follow-up was sent to Jose Razon, enclosing the same assessment notices for the years 1947
letters to Jose Razon for the payment of the assessed tax liability of Haig and 1948, in the total amount of P30,080.00 (Exhs. R & S, pp. 444, 445, BIR rec.).
Assadourian. According to the latest letter, Exhibit J (p. 458 BIR rec.), the 1946
income tax liability of Haig Assadourian amounted to P73,522.62 as of December
Acting on the information contained in the letter of J. Laurea, the Collector of
31, 1951.
Internal Revenue, on December 29, 1952, wrote a letter (Exh. 12-Jai-Alai, pp. 335-
336 BIR rec.) to Vicente Madrigal, assessing and demanding from him the payment
Relative to the second cause of action, the following facts were established. In of the withholding tax in the amount of P30,080.00 on the P200,000.00 he paid to
1949, while BIR Examiner Narciso Rosales was examining the books of the Jai-Alai Haig Assadourian for the year 1947 and 1948 and which he should have withheld
Corporation for income tax purposes, he came across an indenture (Exh. K, pp. 57- pursuant to Section 53 (b) and (c) of the Tax Code. Meanwhile, on January 2, 1953,
58, CTA rec.) executed between Haig Assadourian and the Jai-Alai Corporation. It the Collector of Internal Revenue wrote to the Solicitor General, requesting the
appears from this indenture that on August 6, 1947, the Jai-Alai Corporation, duly inclusion of Vicente Madrigal as party defendant in this case, (Exh. 13-Jai-Alai, p.
represented by its Vice-President, Jose Razon, entered into a contract in the City of 339 BIR rec.)
Los Angeles, California, U.S.A. with Haig Assadourian, as manager of the Jai-Alai
Corporation, whereby the latter, in consideration of the amount of P200,000.00,

27
On January 10, 1953, J. Laurea, this time acting for and in behalf of Vicente became personally liable because of his failure to withhold the tax due on the
Madrigal, wrote a letter (Exh. 6-Jai-Alai, p. 442 BIR rec.) to the Collector of Internal income of Assadourian.
Revenue in connection with the letter of demand dated December 29, 1952,
claiming among others, that as "Senator Madrigal has made the payment of the
Having thus limited and clarified the true issue before Us, We now proceed to
purchase price of the right to the legal representative of Mr. Assadourian in Manila,"
consider the claim of Jai-Alai (and also of Razon as appellee) to the effect that
he is not liable for the payment of withholding tax.
Assadourian was not and could not be considered as a non-resident alien not
engaged in trade or business in the Philippines within the purview of Section 53 (b)
The substantial points raised by the Republic as appellant (G.R. No. L-17462) are of the National Internal Revenue Code which provides:
that the Court of Tax Appeals, erred: firstly, in not holding the Jai-Alai liable, under
Section 53 (b) of the National Internal Revenue Code, for the payment of
(b) Nonresident aliens. — All persons, corporations and general co-partnerships
withholding tax on the amount of P120,000.00 paid by Madrigal & Co., Inc. or by
(companias colectivas), in whatever capacity acting, including lessees or mortgagors
Vicente Madrigal to Assadourian; secondly, in not holding Jose Razon, similarly
of real property, trustees acting in any trust capacity, executors, administrators,
liable, under the same legal provision, for the tax due on sum of P160,000.00 paid
receivers, conservators, fiduciaries, employers, and all officers and employees of the
to the same party, and lastly, in not awarding 1% interests monthly on the amount
Government of the Philippines having the control, receipt, custody, disposal, or
the Court found due from the Jai-Alai to the Republic, in accordance with Section 51
payment of interest, dividends, rents, salaries, wages, premiums, annuities,
(e) of the National Internal Revenue Code.
compensation, remunerations, emoluments, or other fixed or determinable annual
or periodical gains, profits, and income of any non-resident alien individual, not
On the other hand, the Jai-Alai as appellant (G.R. No. L-17472) claims that the engaged in trade or business within the Philippines therein, shall (except in the
Court of Tax Appeals erred firstly, in holding that the money paid to Assadourian by cases provided for in subsection (a) of this section) deduct and withhold from such
Jai-Alai was salary, emolument, remuneration or determinable profit or income annual or periodical gains, profits, and income a tax equal to twenty per centum
within the purview of Section 53 (b) of the National Internal Revenue Code, instead thereof: Provided, That no such deduction or withholding shall be required in the
of declaring that it was the consideration of a contract of purchase and sale of an case of dividends paid by a foreign corporation unless (1) such corporation is
inchoate or contingent interest pertaining to Assadourian, and erred, consequently engaged in trade or business within the Philippines and (2) more than eighty-five
in finding the Jai-Alai liable as withholding agent for the payment of the total sum of per centum of the gross income of such corporation for the three-year period ending
P12,000.00 representing the alleged withholding tax on the amount of P80,000.00 it with the close of its taxable year preceding the declaration of such dividends (or for
paid to Assadourian; secondly, in holding the aforesaid provision of the National such part of such period as the corporation has been in existence) was derived from
Internal Revenue Code applicable to the Jai-Alai inspite of the fact that Assadourian sources within the Philippines as determined under the provisions of section thirty-
— to whom the amount aforesaid was paid — was not and could not be considered seven: Provided, Further, That the Commissioner of Internal Revenue may
as a non-resident alien not engaged in trade or business in the Philippines within authorize such tax to be deducted and withheld from the interest upon any
the purview of the tax code; thirdly, in finding that when the payments in question securities the owner of which are not known to the withholding agent.
were made to, and accepted by Jose Razon, the latter, as Vice-President of the Jai-
Alai, was but an extension of the latter's personality, and finally, in not holding that
As is obvious, the above contention is decisive upon Razon and Jai-Alai's liability as
the right to recover said withholding tax from the Jai-Alai had already prescribed.
withholding agents. Were it true that Assadourian was not and could not be
considered as a non-resident alien not engaged in trade or business in the
Although the Court of Tax Appeals dismissed, as against the now deceased Jose Philippines, there could be no case for the application of the legal provision.
Razon, the two causes of action alleged in the complaint filed by the Republic, it concerning withholding agents, with the result that the Republic would have no case
appears from the errors assigned in the latter's brief in G.R. No. L-17462 that it against the Jai-Alai and Razon.
questions only that portion of the appealed decision which disposes of the second
cause of action where it seeks to hold Razon and the Jai-Alai jointly and severally
The question, however, is one no longer reviewable because it is one of fact. The
liable as withholding agents for the withholding tax due on the income received by
Court of Tax Appeals having definitely found and declared as a fact that Assadourian
Assadourian for the years 1947 and 1948. This appeal, therefore, does not cover
was a non-resident alien not engaged in trade or business in the Philippines, We
that portion of the aforesaid decision dismissing the first cause of action which was
must necessarily follow the settled rule in this jurisdiction that findings of fact of the
for the recovery from Jose Razon alone of the amount of P73,522.62 representing
Court of Tax Appeals are not reviewable by Us, as long as they are supported by
Assadourian's alleged income tax liability for the year 1946, computed as of
substantial evidence — which, in our opinion, is precisely the case in the one now
December 31, 1951.
before Us (Gutierrez, etc. vs. Court of Tax Appeals, et al., 54 O.G. 2912; Section 2,
Rule 44, Rules of Court).
It must be stated further that the Republic sued Razon in relation to both causes of
action either as guarantor of Assadourian's tax liability, or as the latter's attorney-
Jai Alai's next contention is that the total amount of P200,000.00 paid to
in-fact with the obligation to file his income tax returns and pay or compromise the
Assadourian in the years 1947 and 1948 was not payment of "interest, dividends, . .
tax due on his income, or — inferentially at least — in his personal capacity as he
. compensation, remunerations, emoluments, or other fixed or determinable annual

28
or periodical gains, profits, and income" within the purview of Section 53 (b) of the corresponding withholding tax should have been withheld by "such persons,
National Internal Revenue Code, but was, instead, the consideration for the sale of corporations, and general co-partnerships who had the control, receipt, custody,
an inchoate or contingent interest belonging to Assadourian to which said legal disposal, or payment thereof to the person entitled to it."
provision does not apply.
But both the Jai-Alai and Razon claim not to fall under the provisions of Section 53
When and how, on several occasions, the amount aforesaid was paid to Assadourian (b) of the National Internal Revenue Code precisely because they did not have the
is shown by the record as follows: control, receipt, custody, or disposal of the alleged taxable amount. Again we find
this to be without merit.
In the year 1947, the amount of P40,000.00 was paid directly by the Jai-Alai on
September 2 by telegraphic transfer (through the Bank of the Philippine Islands) With respect to the P40,000.00 sent or paid by the Jai-Alai itself by telegraphic
sent to Assadourian at 277 South Beverly Drive, Beverly Hills, California, U.S.A. On transfer to Assadourian on September 2, 1947, its liability is beyond question.
November 7, November 20, December 17, and December 26, all of the same year
1947, four different amounts of P20,000.00 cash were paid, the first two by the Jai-
With respect to the two different amounts of P20,000.00, each paid to Jose Razon
Alai and the remaining two by Madrigal & Company, Inc. to Jose Razon for and on
on November 7 and November 20 of the year 1947, it is also clear that it was the
behalf of Assadourian. On February 11, March 11, April 10 and June 2, all of the
Jai Alai, through its then Vice-President Jose Razon, who had custody and had
year 1948, four separate amounts of P20,000.00 each were paid by Madrigal and
disposed of and paid said amounts. In this connection it must be borne in mind that,
Company, Inc., to Jose Razon. The latter remitted all said amounts to Assadourian
in connection with these two payments and the later payments made in the years
at the address already referred to.
1947 and 1948, Razon had a dual personality: he acted as the Vice-President and,
therefore, the representative of the Jai-Alai, and at the same time as the attorney-
According to the evidence, Assadourian, an Egyptian citizen, was admitted to the in-fact of Assadourian. In Exhibit C (BIR record p. 468) he clearly admitted that he
Philippines on September 13, 1940. From October of that year to January 1945, he was such attorney-in-fact. When he received the amounts from the Jai-Alai to effect
was the general manager of the Jai-Alai Corporation, a duly organized entity under the payment to Assadourian, he at the same time accepted the payment as
the laws of the Philippines engaged during all that time, under the name of Jai-Alai attorney-in-fact of the latter with the obligation of remitting them to him — as, in
Stadium, in a form of legalized gambling, in which corporation Assadourian and his fact, he did. Consequently, not only the Jai-Alai but Razon also must be deemed to
wife owned 200 shares of stock. On March 5, 1946, after he had appointed Jose be withholding agents within the purview of Section 53 (b) of the National Internal
Razon as his attorney-in-fact or agent "for the purpose of filing my (his) tax returns Revenue Code, and it was their duty, before actually paying the amounts to
and paying and compromising the taxes which may be assessed against me (him) Assadourian, to deduct the corresponding income tax due thereon.
during my (his) absence" and after securing a tax clearance upon the guaranty of
one Jack George, Assadourian left the Philippines for the United States. Since then
In relation to the different amounts of P20,000.00 each paid to Jose Razon under
he had been residing in the City of Los Angeles, California. His re-entry permit
similar circumstances on December 17 and December 24, 1947, and on February
expired, in fact, on October 14, 1947. On August 5, 1947 and in said city, the Jai-
11, March 11, April 10 and June 2, of the year 1948, it is claimed that they were not
Alai, represented by its Vice-President Jose Razon, entered into a contract with
payments made by the Jai-Alai but by Madrigal & Company, Inc., and that inasmuch
Assadourian, whereby the latter, in consideration of the sum of P200,000.00,
as the former did not have the control over said amounts and did not dispose of or
acknowledged full payment of all his claim for percentages earned by the Jai-Alai
pay them to Assadourian or his representative, the legal provision already referred
Stadium for the years 1940 to 1945, and to be earned during the years 1946 to
to does not apply to it. This could be true, indeed, if the record did not sufficiently
1950 (Exhibit K). The contract expressly terminated the management contracts
disclose that Vicente Madrigal was the controlling stockholder of the Madrigal &
executed between the same parties on July 30, 1939 and November 18, 1940,
Company, Inc. and of the Jai-Alai Corporation up to the time of the trial below
pursuant to which Assadourian, as manager of the Jai-Alai Stadium located in the
(transcript, p. 207). In fact, the payments of P20,000.00 each made in the name of
City of Manila, was to receive a certain percentage of the receipts of the business.
Madrigal & Company, Inc. were charged to the personal account of Vicente
These "percentages earned, accrued, and uncollected for the years 1940 to 1945,
Madrigal. Therefore, piercing the veil of corporate fiction, it can be said that said
both inclusive," and "percentages to be earned or accrued during the years 1946 to
payments, albeit made in the name of Madrigal & Company, Inc. and later charged
1950 (Exhibit K) were the ones which Assadourian considered fully paid with the
to the personal account of Vicente Madrigal, were really payments made by the Jai-
aforesaid sum of P200,000.00 which he subsequently received.
Alai.

The bare facts stated above are, in our opinion, sufficient to show the lack of merit
What has been said heretofore leads us inexorably to the conclusion that the Jai-Alai
of Jai-Alai's contention that the aforesaid sum was the purchase price of certain
was a withholding Agent, and as such should have withheld the corresponding tax
inchoate or contingent interest belonging to Assadourian. To the contrary, the
from the total amount of P200,000.00, pursuant to Section 53, subparagraph (b)
undisputed facts of the case show beyond cavil that said amount was in payment
and (c) of the Revised Internal Revenue Code.
of percentages or income earned in Assadourian during the years aforesaid out of
the profits realized by the Jai-Alai Stadium. Being so, it was taxable, and the

29
Razon or his Intestate Estate, however, may be held liable in the same capacity only ......
as regards the total amount of P160,000.00 which he had received and disposed of 25%
simultaneously as Vice-President of the Jai-Alai and as attorney-in-fact of 2,400.00
surcharge . . . . . . . . . . . . . . . . . . . . . . .
Assadourian, because he had no part in the payment of the first amount of Total amount due . . . . . . . . . . . . . . P12,000.00
P40,000.00 by the Jai-Alai through telegraphic transfer sent directly to Assadourian
12% withholding tax on the amount of
in the City of Los Angeles, California.
P80,000.00 paid to Assadourian, through
him, in the year 1948 . . . . . . . . . . . . . . .
With the above, We consider as sufficiently disposed of the first, third and fourth ... P9,600.00
assignments of error made by the Republic in its brief as appellant, as well as the 25%
first, second and third assignments of error made in the Jai-Alai's brief in the same 2,400.00
surcharge . . . . . . . . . . . . . . . . . . . . . . .
capacity. P12,000.00
Total amount due . . . . . . . . . . . . . .
As a result, the decision appealed from is modified by sentencing the Jai-Alai to pay
the Republic the corresponding withholding tax on the total of the amounts paid to
Assadourian, as follows: In addition, the same Intestate Estate is ordered to pay interest on both amounts at
the rate of 1% per month (Section 51 [d] and [e] of the Revised Internal Revenue
Code) from January 15, 1953, in accordance with the income tax assessment
12% withholding tax on the total amoun notices Exhibits R and S.
to of P120,000.00 paid to Assadourian the
year 1947 . . . . . . . . . . . . . . . . . . . . . . .
..... P14,400.00 The liability of the Jai-Alai and of the Intestate Estate of the deceased Jose Razon
25 % surcharge . . . . . . . . . . . . . . . . . . . for the withholding taxes, surcharges, and interests due on the amount of
9,600.00 P80,000.00 paid to Assadourian in the year 1947 and the total amount of
....
P80,000.00 paid to the same party in the year 1948 shall be joint and several.

Total amount due . . . . . . . . . . . . . . . . P18,000.00


The procedural question raised by the Republic in its second assignment of error
needs no further discussion in view of our resolution on the other four assignments
On the total amount of P80,000.00 paid to of error of the same party.
the same party in the year 1948, 12%
withholding
Now, to the last point raised in the brief of the Jai-Alai as appellant to the effect that
tax . . . . . . . . . . . . . . . . . . . . . . . . P9,600.00
the cause of action of the Republic to recover the withholding taxes in question had
25% already prescribed when the action was commenced below. In refutation thereof,
2,400.00
surcharge . . . . . . . . . . . . . . . . . . . . . . . We deem it sufficient to quote the following portion of the decision under review:

Total amount due . . . . . . . . . . . . . . . . 12,000.00 In connection with the defense of defendant Jai-Alai Corporation that the
right to collect the tax has already prescribed, the record shows that it
failed to file a withholding tax return for the amount of P80,000.00 paid to
Haig Assadourian in 1947. For its omission to file a withholding tax return,
In addition to these amounts of P18,000.00 and P12,000.00, the Jai-Alai is ordered Section 332 (c) of the Tax Code, which provides that 'a proceeding in court
to pay interest at the rate of 1% per month (Section 51 [d] and [e] of the Revised for the collection of such tax may be begun without assessment, at any
Internal Revenue Code) from November 20, 1952, in accordance with the income time within ten years after the discovery of the . . . omission,' should be
tax assessment notices Exhibits O and P. applied. The failure to file a return was discovered in 1949, during the
investigation conducted by BIR examiner Narciso Rosales. The judicial suit
Similarly, judgment is hereby rendered sentencing the Intestate Estate of Jose was initiated on January 16, 1953 when the Jai-Alai Corporation was
Razon, herein represented by the Administratrix, to pay the Republic as follows: included as party defendant in the amended complaint. Only four (4) years
elapsed from the time of the discovery of the omission to file a return to
the filing of a judicial action against the Jai-Alai Corporation consequently,
12% withholding tax on the total amount P9,600.00 the right to judicially collect the withholding tax has not prescribed.
paid to Assadourian through him,
amounting to P880,000.00, in the year
1947 . . . . . . . . . . . . . . . . . . . . . . . . . .

30
Wherefore, modified as above indicated, the decision appealed from is affirmed in defendant's breach of the contract of carriage and that, consequently, articles 1101-
all other respects. With costs. 1107 of the Civil Code, and not article 1903, are applicable. The court further found
that the breach of the contract was not due to fortuitous events and that, therefore,
the defendant was liable in damages.
G.R. No. L-19495             February 2, 1924
HONORIO LASAM, ET AL., plaintiffs-appellants,
vs. FRANK SMITH, JR., defendant-appellant. In our opinion, the conclusions of the court below are entirely correct. That upon the
facts stated the defendant's liability, if any, is contractual, is well settled by previous
decisions of the court, beginning with the case of Rakes vs. Atlantic, Gulf & Pacific
The plaintiff are husband and wife and this action is brought to recover damages in
Co. (7 Phil., 359), and the distinction between extra-contractual liability and
the sum of P20,000 for physical injuries sustained by them in an automobile
contractual liability has been so ably and exhaustively discussed in various other
accident. The trial court rendered a judgment in their favor for the sum of
cases, that nothing further need here be said upon that subject. (See Cangco vs.
P1,254.10, with legal interest from the date of the judgment. Both the plaintiffs and
Manila Railroad Co., 38 Phil., 768; Manila Railroad Co. vs. Compania Trasatlantica
the defendant appeal, the former maintaining that the damages awarded are
and Atlantic, Gulf & Pacific Co., 38 Phil., 875; De Guia vs. Manila Electric Railroad &
insufficient while the latter denies all liability for any damages whatever.
Light Co., 40 Phil., 706.) It is sufficient to reiterate that the source of the
defendant's legal liability is the contract of carriage; that by entering into that
It appears from the evidence that on February 27, 1918, the defendant was the contract he bound himself to carry the plaintiffs safely and securely to their
owner of a public garage in the town of San Fernando, La Union, and engaged in the destination; and that having failed to do so he is liable in damages unless he shows
business of carrying passengers for hire from the one point to another in the that the failure to fulfill his obligation was due to causes mentioned in article 1105
Province of La Union and the surrounding provinces. On the date mentioned, he of the Civil Code, which reads as follows:
undertook to convey the plaintiffs from San Fernando to Currimao, Ilocos Norte, in a
Ford automobile. On leaving San Fernando, the automobile was operated by a
No one shall be liable for events which could not be foreseen or which, even if
licensed chauffeur, but after having reached the town of San Juan, the chauffeur
foreseen, were inevitable, with the exception of the cases in which the law expressly
allowed his assistant, Remigio Bueno, to drive the car. Bueno held no driver's
provides otherwise and those in which the obligation itself imposes such liability.
license, but had some experience in driving, and with the exception of some slight
engine trouble while passing through the town of Luna, the car functioned well until
after the crossing of the Abra River in Tagudin, when, according to the testimony of This brings us to the principal question in the case:
the witnesses for the plaintiffs, defects developed in the steering gear so as to make
accurate steering impossible, and after zigzagging for a distance of about half a
What is meant by "events which cannot be foreseen and which, having been
kilometer, the car left the road and went down a steep embankment.
foreseen, are inevitable?" The Spanish authorities regard the language employed as
an effort to define the term caso fortuito and hold that the two expressions are
The defendant, in his testimony, maintains that there was no defect in the steering synonymous. (Manresa, Comentarios al Codigo Civil Español, vol. 8, pp. 88 et seq.;
gear, neither before nor after the accident, and expresses the opinion that the Scævola, Codigo Civil, vol. 19, pp. 526 et seq.)
swaying or zigzagging of the car must have been due to its having been driven at
an excessive rate of speed. This may possibly be true, but it is, from our point of
The antecedent to article 1105 is found in Law 11, Title 33, Partida 7, which
view, immaterial whether the accident was caused by negligence on the part of the
defines caso fortuito as "occasion que a case por aventura de que non se puede
defendant's employees, or whether it was due to defects in the automobile; the
ante ver. E son estos, derrivamientos de casas e fuego que se enciende a so ora, e
result would be practically the same in either event.
quebrantamiento de navio, fuerca de ladrones. . . . (An event that takes place by
accident and could not have been foreseen. Examples of this are destruction of
In going over the bank of the road, the automobile was overturned and the plaintiffs houses, unexpected fire, shipwreck, violence of robbers. . . .)"
pinned down under it. Mr. Lasam escaped with a few contusions and a "dislocated"
rib , but his wife, Joaquina Sanchez, received serious injuries, among which was a
Escriche defines caso fortuito as "an unexpected event or act of God which could
compound fracture of one of the bones in her left wrist. She also appears to have
either be foreseen nor resisted, such as floods, torrents, shipwrecks, conflagrations,
suffered a nervous breakdown from which she had not fully recovered at the time of
lightning, compulsion, insurrections, destructions, destruction of buildings by
the trial.
unforseen accidents and other occurrences of a similar nature."

The complaint in the case was filed about a year and a half after the occurrence
In discussing and analyzing the term caso fortuito the Enciclopedia Juridica
above related. It alleges, among other things, that the accident was due to defects
Española says: "In a legal sense and, consequently, also in relation to contracts, a
in the automobile as well as to the incompetence and negligence of the chauffeur,
caso fortuito presents the following essential characteristics: (1) The cause of the
and the case appears to have been tried largely upon the theory that it sounds in
unforeseen and unexpected occurrence, or of the failure of the debtor to comply
tort and that the liability of the defendant is governed by article 1903 of the Civil
with his obligation, must be independent of the human will. (2) It must be
Code. The trial court held, however, that the cause of action rests on the
31
impossible to foresee the event which constitutes the caso fortuito, or if it can be medical treatment for several years. We agree with the court below that the
foreseen, it must be impossible to avoid. (3) The occurrence must be such as to defendant should not be charged with these expenses.
render it impossible for the debtor to fulfill his obligation in a normal manner. And
(4) the obligor (debtor) must be free from any participation in the aggravation of
For the reasons stated, the judgment appealed from is affirmed, without costs in
the injury resulting to the creditor." (5 Enciclopedia Juridica Española, 309.)
this instance. So ordered.

As will be seen, these authorities agree that some extraordinary circumstance


G.R. No. 177279               October 13, 2010
independent of the will of the obligor, or of his employees, is an essential element of
COMMISSIONER OF INTERNAL REVENUE, Petitioner,
a caso fortuito. Turning to the present case, it is at once apparent that this element
vs. HON. RAUL M. GONZALEZ, Secretary of Justice, L. M. CAMUS
is lacking. It is not suggested that the accident in question was due to an act of God
ENGINEERING CORPORATION (represented by LUIS M. CAMUS and LINO D.
or to adverse road conditions which could not have been foreseen. As far as the
MENDOZA), Respondents.
records shows, the accident was caused either by defects in the automobile or else
through the negligence of its driver. That is not a caso fortuito.
This is a petition for review on certiorari under Rule 45 of the 1997 Rules of Civil
Procedure, as amended, assailing the Decision1 dated October 31, 2006 and
We agree with counsel that neither under the American nor Spanish law is a carrier
Resolution2 dated March 6, 2007 of the Court of Appeals (CA) in CA-G.R. SP No.
of passengers an absolute insurer against the risks of travel from which the
93387 which affirmed the Resolution3 dated December 13, 2005 of respondent
passenger may protect himself by exercising ordinary care and diligence. The case
Secretary of Justice in I.S. No. 2003-774 for violation of Sections 254 and 255 of
of Alba vs. Sociedad Anonima de Tranvias, Jurisprudencia Civil, vol. 102, p. 928,
the National Internal Revenue Code of 1997 (NIRC).
cited by the defendant in support of his contentions, affords a good illustration of
the application of this principle. In that case Alba, a passenger on a street car, was
standing on the platform of the car while it was in motion. The car rounded a curve The facts as culled from the records:
causing Alba to lose his balance and fall off the platform, sustaining severe injuries.
In an action brought by him to recover damages, the supreme court of Spain held Pursuant to Letter of Authority (LA) No. 00009361 dated August 25, 2000 issued by
that inasmuch as the car at the time of the accident was travelling at a moderate then Commissioner of Internal Revenue (petitioner) Dakila B. Fonacier, Revenue
rate of speed and there was no infraction of the regulations, and the plaintiff was Officers Remedios C. Advincula, Jr., Simplicio V. Cabantac, Jr., Ricardo L. Suba, Jr.
exposed to no greater danger than that inherent in that particular mode of travel, and Aurelio Agustin T. Zamora supervised by Section Chief Sixto C. Dy, Jr. of the
the plaintiff could not recover, especially so since he should have been on his guard Tax Fraud Division (TFD), National Office, conducted a fraud investigation for all
against a contingency as natural as that of losing his balance to a greater or less internal revenue taxes to ascertain/determine the tax liabilities of respondent L. M.
extent when the car rounded the curve. Camus Engineering Corporation (LMCEC) for the taxable years 1997, 1998 and
1999.4 The audit and investigation against LMCEC was precipitated by the
But such is not the present case; here the passengers had no means of avoiding the information provided by an "informer" that LMCEC had substantial underdeclared
danger or escaping the injury. income for the said period. For failure to comply with the subpoena duces tecum
issued in connection with the tax fraud investigation, a criminal complaint was
instituted by the Bureau of Internal Revenue (BIR) against LMCEC on January 19,
The plaintiffs maintain that the evidence clearly establishes that they are entitled to
2001 for violation of Section 266 of the NIRC (I.S. No. 00-956 of the Office of the
damages in the sum of P7,832.80 instead of P1,254.10 as found by the trial court,
City Prosecutor of Quezon City).5
and their assignments of error relate to this point only.

Based on data obtained from an "informer" and various clients of LMCEC, 6 it was
There can be no doubt that the expenses incurred by the plaintiffs as a result of the
discovered that LMCEC filed fraudulent tax returns with substantial
accident greatly exceeded the amount of the damages awarded. But bearing in
underdeclarations of taxable income for the years 1997, 1998 and 1999. Petitioner
mind that in determining the extent of the liability for losses or damages resulting
thus assessed the company of total deficiency taxes amounting to ₱430,958,005.90
from negligence in the fulfillment of a contractual obligation, the courts have "a
(income tax - ₱318,606,380.19 and value-added tax [VAT] - ₱112,351,625.71)
discretionary power to moderate the liability according to the circumstances" (De
covering the said period. The Preliminary Assessment Notice (PAN) was received by
Guia vs. Manila Electric Railroad & Light Co., 40 Phil., 706; art. 1103, Civil Code),
LMCEC on February 22, 2001.7
we do not think that the evidence is such as to justify us in interfering with the
discretion of the court below in this respect. As pointed out by that court in its well-
reasoned and well-considered decision, by far the greater part of the damages LMCEC’s alleged underdeclared income was summarized by petitioner as follows:
claimed by the plaintiffs resulted from the fracture of a bone in the left wrist of
Joaquina Sanchez and from her objections to having a decaying splinter of the bone
removed by a surgical operation. As a consequence of her refusal to submit such an Year Income Income Undeclared Percentage of
operation, a series of infections ensued and which required constant and expensive Per ITR Per Income Underdeclaratio

32
Investigation n YEAR   AMOUNT OF
TAXES
1997 96,638,540.0 283,412,140.8 186,733,600.8 193.30% PAID
0 4 4
1997 Termination Letter Under Letter of Authority EWT - P 6,000.00
1998 86,793,913.0 236,863,236.8 150,069,323.8 172.90% No. 174600 Dated November 4, 1998 VAT - 540,605.02
0 1 1 IT - 3,000.00

1999 88,287,792.0 251,507,903.1 163,220,111.1 184.90%8 1998 ERAP Program pursuant WC - 38,404.55
0 3 3 to RR #2-99 VAT - 61,635.40

1999 VAP Program pursuant IT - 878,495.28


In view of the above findings, assessment notices together with a formal letter of to RR #8-2001 VAT -
demand dated August 7, 2002 were sent to LMCEC through personal service on 1,324,317.0016
October 1, 2002.9 Since the company and its representatives refused to receive the
said notices and demand letter, the revenue officers resorted to constructive
service10 in accordance with Section 3, Revenue Regulations (RR) No. 12-99 11. LMCEC argued that petitioner is now estopped from further taking any action
against it and its corporate officers concerning the taxable years 1997 to 1999. With
the grant of immunity from audit from the company’s availment of ERAP and VAP,
On May 21, 2003, petitioner, through then Commissioner Guillermo L. Parayno, Jr.,
which have a feature of a tax amnesty, the element of fraud is negated the moment
referred to the Secretary of Justice for preliminary investigation its complaint
the Bureau accepts the offer of compromise or payment of taxes by the taxpayer.
against LMCEC, Luis M. Camus and Lino D. Mendoza, the latter two were sued in
The act of the revenue officers in finding justification under Section 6(B) of the NIRC
their capacities as President and Comptroller, respectively. The case was docketed
(Best Evidence Obtainable) is misplaced and unavailing because they were not able
as I.S. No. 2003-774. In the Joint Affidavit executed by the revenue officers who
to open the books of the company for the second time, after the routine
conducted the tax fraud investigation, it was alleged that despite the receipt of the
examination, issuance of termination letter and the availment of ERAP and VAP.
final assessment notice and formal demand letter on October 1, 2002, LMCEC failed
LMCEC thus maintained that unless there is a prior determination of fraud supported
and refused to pay the deficiency tax assessment in the total amount of
by documents not yet incorporated in the docket of the case, petitioner cannot just
₱630,164,631.61, inclusive of increments, which had become final and executory as
issue LAs without first terminating those previously issued. It emphasized the fact
a result of the said taxpayer’s failure to file a protest thereon within the thirty (30)-
that the BIR officers who filed and signed the Affidavit-Complaint in this case were
day reglementary period.12
the same ones who appeared as complainants in an earlier case filed against Camus
for his alleged "failure to obey summons in violation of Section 5 punishable under
Camus and Mendoza filed a Joint Counter-Affidavit contending that LMCEC cannot Section 266 of the NIRC of 1997" (I.S. No. 00-956 of the Office of the City
be held liable whatsoever for the alleged tax deficiency which had become due and Prosecutor of Quezon City). After preliminary investigation, said case was dismissed
demandable. Considering that the complaint and its annexes all showed that the for lack of probable cause in a Resolution issued by the Investigating Prosecutor on
suit is a simple civil action for collection and not a tax evasion case, the Department May 2, 2001.17
of Justice (DOJ) is not the proper forum for BIR’s complaint. They also assail as
invalid the assessment notices which bear no serial numbers and should be shown
LMCEC further asserted that it filed on April 20, 2001 a protest on the PAN issued by
to have been validly served by an Affidavit of Constructive Service executed and
petitioner for having no basis in fact and law. However, until now the said protest
sworn to by the revenue officers who served the same. As stated in LMCEC’s letter-
remains unresolved. As to the alleged informant who purportedly supplied the
protest dated December 12, 2002 addressed to Revenue District Officer (RDO)
"confidential information," LMCEC believes that such person is fictitious and his true
Clavelina S. Nacar of RD No. 40, Cubao, Quezon City, the company had already
identity and personality could not be produced. Hence, this case is another form of
undergone a series of routine examinations for the years 1997, 1998 and 1999;
harassment against the company as what had been found by the Office of the City
under the NIRC, only one examination of the books of accounts is allowed per
Prosecutor of Quezon City in I.S. No. 00-956. Said case and the present case both
taxable year.13
have something to do with the audit/examination of LMCEC for taxable years 1997,
1998 and 1999 pursuant to LA No. 00009361.18
LMCEC further averred that it had availed of the Bureau’s Tax Amnesty Programs
(Economic Recovery Assistance Payment [ERAP] Program and the Voluntary
In the Joint Reply-Affidavit executed by the Bureau’s revenue officers, petitioner
Assessment Program [VAP]) for 1998 and 1999; for 1997, its tax liability was
disagreed with the contention of LMCEC that the complaint filed is not criminal in
terminated and closed under Letter of Termination 14 dated June 1, 1999 issued by
nature, pointing out that LMCEC and its officers Camus and Mendoza were being
petitioner and signed by the Chief of the Assessment Division. 15 LMCEC claimed it
charged for the criminal offenses defined and penalized under Sections 254
made payments of income tax, VAT and expanded withholding tax (EWT), as
(Attempt to Evade or Defeat Tax) and 255 (Willful Failure to Pay Tax) of the NIRC.
follows:
33
This finds support in Section 205 of the same Code which provides for failure of LMCEC to submit or present its books of accounts and other accounting
administrative (distraint, levy, fine, forfeiture, lien, etc.) and judicial (criminal or records for examination despite the issuance of subpoena duces tecum against
civil action) remedies in order to enforce collection of taxes. Both remedies may be Camus in his capacity as President of LMCEC. While indeed a Resolution was issued
pursued either independently or simultaneously. In this case, the BIR decided to by Asst. City Prosecutor Titus C. Borlas on May 2, 2001 dismissing the complaint,
simultaneously pursue both remedies and thus aside from this criminal action, the the same is still on appeal and pending resolution by the DOJ. The determination of
Bureau also initiated administrative proceedings against LMCEC. 19 probable cause in said case is confined to the issue of whether there was already a
violation of the NIRC by Camus in not complying with the subpoena duces tecum
issued by the BIR.24
On the lack of control number in the assessment notice, petitioner explained that
such is a mere office requirement in the Assessment Service for the purpose of
internal control and monitoring; hence, the unnumbered assessment notices should Petitioner contended that precisely the reason for the issuance to the TFD of LA No.
not be interpreted as irregular or anomalous. Petitioner stressed that LMCEC already 00009361 by the Commissioner is because the latter agreed with the findings of the
lost its right to file a protest letter after the lapse of the thirty (30)-day investigating revenue officers that fraud exists in this case. In the conduct of their
reglementary period. LMCEC’s protest-letter dated December 12, 2002 to RDO investigation, the revenue officers observed the proper procedure under Revenue
Clavelina S. Nacar, RD No. 40, Cubao, Quezon City was actually filed only on Memorandum Order (RMO) No. 49-2000 wherein it is required that before the
December 16, 2002, which was disregarded by the petitioner for being filed out of issuance of a Letter of Authority against a particular taxpayer, a preliminary
time. Even assuming for the sake of argument that the assessment notices were investigation should first be conducted to determine if a prima facie case for tax
invalid, petitioner contended that such could not affect the present criminal fraud exists. As to the allegedly unresolved protest filed on April 20, 2001 by LMCEC
action,20 citing the ruling in the landmark case of Ungab v. Cusi, Jr. 21 over the PAN, this has been disregarded by the Bureau for being pro forma and
having been filed beyond the 15-day reglementary period. A subsequent letter
dated April 20, 2001 was filed with the TFD and signed by a certain Juan Ventigan.
As to the Letter of Termination signed by Ruth Vivian G. Gandia of the Assessment
However, this was disregarded and considered a mere scrap of paper since the said
Division, Revenue Region No. 7, Quezon City, petitioner pointed out that LMCEC
signatory had not shown any prior authorization to represent LMCEC. Even
failed to mention that the undated Certification issued by RDO Pablo C. Cabreros, Jr.
assuming said protest letter was validly filed on behalf of the company, the issuance
of RD No. 40, Cubao, Quezon City stated that the report of the 1997 Internal
of a Formal Demand Letter and Assessment Notice through constructive service on
Revenue taxes of LMCEC had already been submitted for review and approval of
October 1, 2002 is deemed an implied denial of the said protest. Lastly, the details
higher authorities. LMCEC also cannot claim as excuse from the reopening of its
regarding the "informer" being confidential, such information is entitled to some
books of accounts the previous investigations and examinations. Under Section 235
degree of protection, including the identity of the informant against LMCEC. 25
(a), an exception was provided in the rule on once a year audit examination in case
of "fraud, irregularity or mistakes, as determined by the Commissioner". Petitioner
explained that the distinction between a Regular Audit Examination and Tax Fraud In their Joint Rejoinder-Affidavit,26 Camus and Mendoza reiterated their argument
Audit Examination lies in the fact that the former is conducted by the district offices that the identity of the alleged informant is crucial to determine if he/she is qualified
of the Bureau’s Regional Offices, the authority emanating from the Regional under Section 282 of the NIRC. Moreover, there was no assessment that has
Director, while the latter is conducted by the TFD of the National Office only when already become final, the validity of its issuance and service has been put in issue
instances of fraud had been determined by the petitioner.22 being anomalous, irregular and oppressive. It is contended that for criminal
prosecution to proceed before assessment, there must be a prima facie showing of a
willful attempt to evade taxes. As to LMCEC’s availment of the VAP and ERAP
Petitioner further asserted that LMCEC’s claim that it was granted immunity from
programs, the certificate of immunity from audit issued to it by the BIR is plain and
audit when it availed of the VAP and ERAP programs is misleading. LMCEC failed to
simple, but petitioner is now saying it has the right to renege with impunity from its
state that its availment of ERAP under RR No. 2-99 is not a grant of absolute
undertaking. Though petitioner deems LMCEC not qualified to avail of the benefits of
immunity from audit and investigation, aside from the fact that said program was
VAP, it must be noted that if it is true that at the time the petitioner filed I.S. No.
only for income tax and did not cover VAT and withholding tax for the taxable year
00-956 sometime in January 2001 it had already in its custody that "Confidential
1998. As for LMCEC’S availment of VAP in 1999 under RR No. 8-2001 dated August
Information No. 29-2000 dated July 7, 2000", these revenue officers could have
1, 2001 as amended by RR No. 10-2001 dated September 3, 2001, the company
rightly filed the instant case and would not resort to filing said criminal complaint for
failed to state that it covers only income tax and VAT, and did not include
refusal to comply with a subpoena duces tecum.
withholding tax. However, LMCEC is not actually entitled to the benefits of VAP
under Section 1 (1.1 and 1.2) of RR No. 10-2001. As to the principle of estoppel
invoked by LMCEC, estoppel clearly does not lie against the BIR as this involved the On September 22, 2003, the Chief State Prosecutor issued a Resolution 27 finding no
exercise of an inherent power by the government to collect taxes. 23 sufficient evidence to establish probable cause against respondents LMCEC, Camus
and Mendoza. It was held that since the payments were made by LMCEC under
ERAP and VAP pursuant to the provisions of RR Nos. 2-99 and 8-2001 which were
Petitioner also pointed out that LMCEC’s assertion correlating this case with I.S. No.
offered to taxpayers by the BIR itself, the latter is now in estoppel to insist on the
00-956 is misleading because said case involves another violation and offense
criminal prosecution of the respondent taxpayer. The voluntary payments made
(Sections 5 and 266 of the NIRC). Said case was filed by petitioner due to the
thereunder are in the nature of a tax amnesty. The unnumbered assessment notices
34
were found highly irregular and thus their validity is suspect; if the amounts Anent the earlier case filed against the same taxpayer (I.S. No. 00-956), the
indicated therein were collected, it is uncertain how these will be accounted for and Secretary of Justice found petitioner to have engaged in forum shopping in view of
if it would go to the coffers of the government or elsewhere. On the required prior the fact that while there is still pending an appeal from the Resolution of the City
determination of fraud, the Chief State Prosecutor declared that the Office of the Prosecutor of Quezon City in said case, petitioner hurriedly filed the instant case,
City Prosecutor in I.S. No. 00-956 has already squarely ruled that (1) there was no which not only involved the same parties but also similar substantial issues (the
prior determination of fraud, (2) there was indiscriminate issuance of LAs, and (3) joint complaint-affidavit also alleged the issuance of LA No. 00009361 dated August
the complaint was more of harassment. In view of such findings, any ensuing LA is 25, 2000). Clearly, the evidence of litis pendentia is present. Finally, respondent
thus defective and allowing the collection on the assailed assessment notices would Secretary noted that if indeed LMCEC committed fraud in the settlement of its tax
already be in the context of a "fishing expedition" or "witch-hunting." Consequently, liabilities, then at the outset, it should have been discovered by the agents of
there is nothing to speak of regarding the finality of assessment notices in the petitioner, and consequently petitioner should not have issued the Letter of
aggregate amount of ₱630,164,631.61. Termination and the Certificate of Immunity From Audit. Petitioner thus should have
been more circumspect in the issuance of said documents. 32
Petitioner filed a motion for reconsideration which was denied by the Chief State
Prosecutor.28 Its motion for reconsideration having been denied, petitioner challenged the ruling
of respondent Secretary via a certiorari petition in the CA.
Petitioner appealed to respondent Secretary of Justice but the latter denied its
petition for review under Resolution dated December 13, 2005. 29 On October 31, 2006, the CA rendered the assailed decision 33 denying the petition
and concurred with the findings and conclusions of respondent Secretary.
Petitioner’s motion for reconsideration was likewise denied by the appellate
The Secretary of Justice found that petitioner’s claim that there is yet no finality as
court.34 It appears that entry of judgment was issued by the CA stating that its
to LMCEC’s payment of its 1997 taxes since the audit report was still pending review
October 31, 2006 Decision attained finality on March 25, 2007.35 However, the said
by higher authorities, is unsubstantiated and misplaced. It was noted that the
entry of judgment was set aside upon manifestation by the petitioner that it has
Termination Letter issued by the Commissioner on June 1, 1999 is explicit that the
filed a petition for review before this Court subsequent to its receipt of the
matter is considered closed. As for taxable year 1998, respondent Secretary stated
Resolution dated March 6, 2007 denying petitioner’s motion for reconsideration on
that the record shows that LMCEC paid VAT and withholding tax in the amount of
March 20, 2007.36
₱61,635.40 and ₱38,404.55, respectively. This eventually gave rise to the issuance
of a certificate of immunity from audit for 1998 by the Office of the Commissioner of
Internal Revenue. For taxable year 1999, respondent Secretary found that pursuant The petition is anchored on the following grounds:
to earlier LA No. 38633 dated July 4, 2000, LMCEC’s 1999 tax liabilities were still
pending investigation for which reason LMCEC assailed the subsequent issuance of
I. The Honorable Court of Appeals erroneously sustained the findings of the
LA No. 00009361 dated August 25, 2000 calling for a similar investigation of its
Secretary of Justice who gravely abused his discretion by dismissing the complaint
alleged 1999 tax deficiencies when no final determination has yet been arrived on
based on grounds which are not even elements of the offenses charged.
the earlier LA No. 38633.30

II. The Honorable Court of Appeals erroneously sustained the findings of the
On the allegation of fraud, respondent Secretary ruled that petitioner failed to
Secretary of Justice who gravely abused his discretion by dismissing petitioner’s
establish the existence of the following circumstances indicating fraud in the
evidence, contrary to law.
settlement of LMCEC’s tax liabilities: (1) there must be intentional and substantial
understatement of tax liability by the taxpayer; (2) there must be intentional and
substantial overstatement of deductions or exemptions; and (3) recurrence of the III. The Honorable Court of Appeals erroneously sustained the findings of the
foregoing circumstances. First, petitioner miserably failed to explain why the Secretary of Justice who gravely abused his discretion by inquiring into the validity
assessment notices were unnumbered; second, the claim that the tax fraud of a Final Assessment Notice which has become final, executory and demandable
investigation was precipitated by an alleged "informant" has not been corroborated pursuant to Section 228 of the Tax Code of 1997 for failure of private respondent to
nor was it clearly established, hence there is no other conclusion but that the file a protest against the same.37
Bureau engaged in a "fishing expedition"; and furthermore, petitioner’s course of
action is contrary to Section 235 of the NIRC allowing only once in a given taxable The core issue to be resolved is whether LMCEC and its corporate officers may be
year such examination and inspection of the taxpayer’s books of accounts and other prosecuted for violation of Sections 254 (Attempt to Evade or Defeat Tax) and 255
accounting records. There was no convincing proof presented by petitioner to show (Willful Failure to Supply Correct and Accurate Information and Pay Tax).
that the case of LMCEC falls under the exceptions provided in Section 235.
Respondent Secretary duly considered the issuance of Certificate of Immunity from
Audit and Letter of Termination dated June 1, 1999 issued to LMCEC. 31 Petitioner filed the criminal complaint against the private respondents for violation
of the following provisions of the NIRC, as amended:

35
SEC. 254. Attempt to Evade or Defeat Tax. – Any person who willfully attempts in There is no dispute that prior to the filing of the complaint with the DOJ, the report
any manner to evade or defeat any tax imposed under this Code or the payment on the tax fraud investigation conducted on LMCEC disclosed that it made
thereof shall, in addition to other penalties provided by law, upon conviction substantial underdeclarations in its income tax returns for 1997, 1998 and 1999.
thereof, be punished by a fine of not less than Thirty thousand pesos (P30,000) but Pursuant to RR No. 12-99,38 a PAN was sent to and received by LMCEC on February
not more than One hundred thousand pesos (P100,000) and suffer imprisonment of 22, 2001 wherein it was notified of the proposed assessment of deficiency taxes
not less than two (2) years but not more than four (4) years: Provided, That the amounting to ₱430,958,005.90 (income tax - ₱318,606,380.19 and VAT -
conviction or acquittal obtained under this Section shall not be a bar to the filing of ₱112,351,625.71) covering taxable years 1997, 1998 and 1999.39 In response to
a civil suit for the collection of taxes. said PAN, LMCEC sent a letter-protest to the TFD, which denied the same on April
12, 2001 for lack of legal and factual basis and also for having been filed beyond the
15-day reglementary period.40
SEC. 255. Failure to File Return, Supply Correct and Accurate Information, Pay Tax,
Withhold and Remit Tax and Refund Excess Taxes Withheld on Compensation. – Any
person required under this Code or by rules and regulations promulgated As mentioned in the PAN, the revenue officers were not given the opportunity to
thereunder to pay any tax, make a return, keep any record, or supply any correct examine LMCEC’s books of accounts and other accounting records because its
and accurate information, who willfully fails to pay such tax, make such return, keep officers failed to comply with the subpoena duces tecum earlier issued, to verify its
such record, or supply such correct and accurate information, or withhold or remit alleged underdeclarations of income reported by the Bureau’s informant under
taxes withheld, or refund excess taxes withheld on compensations at the time or Section 282 of the NIRC. Hence, a criminal complaint was filed by the Bureau
times required by law or rules and regulations shall, in addition to other penalties against private respondents for violation of Section 266 which provides:
provided by law, upon conviction thereof, be punished by a fine of not less than Ten
thousand pesos (P10,000) and suffer imprisonment of not less than one (1) year
SEC. 266. Failure to Obey Summons. – Any person who, being duly summoned to
but not more than ten (10) years.
appear to testify, or to appear and produce books of accounts, records,
memoranda, or other papers, or to furnish information as required under the
Respondent Secretary concurred with the Chief State Prosecutor’s conclusion that pertinent provisions of this Code, neglects to appear or to produce such books of
there is insufficient evidence to establish probable cause to charge private accounts, records, memoranda, or other papers, or to furnish such information,
respondents under the above provisions, based on the following findings: (1) the shall, upon conviction, be punished by a fine of not less than Five thousand pesos
tax deficiencies of LMCEC for taxable years 1997, 1998 and 1999 have all been (P5,000) but not more than Ten thousand pesos (P10,000) and suffer imprisonment
settled or terminated, as in fact LMCEC was issued a Certificate of Immunity and of not less than one (1) year but not more than two (2) years.
Letter of Termination, and availed of the ERAP and VAP programs; (2) there was no
prior determination of the existence of fraud; (3) the assessment notices are
It is clear that I.S. No. 00-956 involves a separate offense and hence litis pendentia
unnumbered, hence irregular and suspect; (4) the books of accounts and other
is not present considering that the outcome of I.S. No. 00-956 is not determinative
accounting records may be subject to audit examination only once in a given
of the issue as to whether probable cause exists to charge the private respondents
taxable year and there is no proof that the case falls under the exceptions provided
with the crimes of attempt to evade or defeat tax and willful failure to supply correct
in Section 235 of the NIRC; and (5) petitioner committed forum shopping when it
and accurate information and pay tax defined and penalized under Sections 254 and
filed the instant case even as the earlier criminal complaint (I.S. No. 00-956)
255, respectively. For the crime of tax evasion in particular, compliance by the
dismissed by the City Prosecutor of Quezon City was still pending appeal.
taxpayer with such subpoena, if any had been issued, is irrelevant. As we held in
Ungab v. Cusi, Jr.,41 "[t]he crime is complete when the [taxpayer] has x x x
Petitioner argues that with the finality of the assessment due to failure of the knowingly and willfully filed [a] fraudulent [return] with intent to evade and defeat x
private respondents to challenge the same in accordance with Section 228 of the x x the tax." Thus, respondent Secretary erred in holding that petitioner committed
NIRC, respondent Secretary has no jurisdiction and authority to inquire into its forum shopping when it filed the present criminal complaint during the pendency of
validity. Respondent taxpayer is thereby allowed to do indirectly what it cannot do its appeal from the City Prosecutor’s dismissal of I.S. No. 00-956 involving the act
directly – to raise a collateral attack on the assessment when even a direct of disobedience to the summons in the course of the preliminary investigation on
challenge of the same is legally barred. The rationale for dismissing the complaint LMCEC’s correct tax liabilities for taxable years 1997, 1998 and 1999.
on the ground of lack of control number in the assessment notice likewise betrays a
lack of awareness of tax laws and jurisprudence, such circumstance not being an
In the Details of Discrepancies attached as Annex B of the PAN, 42 private
element of the offense. Worse, the final, conclusive and undisputable evidence
respondents were already notified that inasmuch as the revenue officers were not
detailing a crime under our taxation laws is swept under the rug so easily on mere
given the opportunity to examine LMCEC’s books of accounts, accounting records
conspiracy theories imputed on persons who are not even the subject of the
and other documents, said revenue officers gathered information from third parties.
complaint.
Such procedure is authorized under Section 5 of the NIRC, which provides:

We grant the petition.


SEC. 5. Power of the Commissioner to Obtain Information, and to Summon,
Examine, and Take Testimony of Persons. – In ascertaining the correctness of any

36
return, or in making a return when none has been made, or in determining the We do not agree.
liability of any person for any internal revenue tax, or in collecting any such liability,
or in evaluating tax compliance, the Commissioner is authorized:
A notice of assessment is:

(A) To examine any book, paper, record or other data which may be relevant or
[A] declaration of deficiency taxes issued to a [t]axpayer who fails to respond to a
material to such inquiry;
Pre-Assessment Notice (PAN) within the prescribed period of time, or whose reply to
the PAN was found to be without merit. The Notice of Assessment shall inform the
(B) To obtain on a regular basis from any person other than the person whose [t]axpayer of this fact, and that the report of investigation submitted by the
internal revenue tax liability is subject to audit or investigation, or from any office or Revenue Officer conducting the audit shall be given due course.
officer of the national and local governments, government agencies and
instrumentalities, including the Bangko Sentral ng Pilipinas and government-owned
The formal letter of demand calling for payment of the taxpayer’s deficiency tax or
or -controlled corporations, any information such as, but not limited to, costs and
taxes shall state the fact, the law, rules and regulations or jurisprudence on
volume of production, receipts or sales and gross incomes of taxpayers, and the
which the assessment is based, otherwise the formal letter of demand and
names, addresses, and financial statements of corporations, mutual fund
the notice of assessment shall be void.44
companies, insurance companies, regional operating headquarters of multinational
companies, joint accounts, associations, joint ventures or consortia and registered
partnerships, and their members; As it is, the formality of a control number in the assessment notice is not a
requirement for its validity but rather the contents thereof which should inform the
taxpayer of the declaration of deficiency tax against said taxpayer. Both the formal
(C) To summon the person liable for tax or required to file a return, or any officer or
letter of demand and the notice of assessment shall be void if the former failed to
employee of such person, or any person having possession, custody, or care of the
state the fact, the law, rules and regulations or jurisprudence on which the
books of accounts and other accounting records containing entries relating to the
assessment is based, which is a mandatory requirement under Section 228 of the
business of the person liable for tax, or any other person, to appear before the
NIRC.
Commissioner or his duly authorized representative at a time and place specified in
the summons and to produce such books, papers, records, or other data, and to
give testimony; Section 228 of the NIRC provides that the taxpayer shall be informed in writing of
the law and the facts on which the assessment is made. Otherwise, the assessment
is void. To implement the provisions of Section 228 of the NIRC, RR No. 12-99 was
(D) To take such testimony of the person concerned, under oath, as may be
enacted. Section 3.1.4 of the revenue regulation reads:
relevant or material to such inquiry; x x x

3.1.4. Formal Letter of Demand and Assessment Notice. – The formal letter of


Private respondents’ assertions regarding the qualifications of the "informer" of the
demand and assessment notice shall be issued by the Commissioner or his duly
Bureau deserve scant consideration. We have held that the lack of consent of the
authorized representative. The letter of demand calling for payment of the
taxpayer under investigation does not imply that the BIR obtained the information
taxpayer’s deficiency tax or taxes shall state the facts, the law, rules and
from third parties illegally or that the information received is false or malicious. Nor
regulations, or jurisprudence on which the assessment is based, otherwise,
does the lack of consent preclude the BIR from assessing deficiency taxes on the
the formal letter of demand and assessment notice shall be void. The same
taxpayer based on the documents.43 In the same vein, herein private respondents
shall be sent to the taxpayer only by registered mail or by personal delivery. x x
cannot be allowed to escape criminal prosecution under Sections 254 and 255 of the
x.45 (Emphasis supplied.)
NIRC by mere imputation of a "fictitious" or disqualified informant under Section
282 simply because other than disclosure of the official registry number of the third
party "informer," the Bureau insisted on maintaining the confidentiality of the The Formal Letter of Demand dated August 7, 2002 contains not only a detailed
identity and personal circumstances of said "informer." computation of LMCEC’s tax deficiencies but also details of the specified
discrepancies, explaining the legal and factual bases of the assessment. It also
reiterated that in the absence of accounting records and other documents necessary
Subsequently, petitioner sent to LMCEC by constructive service allowed under
for the proper determination of the company’s internal revenue tax liabilities, the
Section 3 of RR No. 12-99, assessment notice and formal demand informing the
investigating revenue officers resorted to the "Best Evidence Obtainable" as
said taxpayer of the law and the facts on which the assessment is made, as
provided in Section 6(B) of the NIRC (third party information) and in accordance
required by Section 228 of the NIRC. Respondent Secretary, however, fully
with the procedure laid down in RMC No. 23-2000 dated November 27, 2000. Annex
concurred with private respondents’ contention that the assessment notices were
"A" of the Formal Letter of Demand thus stated:
invalid for being unnumbered and the tax liabilities therein stated have already been
settled and/or terminated.
Thus, to verify the validity of the information previously provided by the informant,
the assigned revenue officers resorted to third party information. Pursuant to
37
Section 5(B) of the NIRC of 1997, access letters requesting for information and the from audit and enforcement activities." The program named as "Economic Recovery
submission of certain documents (i.e., Certificate of Income Tax Withheld at Source Assistance Payment (ERAP) Program" granted immunity from audit and
and/or Alphabetical List showing the income payments made to L.M. Camus investigation of income tax, VAT and percentage tax returns for 1998. It expressly
Engineering Corporation for the taxable years 1997 to 1999) were sent to the excluded withholding tax returns (whether for income, VAT, or percentage tax
various clients of the subject corporation, including but not limited to the following: purposes). Since such immunity from audit and investigation does not preclude the
collection of revenues generated from audit and enforcement activities, it follows
that the Bureau is likewise not barred from collecting any tax deficiency discovered
1. Ayala Land Inc.
as a result of tax fraud investigations. Respondent Secretary’s opinion that RR No.
2. Filinvest Alabang Inc.
2-99 contains the feature of a tax amnesty is thus misplaced.
3. D.M. Consunji, Inc.
4. SM Prime Holdings, Inc.
5. Alabang Commercial Corporation Tax amnesty is a general pardon to taxpayers who want to start a clean tax slate. It
6. Philam Properties Corporation also gives the government a chance to collect uncollected tax from tax evaders
7. SM Investments, Inc. without having to go through the tedious process of a tax case. 51 Even assuming
8. Shoemart, Inc. arguendo that the issuance of RR No. 2-99 is in the nature of tax amnesty, it bears
9. Philippine Securities Corporation noting that a tax amnesty, much like a tax exemption, is never favored nor
10. Makati Development Corporation presumed in law and if granted by statute, the terms of the amnesty like that of a
tax exemption must be construed strictly against the taxpayer and liberally in favor
of the taxing authority.52
From the documents gathered and the data obtained therein, the substantial
underdeclaration as defined under Section 248(B) of the NIRC of 1997 by your
corporation of its income had been confirmed. x x x x46 (Emphasis supplied.) For the same reason, the availment by LMCEC of VAP under RR No. 8-2001 as
amended by RR No. 10-2001, through payment supposedly made in October 29,
2001 before the said program ended on October 31, 2001, did not amount to
In the same letter, Assistant Commissioner Percival T. Salazar informed private
settlement of its assessed tax deficiencies for the period 1997 to 1999, nor
respondents that the estimated tax liabilities arising from LMCEC’s underdeclaration
immunity from prosecution for filing fraudulent return and attempt to evade or
amounted to ₱186,773,600.84 in 1997, ₱150,069,323.81 in 1998 and
defeat tax. As correctly asserted by petitioner, from the express terms of the
₱163,220,111.13 in 1999. These figures confirmed that the non-declaration by
aforesaid revenue regulations, LMCEC is not qualified to avail of the VAP granting
LMCEC for the taxable years 1997, 1998 and 1999 of an amount exceeding 30%
taxpayers the privilege of last priority in the audit and investigation of all internal
income47 declared in its return is considered a substantial underdeclaration of
revenue taxes for the taxable year 2000 and all prior years under certain conditions,
income, which constituted prima facie evidence of false or fraudulent return under
considering that first, it was issued a PAN on February 19, 2001, and second, it was
Section 248(B)48 of the NIRC, as amended.49
the subject of investigation as a result of verified information filed by a Tax
Informer under Section 282 of the NIRC duly recorded in the BIR Official Registry as
On the alleged settlement of the assessed tax deficiencies by private respondents, Confidential Information (CI) No. 29-200053 even prior to the issuance of the PAN.
respondent Secretary found the latter’s claim as meritorious on the basis of the
Certificate of Immunity From Audit issued on December 6, 1999 pursuant to RR No.
Section 1 of RR No. 8-2001 provides:
2-99 and Letter of Termination dated June 1, 1999 issued by Revenue Region No. 7
Chief of Assessment Division Ruth Vivian G. Gandia. Petitioner, however, clarified
that the certificate of immunity from audit covered only income tax for the year SECTION 1. COVERAGE. – x x x
1997 and does not include VAT and withholding taxes, while the Letter of
Termination involved tax liabilities for taxable year 1997 (EWT, VAT and income
Any person, natural or juridical, including estates and trusts, liable to pay any of the
taxes) but which was submitted for review of higher authorities as per the
above-cited internal revenue taxes for the above specified period/s who, due to
Certification of RD No. 40 District Officer Pablo C. Cabreros, Jr. 50 For 1999, private
inadvertence or otherwise, erroneously paid his internal revenue tax liabilities or
respondents supposedly availed of the VAP pursuant to RR No. 8-2001.
failed to file tax return/pay taxes may avail of the Voluntary Assessment Program
(VAP), except those falling under any of the following instances:
RR No. 2-99 issued on February 7, 1999 explained in its Policy Statement that
considering the scarcity of financial and human resources as well as the time
1.1 Those covered by a Preliminary Assessment Notice (PAN), Final Assessment
constraints within which the Bureau has to "clean the Bureau’s backlog of unaudited
Notice (FAN), or Collection Letter issued on or before July 31, 2001; or
tax returns in order to keep updated and be focused with the most current
accounts" in preparation for the full implementation of a computerized tax
administration, the said revenue regulation was issued "providing for last priority in 1.2 Persons under investigation as a result of verified information filed by a Tax
audit and investigation of tax returns" to accomplish the said objective "without, Informer under Section 282 of the Tax Code of 1997, duly processed and recorded
however, compromising the revenue collection that would have been generated in the BIR Official Registry Book on or before July 31, 2001;

38
1.3 Tax fraud cases already filed and pending in courts for adjudication; and for the TFD to conduct a formal fraud investigation of LMCEC.57 Consequently,
respondent Secretary’s ruling that the filing of criminal complaint for violation of
Sections 254 and 255 of the NIRC cannot prosper because of lack of prior
Moreover, private respondents cannot invoke LMCEC’s availment of VAP to foreclose
determination of the existence of fraud, is bereft of factual basis and contradicted
any subsequent audit of its account books and other accounting records in view of
by the evidence on record.
the strong finding of underdeclaration in LMCEC’s payment of correct income tax
liability by more than 30% as supported by the written report of the TFD detailing
the facts and the law on which such finding is based, pursuant to the tax fraud Tax assessments by tax examiners are presumed correct and made in good faith,
investigation authorized by petitioner under LA No. 00009361. This conclusion finds and all presumptions are in favor of the correctness of a tax assessment unless
support in Section 2 of RR No. 8-2001 as amended by RR No. 10-2001 provides: proven otherwise.58 We have held that a taxpayer’s failure to file a petition for
review with the Court of Tax Appeals within the statutory period rendered the
disputed assessment final, executory and demandable, thereby precluding it from
SEC. 2. TAXPAYER’S BENEFIT FROM AVAILMENT OF THE VAP. – A taxpayer
interposing the defenses of legality or validity of the assessment and prescription of
who has availed of the VAP shall not be audited except upon authorization and
the Government’s right to assess.59 Indeed, any objection against the assessment
approval of the Commissioner of Internal Revenue when there is strong evidence or
should have been pursued following the avenue paved in Section 229 (now Section
finding of understatement in the payment of taxpayer’s correct tax liability by more
228) of the NIRC on protests on assessments of internal revenue taxes. 60
than thirty percent (30%) as supported by a written report of the appropriate office
detailing the facts and the law on which such finding is based: Provided, however,
that any VAP payment should be allowed as tax credit against the deficiency tax Records bear out that the assessment notice and Formal Letter of Demand dated
due, if any, in case the concerned taxpayer has been subjected to tax audit. August 7, 2002 were duly served on LMCEC on October 1, 2002. Private
respondents did not file a motion for reconsideration of the said assessment notice
and formal demand; neither did they appeal to the Court of Tax Appeals. Section
Given the explicit conditions for the grant of immunity from audit under RR No. 2-
228 of the NIRC61 provides the remedy to dispute a tax assessment within a certain
99, RR No. 8-2001 and RR No. 10-2001, we hold that respondent Secretary gravely
period of time. It states that an assessment may be protested by filing a request for
erred in declaring that petitioner is now estopped from assessing any tax deficiency
reconsideration or reinvestigation within 30 days from receipt of the assessment by
against LMCEC after issuance of the aforementioned documents of immunity from
the taxpayer. No such administrative protest was filed by private respondents
audit/investigation and settlement of tax liabilities. It is axiomatic that the State can
seeking reconsideration of the August 7, 2002 assessment notice and formal letter
never be in estoppel, and this is particularly true in matters involving taxation. The
of demand. Private respondents cannot belatedly assail the said assessment, which
errors of certain administrative officers should never be allowed to jeopardize the
they allowed to lapse into finality, by raising issues as to its validity and correctness
government’s financial position.54
during the preliminary investigation after the BIR has referred the matter for
prosecution under Sections 254 and 255 of the NIRC.
Respondent Secretary’s other ground for assailing the course of action taken by
petitioner in proceeding with the audit and investigation of LMCEC -- the alleged
As we held in Marcos II v. Court of Appeals62:
violation of the general rule in Section 235 of the NIRC allowing the examination
and inspection of taxpayer’s books of accounts and other accounting records only
once in a taxable year -- is likewise untenable. As correctly pointed out by It is not the Department of Justice which is the government agency tasked to
petitioner, the discovery of substantial underdeclarations of income by LMCEC for determine the amount of taxes due upon the subject estate, but the Bureau of
taxable years 1997, 1998 and 1999 upon verified information provided by an Internal Revenue, whose determinations and assessments are presumed correct
"informer" under Section 282 of the NIRC, as well as the necessity of obtaining and made in good faith. The taxpayer has the duty of proving otherwise. In the
information from third parties to ascertain the correctness of the return filed or absence of proof of any irregularities in the performance of official duties, an
evaluation of tax compliance in collecting taxes (as a result of the disobedience to assessment will not be disturbed. Even an assessment based on estimates is prima
the summons issued by the Bureau against the private respondents), are facie valid and lawful where it does not appear to have been arrived at arbitrarily or
circumstances warranting exception from the general rule in Section 235. 55 capriciously. The burden of proof is upon the complaining party to show clearly that
the assessment is erroneous. Failure to present proof of error in the assessment will
justify the judicial affirmance of said assessment. x x x.
As already stated, the substantial underdeclared income in the returns filed by
LMCEC for 1997, 1998 and 1999 in amounts equivalent to more than 30% (the
computation in the final assessment notice showed underdeclarations of almost Moreover, these objections to the assessments should have been raised,
200%) constitutes prima facie evidence of fraudulent return under Section 248(B) considering the ample remedies afforded the taxpayer by the Tax Code, with the
of the NIRC. Prior to the issuance of the preliminary and final notices of assessment, Bureau of Internal Revenue and the Court of Tax Appeals, as described earlier, and
the revenue officers conducted a preliminary investigation on the information and cannot be raised now via Petition for Certiorari, under the pretext of grave abuse of
documents showing substantial understatement of LMCEC’s tax liabilities which were discretion. The course of action taken by the petitioner reflects his disregard or even
provided by the Informer, following the procedure under RMO No. 15-95. 56 Based on repugnance of the established institutions for governance in the scheme of a well-
the prima facie finding of the existence of fraud, petitioner issued LA No. 00009361 ordered society. The subject tax assessments having become final, executory and

39
enforceable, the same can no longer be contested by means of a disguised protest. P11,197.40. Having failed to declare for tax purposes the P0.20 dance fee payable
In the main, Certiorari may not be used as a substitute for a lost appeal or remedy. to the "bailarinas" which petitioner collected as part of his business, respondent
This judicial policy becomes more pronounced in view of the absence of sufficient assessed against him a deficiency amusement tax, including 50 per cent surcharge,
attack against the actuations of government. (Emphasis supplied.) in the amount of P17,616.05. Respondent also assessed against petitioner the
further sum of P300.00 as penalty in settlement of his violation of Section 260 of
the Tax Code and the Bookkeeping Regulations.
The determination of probable cause is part of the discretion granted to the
investigating prosecutor and ultimately, the Secretary of Justice. However, this
Court and the CA possess the power to review findings of prosecutors in preliminary From the above assessment, petitioner took the case on appeal to the Court of Tax
investigations. Although policy considerations call for the widest latitude of Appeals where, after due hearing, said court rendered decision affirming the
deference to the prosecutor’s findings, courts should never shirk from exercising contention of respondent insofar as he holds petitioner liable to pay the sum of
their power, when the circumstances warrant, to determine whether the P17,616.05 as deficiency amusement tax and surcharge for the period from
prosecutor’s findings are supported by the facts, or by the law. In so doing, courts January, 1947 to August, 1950. However, the Court of Tax Appeals rejected the
do not act as prosecutors but as organs of the judiciary, exercising their mandate imposition of the penalty in the sum of P300.00 alleging lack of power or authority
under the Constitution, relevant statutes, and remedial rules to settle cases and to order the payment of such penalty. In due time, petitioner filed the present
controversies.63 Clearly, the power of the Secretary of Justice to review does not petition for review.
preclude this Court and the CA from intervening and exercising our own powers of
review with respect to the DOJ’s findings, such as in the exceptional case in which
The law under which the deficieny amusement tax was collected from petitioner for
grave abuse of discretion is committed, as when a clear sufficiency or insufficiency
his alleged gross receipts from January, 1947 to August, 1950 is Section 260 of the
of evidence to support a finding of probable cause is ignored. 64
Tax Code, the pertinent portion of which reads:

WHEREFORE, the petition is GRANTED. The Decision dated October 31, 2006 and
In the case of cockpits, cabarets, and night clubs, there shall be collected
Resolution dated March 6, 2007 of the Court of Appeals in CA-G.R. SP No. 93387
from the proprietors, lessees, or operators a tax equivalent to ten per
are hereby REVERSED and SET ASIDE. The Secretary of Justice is hereby DIRECTED
centum . . . of the gross receipts, irrespective of whether or not any
to order the Chief State Prosecutor to file before the Regional Trial Court of Quezon
amount is charged or paid for admission. . . . For the purpose of
City, National Capital Judicial Region, the corresponding Information against L. M.
amusement tax, the term "gross receipts" embraces all the receipts of the
Camus Engineering Corporation, represented by its President Luis M. Camus and
proprietor, lessee, or operator of the amusement place.
Comptroller Lino D. Mendoza, for Violation of Sections 254 and 255 of the National
Internal Revenue Code of 1997.
It would appear that the owner or operator of a cabaret is required to pay an
amusement tax equivalent to 10 per cent of the gross receipts of his business
G.R. No. L-13387             March 28, 1960
"irrespective of whether or not any amount is charged or paid for admission. The
SY CHIUCO, petitioner,
law further adds that, for the purposes of amusement tax, the term "gross receipts"
vs. COLLECTOR OF INTERNAL REVENUE, respondent.
embraces all the receipts of the proprietor or operator of the business. The question
that now arises is: What should be considered as gross receipts of the La Loma
Petitioner was the owner and operator of the La Loma Cabaret located at La Loma, Cabaret operated by petitioner? Does this term include only what it collects from its
Quezon City from 1926 to January, 1956. The customers who patronized the customers as admission fee to the cabaret, or it should also include the dance fee
cabaret were charged P0.30 per dance, P0.10 to be paid before entering the dance that is charged by the cabaret as compensation for its "bailarinas"?
hall and the remaining P0.20 to be paid to the "bailarinas" after the dance. The
customers were informed of the fees to be paid per dance by means of posters
Petitioner contends that it should only include what he collects as admission fee,
found in conspicuous places of the cabaret stating:
and not those representing the dance fee because they do not go to the operator,
but to the "bailarinas". In other words, petitioner contends that because those
Gate P0.10 dance fees go to the "bailarinas", they could not be considered as part of the gross
receipts of the cabaret.
Ladies 20
A Dance Total P0.30 With this contention we disagree. A cabaret is a place of amusement where
customers go because of their desire to dance and where the "bailarinas" are the
During the period from January, 1947 to August, 1950, petitioner declared in his main attraction. Dancing is the main business and customers patronize the place
return only the following gross receipts: receipts from gate admissions at P0.10 attracted by the "bailarinas". As a matter of fact, "bailarinas" are the indispensable
each, P59,160.40; receipts from restaurant sales, P5,339.90; receipts from bar factor in the operation of the business. Whatever is paid to them should, therefore,
sales, P47,459.10, and paid thereon a 10 per cent amusement tax in the amount of be considered as paid on account of the business, and as such it should be
considered as part of petitioner's gross receipts.
40
That the foregoing is the correct interpretation of the term "gross receipts" can be WHEREFORE, the Court, finds the assessments for allegedly deficient income and
gleaned from the very terminology of the law where in referring to the gross sales taxes for petitioner's fiscal year ending September 30, 1981 covered by
receipts the operation of the cabaret may realize it includes mainly all receipts Demand Letter NO. FAS-1B-81-87 and Assessment Notices Nos. FAS-1-81-87-
"irrespective of whether or not any amount is charged or paid for admission." The 005824, FAS-4-81-87-005825 and FAS-4-81-87-005826 (all dated July 29, 1987) in
law undoubtedly mainly contemplates to include the fees that may be paid by the the total amount of P19,535,183.44 to be NULL AND VOID for having been issued
customer for the privilege of dancing for it considers as incidental what may be paid beyond the five-year prescriptive period provided by law. 3
by the customer as admission fee. In other words, the law in effect considers the
amount charged against the customers for dancing with the "bailarinas" as the main
The undisputed facts of the case as recited in the Decision (Annex "A") of the Court
gross receipts of the cabaret, the admission fee thereto being merely incidental. In
of Appeals, are:4
this respect, we are in full accord with the following pronouncement of the Court of
Tax Appeals:
On January 15, 1982, Carnation Phils. Inc. (Carnation), filed its Corporation Annual
Income Tax Return for taxable year ending September 30, 1981; and its
We hold that when an operator, proprietor or lessee of a cabaret takes it
Manufacturers/Producers Percentage Tax Return for the quarter ending September
upon himself to set a fixed dance fee and thereby tends to the collection of
30, 1981.5
the same for the benefit of his "bailarinas" or hostesses, the income
derived therefrom forms part of his gross receipts and therefore subject to
amusement tax. By such imposition, the operator becomes the principal On October 13, 1986, March 16, 1987 and May 18, 1987, Carnation, through its
party to the implied contract of lease of services with his customers in Senior Vice President Jaime O. Lardizabal, signed three separate "waivers of the
place of the "bailarinas" or hostesses under his employ and therefore Statute of Limitations Under the National Internal Revenue Code" wherein it:
subject to the resulting liabilities as such contracting party.
. . . waives the running of the prescriptive period provided for in sections 318 and
Petitioner, however, contends that the Court of Tax Appeals erred in charging 319 and other related provisions of the National Internal Revenue Code and
against him the surcharge of 50 per cent on the amount he allegedly under declared consents to the assessment and collection of the taxes which may be found due
for the reason that there is no evidence on record to show that he defrauded the after reinvestigation and reconsideration at anytime before or after the lapse of the
Government. While there is no direct evidence to show actual fraud on the part of period of limitations fixed by said sections 318 and 319 and other relevant
petitioner, however, the circumstances found by the Court of Tax Appeals indicate provisions of the National Internal Revenue Code, but not after (13 April 1987 for
that he has deliberately omitted in his book a sizeable portion of his taxable income the earlier-executed waiver, or June 14, 1987 for the later waiver, or July 30, 1987
which in substance amounts to fraud. In the circumstances, we are not prepared to for the subsequent waiver, as the case may be). However, the taxpayer (petitioner
disturb the finding of the Court of Tax Appeals on this matter even if there is no herein) does not waive any prescription already accrued in its favor.
direct evidence that fraud was committed.
The waivers were not signed by the BIR Commissioner or any of his agents.
As regards the contention that the collection of the tax in question has already
prescribed, it appears that this question was not raised as an issue in the petition On August 5, 1987, Carnation received BIR's letter of demand dated July 29, 1987
for review filed by petitioner in the Court of Tax Appeals. It was not even touched asking the said corporation to pay P1,442,586.56 as deficiency income tax,
by him in the memorandum he submitted. There is, therefore, enough reason to P14,152,683.85 as deficiency sales tax and P3,939,913.03 as deficiency sales tax
believe that petitioner has waived this defense and so it cannot now be entertained. on undeclared sales, all for the year 1981. This demand letter was accompanied by
To hold otherwise would be to deprive respondent of his right to show the contrary, assessment Notices Nos. FAS-4-81-87-005824, FAS-4-81-87-005825 and FAS-4-
this matter being evidentiary in nature. 81-87-005826.

Wherefore, the decision appealed from is affirmed, with costs against petitioner. In a basic protest dated August 17, 1987, Carnation disputed the assessments and
requested a reconsideration and reinvestigation thereof.
G.R. No. 115712 February 25, 1999
COMMISSIONER OF INTERNAL REVENUE, petitioner, On September 30, 1987, Carnation filed a supplemental protest.
vs. COURT OF APPEALS, COURT OF TAX APPEALS and CARNATION
PHILIPPINES, INC. (now merged with Nestle Phils, Inc.), respondent.
These protests were denied by the BIR Commissioner in a letter dated March 15,
1988.
Before the Court is an appeal from the decision of the Court of Appeals  dated May
1

31, 1994, which affirmed in toto the decision of the Court of Tax Appeals2 dated
January 26, 1993, the dispositive portion of which reads: Whereupon, Carnation appealed to the CTA.

41
On January 26, 1993, the CTA issued the questioned order, the dispositive portion Sec. 319. Exceptions as to period of limitation of assessment and collection of
of which reads: taxes. —(a) . . .

WHEREFORE, the Court finds the assessments for allegedly deficient income and (b) Where before the expiration of the time prescribed in the preceding section for
sales taxes for petitioner's fiscal year ending September 30, 1981 covered by the assessment of the tax, both the Commissioner of Internal Revenue and the
Demand Letter No. FAS-1B-81-87 and assessment Notices No. FAS-1-81-87- taxpayer have consented in writing to its assessment after such time, the tax may
005824, FAS-4-81-87-005825, and FAS-4-81-87-005826 (all dated July 29, 1987) be assessed at anytime prior to the expiration of the period agreed upon. The
in the total amount of P19,535,183.44 to be NULL AND VOID for having been period so agreed upon may be extended by subsequent agreement in writing made
issued beyond the five-year prescriptive period provided by law. before the expiration of the period previously agreed upon.

The pivot of inquiry here is whether or not the three (3) waivers signed by the The Court of Appeals itself also passed upon the validity of the waivers executed by
private respondent are valid and binding6 as to toll the running of the prescriptive Carnation, observing thus:
period for assessment and not bar the Government from issuing subject deficiency
tax assessments.
We cannot go along with the petitioner's theory. Section 319 of the Tax code earlier
quoted is clear and explicit that the waiver of the five-year prescriptive period must
Sec. 318 (now Section 203) of the National Internal Revenue Code, the law then be in writing and signed by both the BIR Commissioner and the taxpayer.
applicable reads:
Here, the three waivers signed by Carnation do not bear the written consent of the
Sec 318. Period of Limitations upon assessment and collection. — Except as BIR Commissioner as required by law.
provided in the succeeding section, internal revenue taxes shall be assessed
within five years after the return was filed, and no proceeding in court without
We agree with the CTA in holding "these "waivers" to be invalid and without any
assessment for the collection of such taxes shall be begun after the expiration of
binding effect on petitioner (Carnation) for the reason that there was no consent by
such period. For the purpose of this section, a return filed before the last day
the respondent (Commissioner of Internal Revenue)."
prescribed by law for the filing thereof shall be considered as filed on such last day:
Provided, That this limitation shall not apply to cases already investigated prior to
the approval of this Code. 7 (emphasis ours) The ruling of the Supreme Court in Collector of Internal Revenue vs. Solano 13 is in
point, thus:
The decision of the Court of Appeals affirming what the Court of Tax Appeals
decided, established that subject assessments of July 29, 1987 were issued outside . . . The only agreement that could have suspended the running of the prescriptive
the statutory prescriptive period. Carnation filed its annual income tax and period the collection of the tax in question is, as correctly pointed out by the Court
percentage tax returns for the fiscal year ending September 30, 1981 on January of Tax Appeals, a written agreement between Solano and the Collector, entered into
15, 19828 and November 20, 1981,9 respectively. In accordance with the above- before the expiration of the of the five-year prescriptive period, extending the
quoted provision of law, private respondent's 1981 income and sales taxes could limitation prescribed by law.
have been validly assessed only until January 14, 1987 and November 19, 1986,
respectively. 10 However, Carnation's income and sales taxes were assessed only on For sure, no such written agreement concerning the said three waivers exists
July 29, 1987, beyond the five-year prescriptive period.11 between the petitioner and private respondent Carnation. 14

Petitioner BIR Commissioner contends that the waivers signed by Carnation were Verily, we discern no basis for overruling the aforesaid conclusions arrived at by the
valid although not signed by the BIR Commissioner because (a) when the BIR Court of Appeals. In fact, there is every reason to leave undisturbed the said
agents/examiners extended the period to audit and investigate Carnation's tax conclusions, having in mind the precept that all doubts as to the correctness of such
returns, the BIR gave its implied consent to such waivers; (b) the signature of the conclusions will be resolved in favor of the Court of Appeals. 15 Besides being a
Commissioner is a mere formality and the lack of it does not vitiate binding effect of reiteration of the holding of the Court of Tax Appeals, such decision should be
the waivers; and (c) that a waiver is not a contract but a unilateral act of accorded respect. Thus, the Court held in Philippine Refining Co. vs. Court of
renouncing ones right to avail of the defense of prescription and remains binding in Appeals, 16 that the Court of Tax Appeals is a highly specialized body specifically
accordance with the terms and conditions set forth in the waiver. 12 created for the purpose of reviewing tax cases. As a matter of principle, this Court
will not set aside the conclusion reached by an agency such as the Court of Tax
Petitioner's submission is inaccurate. The same tax code is clear on the matter, to Appeals which is, by the very nature of its function, dedicated exclusively to the
wit: study and consideration of tax problems and has necessarily developed an expertise
on the subject, unless there has been an abuse or improvident exercise of

42
authority. 17 This point becomes more evident in the case under consideration where On December 10, 2001, Nelia Pasco (Pasco), respondent’s accountant, executed a
the findings and conclusions of bath the Court of Tax Appeals and the Court of Waiver of the Defense of Prescription,4 which was notarized on January 22, 2002,
Appeals appear untainted by any abuse of authority, much less grave abuse of received by the BIR Enforcement Service on January 31, 2002 and by the BIR Tax
discretion. Indeed, we find the decision of the latter affirming that of the former free Fraud Division on February 4, 2002, and accepted by the Assistant Commissioner of
from any palpable error.18 the Enforcement Service, Percival T. Salazar (Salazar).

What is more, the waivers in question reveal that they are in no wise unequivocal, This was followed by a second Waiver of Defense of Prescription 5 executed by Pasco
and therefore necessitates for its binding effect the concurrence of the on February 18, 2003, notarized on February 19, 2003, received by the BIR Tax
Commissioner of Internal Revenue. In fact, in his reply dated April 18, 1995, the Fraud Division on February 28, 2003 and accepted by Assistant Commissioner
Solicitor General, representing the Commissioner of Internal Revenue, admitted that Salazar.
subject waivers executed by Carnation were "for end in consideration of the
approval by the Commissioner of Internal Revenue of its request for reinvestigation
On August 25, 2003, the BIR issued a Preliminary Assessment Notice for the taxable
and/or reconsideration of its internal revenue case involving tax assessments for the
year 1998 against the respondent. This was followed by a Formal Letter of Demand
fiscal year ended September 30, 1981 which were all pending at the time". On this
with Assessment Notices for taxable year 1998, dated September 26, 2003 which
basis neither implied consent can be presumed nor can it be contended that the
was received by respondent on November 12, 2003.
waiver required under Sec. 319 of the Tax Code is one which is unilateral nor can it
be said that concurrence to such an agreements a mere formality because it is the
very signatures of both the Commissioner of Internal Revenue and the taxpayer Respondent challenged the assessments by filing its "Protest on Various Tax
which give birth to such a valid agreement. Assessments" on December 3, 2003 and its "Legal Arguments and Documents in
Support of Protests against Various Assessments" on February 2, 2004.
WHEREFORE, the decision of the Court of Appeals is hereby AFFIRMED. No
pronouncement as to costs. On June 22, 2004, the BIR rendered a final Decision6 on the matter, requesting the
immediate payment of the following tax liabilities:
G.R. No. 178087               May 5, 2010
COMMISSIONER OF INTERNAL REVENUE, Petitioner, Kind of Tax Amount
vs. KUDOS METAL CORPORATION, Respondent.
Income Tax ₱ 9,693,897.85
The prescriptive period on when to assess taxes benefits both the government and VAT 13,962,460.90
the taxpayer.1 Exceptions extending the period to assess must, therefore, be strictly
construed. EWT 1,712,336.76

Withholding Tax-Compensation 247,353.24


This Petition for Review on Certiorari seeks to set aside the Decision2 dated March
30, 2007 of the Court of Tax Appeals (CTA) affirming the cancellation of the Penalties 8,000.00
assessment notices for having been issued beyond the prescriptive period and the
Resolution3 dated May 18, 2007 denying the motion for reconsideration.
Total ₱25,624,048.76
Factual Antecedents

On April 15, 1999, respondent Kudos Metal Corporation filed its Annual Income Tax
Return (ITR) for the taxable year 1998. Ruling of the Court of Tax Appeals, Second Division

Pursuant to a Letter of Authority dated September 7, 1999, the Bureau of Internal Believing that the government’s right to assess taxes had prescribed, respondent
Revenue (BIR) served upon respondent three Notices of Presentation of Records. filed on August 27, 2004 a Petition for Review7 with the CTA. Petitioner in turn filed
Respondent failed to comply with these notices, hence, the BIR issued a Subpeona his Answer.8
Duces Tecum dated September 21, 2006, receipt of which was acknowledged by
respondent’s President, Mr. Chan Ching Bio, in a letter dated October 20, 2000. On April 11, 2005, respondent filed an "Urgent Motion for Preferential Resolution of
the Issue on Prescription."9
A review and audit of respondent’s records then ensued.

43
On October 4, 2005, the CTA Second Division issued a Resolution 10 canceling the 2. ACIR, Large Taxpayers Service For large taxpayers cases other than those cases
assessment notices issued against respondent for having been issued beyond the falling under Subsection B hereof
prescriptive period. It found the first Waiver of the Statute of Limitations incomplete
and defective for failure to comply with the provisions of Revenue Memorandum
3. ACIR, Legal Service For cases pending verification and awaiting resolution of
Order (RMO) No. 20-90. Thus:
certain legal issues prior to prescription and for issuance/compliance of Subpoena
Duces Tecum
First, the Assistant Commissioner is not the revenue official authorized to sign the
waiver, as the tax case involves more than ₱1,000,000.00. In this regard, only the
4. ACIR, Assessment Service (AS) For cases which are pending in or subject to
Commissioner is authorized to enter into agreement with the petitioner in extending
review or approval by the ACIR, AS
the period of assessment;

Based on the foregoing, the Assistant Commissioner, Enforcement Service is


Secondly, the waiver failed to indicate the date of acceptance. Such date of
authorized to sign waivers in tax fraud cases. A perusal of the records reveals that
acceptance is necessary to determine whether the acceptance was made within the
the investigation of the subject deficiency taxes in this case was conducted by the
prescriptive period;
National Investigation Division of the BIR, which was formerly named the Tax Fraud
Division. Thus, the subject assessment is a tax fraud case.
Third, the fact of receipt by the taxpayer of his file copy was not indicated on the
original copy. The requirement to furnish the taxpayer with a copy of the waiver is
Nevertheless, the first waiver is still invalid based on the second and third grounds
not only to give notice of the existence of the document but also of the acceptance
stated by the Court in Division. Hence, it did not extend the prescriptive period to
by the BIR and the perfection of the agreement.1avvphi1
assess.

The subject waiver is therefore incomplete and defective. As such, the three-year
Moreover, assuming arguendo that the first waiver is valid, the second waiver is
prescriptive period was not tolled or extended and continued to run. x x x 11
invalid for violating Section 222(b) of the 1997 Tax Code which mandates that the
period agreed upon in a waiver of the statute can still be extended by subsequent
Petitioner moved for reconsideration but the CTA Second Division denied the motion written agreement, provided that it is executed prior to the expiration of the first
in a Resolution12 dated April 18, 2006. period agreed upon. As previously discussed, the exceptions to the law on
prescription must be strictly construed.
Ruling of the Court of Tax Appeals, En Banc
In the case at bar, the period agreed upon in the subject first waiver expired on
December 31, 2002. The second waiver in the instant case which was supposed to
On appeal, the CTA En Banc affirmed the cancellation of the assessment notices.
extend the period to assess to December 31, 2003 was executed on February 18,
Although it ruled that the Assistant Commissioner was authorized to sign the waiver
2003 and was notarized on February 19, 2003. Clearly, the second waiver was
pursuant to Revenue Delegation Authority Order (RDAO) No. 05-01, it found that
executed after the expiration of the first period agreed upon. Consequently, the
the first waiver was still invalid based on the second and third grounds stated by the
same could not have tolled the 3-year prescriptive period to assess. 13
CTA Second Division. Pertinent portions of the Decision read as follows:

Petitioner sought reconsideration but the same was unavailing.


While the Court En Banc agrees with the second and third grounds for invalidating
the first waiver, it finds that the Assistant Commissioner of the Enforcement Service
is authorized to sign the waiver pursuant to RDAO No. 05-01, which provides in part Issue
as follows:
Hence, the present recourse where petitioner interposes that:
A. For National Office cases
THE COURT OF TAX APPEALS EN BANC ERRED IN RULING THAT THE
Designated Revenue Official GOVERNMENT’S RIGHT TO ASSESS UNPAID TAXES OF RESPONDENT
PRESCRIBED.14
1. Assistant Commissioner (ACIR), For tax fraud and policy Enforcement Service
cases Petitioner’s Arguments

44
Petitioner argues that the government’s right to assess taxes is not barred by upon to assess/collect the tax after the regular three-year period of prescription,
prescription as the two waivers executed by respondent, through its accountant, should be filled up.
effectively tolled or extended the period within which the assessment can be made.
In disputing the conclusion of the CTA that the waivers are invalid, petitioner claims
2. The waiver must be signed by the taxpayer himself or his duly authorized
that respondent is estopped from adopting a position contrary to what it has
representative. In the case of a corporation, the waiver must be signed by any of its
previously taken. Petitioner insists that by acquiescing to the audit during the period
responsible officials. In case the authority is delegated by the taxpayer to a
specified in the waivers, respondent led the government to believe that the "delay"
representative, such delegation should be in writing and duly notarized.
in the process would not be utilized against it. Thus, respondent may no longer
repudiate the validity of the waivers and raise the issue of prescription.
3. The waiver should be duly notarized.
Respondent’s Arguments
4. The CIR or the revenue official authorized by him must sign the waiver indicating
that the BIR has accepted and agreed to the waiver. The date of such acceptance by
Respondent maintains that prescription had set in due to the invalidity of the
the BIR should be indicated. However, before signing the waiver, the CIR or the
waivers executed by Pasco, who executed the same without any written authority
revenue official authorized by him must make sure that the waiver is in the
from it, in clear violation of RDAO No. 5-01. As to the doctrine of estoppel by
prescribed form, duly notarized, and executed by the taxpayer or his duly
acquiescence relied upon by petitioner, respondent counters that the principle of
authorized representative.
equity comes into play only when the law is doubtful, which is not present in the
instant case.
5. Both the date of execution by the taxpayer and date of acceptance by the Bureau
should be before the expiration of the period of prescription or before the lapse of
Our Ruling
the period agreed upon in case a subsequent agreement is executed.

The petition is bereft of merit.


6. The waiver must be executed in three copies, the original copy to be attached to
the docket of the case, the second copy for the taxpayer and the third copy for the
Section 20315 of the National Internal Revenue Code of 1997 (NIRC) mandates the Office accepting the waiver. The fact of receipt by the taxpayer of his/her file copy
government to assess internal revenue taxes within three years from the last day must be indicated in the original copy to show that the taxpayer was notified of the
prescribed by law for the filing of the tax return or the actual date of filing of such acceptance of the BIR and the perfection of the agreement.19
return, whichever comes later. Hence, an assessment notice issued after the three-
year prescriptive period is no longer valid and effective. Exceptions however are
A perusal of the waivers executed by respondent’s accountant reveals the following
provided under Section 22216 of the NIRC.
infirmities:

The waivers executed by respondent’s accountant did not extend the period within
1. The waivers were executed without the notarized written authority of Pasco to
which the assessment can be made
sign the waiver in behalf of respondent.

Petitioner does not deny that the assessment notices were issued beyond the three-
2. The waivers failed to indicate the date of acceptance.
year prescriptive period, but claims that the period was extended by the two
waivers executed by respondent’s accountant.
3. The fact of receipt by the respondent of its file copy was not indicated in the
original copies of the waivers.
We do not agree.

Due to the defects in the waivers, the period to assess or collect taxes was not
Section 222 (b) of the NIRC provides that the period to assess and collect taxes
extended. Consequently, the assessments were issued by the BIR beyond the three-
may only be extended upon a written agreement between the CIR and the taxpayer
year period and are void.
executed before the expiration of the three-year period. RMO 20-90 17 issued on
April 4, 1990 and RDAO 05-0118 issued on August 2, 2001 lay down the procedure
for the proper execution of the waiver, to wit: Estoppel does not apply in this case

1. The waiver must be in the proper form prescribed by RMO 20-90. The phrase We find no merit in petitioner’s claim that respondent is now estopped from claiming
"but not after ______ 19 ___", which indicates the expiry date of the period agreed prescription since by executing the waivers, it was the one which asked for
additional time to submit the required documents.

45
In Collector of Internal Revenue v. Suyoc Consolidated Mining Company, 20 the This case has no precedent in this jurisdiction for it is the first time that such has
doctrine of estoppel prevented the taxpayer from raising the defense of prescription risen, but there are several precedents that may be invoked in American
against the efforts of the government to collect the assessed tax. However, it must jurisprudence. As Mr. Justice Cardozo has said: "The applicable principle is
be stressed that in the said case, estoppel was applied as an exception to the fundamental and unquestioned. ‘He who prevents a thing from being done may not
statute of limitations on collection of taxes and not on the assessment of taxes, as avail himself of the nonperformance which he has himself occasioned, for the law
the BIR was able to make an assessment within the prescribed period. More says to him in effect "this is your own act, and therefore you are not damnified."’
important, there was a finding that the taxpayer made several requests or positive "(R. H. Stearns Co. vs. U.S., 78 L. ed., 647). Or, as was aptly said, "The tax could
acts to convince the government to postpone the collection of taxes, viz: have been collected, but the government withheld action at the specific request of
the plaintiff. The plaintiff is now estopped and should not be permitted to raise the
defense of the Statute of Limitations." [Newport Co. vs. U.S., (DC-WIS), 34 F.
It appears that the first assessment made against respondent based on its second
Supp. 588].21
final return filed on November 28, 1946 was made on February 11, 1947. Upon
receipt of this assessment respondent requested for at least one year within which
to pay the amount assessed although it reserved its right to question the Conversely, in this case, the assessments were issued beyond the prescribed
correctness of the assessment before actual payment. Petitioner granted an period. Also, there is no showing that respondent made any request to persuade the
extension of only three months. When it failed to pay the tax within the period BIR to postpone the issuance of the assessments.
extended, petitioner sent respondent a letter on November 28, 1950 demanding
payment of the tax as assessed, and upon receipt of the letter respondent asked for
The doctrine of estoppel cannot be applied in this case as an exception to the
a reinvestigation and reconsideration of the assessment. When this request was
statute of limitations on the assessment of taxes considering that there is a detailed
denied, respondent again requested for a reconsideration on April 25, 1952, which
procedure for the proper execution of the waiver, which the BIR must strictly follow.
was denied on May 6, 1953, which denial was appealed to the Conference Staff. The
As we have often said, the doctrine of estoppel is predicated on, and has its origin
appeal was heard by the Conference Staff from September 2, 1953 to July 16,
in, equity which, broadly defined, is justice according to natural law and right. 22 As
1955, and as a result of these various negotiations, the assessment was finally
such, the doctrine of estoppel cannot give validity to an act that is prohibited by law
reduced on July 26, 1955. This is the ruling which is now being questioned after a
or one that is against public policy.23 It should be resorted to solely as a means of
protracted negotiation on the ground that the collection of the tax has already
preventing injustice and should not be permitted to defeat the administration of the
prescribed.
law, or to accomplish a wrong or secure an undue advantage, or to extend beyond
them requirements of the transactions in which they originate. 24 Simply put, the
It is obvious from the foregoing that petitioner refrained from collecting the tax by doctrine of estoppel must be sparingly applied.
distraint or levy or by proceeding in court within the 5-year period from the filing of
the second amended final return due to the several requests of respondent for
Moreover, the BIR cannot hide behind the doctrine of estoppel to cover its failure to
extension to which petitioner yielded to give it every opportunity to prove its claim
comply with RMO 20-90 and RDAO 05-01, which the BIR itself issued. As stated
regarding the correctness of the assessment. Because of such requests, several
earlier, the BIR failed to verify whether a notarized written authority was given by
reinvestigations were made and a hearing was even held by the Conference Staff
the respondent to its accountant, and to indicate the date of acceptance and the
organized in the collection office to consider claims of such nature which, as the
receipt by the respondent of the waivers. Having caused the defects in the waivers,
record shows, lasted for several months. After inducing petitioner to delay collection
the BIR must bear the consequence. It cannot shift the blame to the taxpayer. To
as he in fact did, it is most unfair for respondent to now take advantage of such
stress, a waiver of the statute of limitations, being a derogation of the taxpayer’s
desistance to elude his deficiency income tax liability to the prejudice of the
right to security against prolonged and unscrupulous investigations, must be
Government invoking the technical ground of prescription.
carefully and strictly construed.25

While we may agree with the Court of Tax Appeals that a mere request for
As to the alleged delay of the respondent to furnish the BIR of the required
reexamination or reinvestigation may not have the effect of suspending the running
documents, this cannot be taken against respondent. Neither can the BIR use this
of the period of limitation for in such case there is need of a written agreement to
as an excuse for issuing the assessments beyond the three-year period because
extend the period between the Collector and the taxpayer, there are cases however
with or without the required documents, the CIR has the power to make
where a taxpayer may be prevented from setting up the defense of prescription
assessments based on the best evidence obtainable.26
even if he has not previously waived it in writing as when by his repeated requests
or positive acts the Government has been, for good reasons, persuaded to postpone
collection to make him feel that the demand was not unreasonable or that no WHEREFORE, the petition is DENIED. The assailed Decision dated March 30, 2007
harassment or injustice is meant by the Government. And when such situation and Resolution dated May 18, 2007 of the Court of Tax Appeals are
comes to pass there are authorities that hold, based on weighty reasons, that such hereby AFFIRMED.
an attitude or behavior should not be countenanced if only to protect the interest of
the Government. SO ORDERED.

46
G.R. No. 170257               September 7, 2011 GRT-94-
RIZAL COMMERCIAL BANKING CORPORATION, Petitioner, 0202-
vs. COMMISSIONER OF INTERNAL REVENUE, Respondent. 2000)
Deficiency Final Withholding Tax
This is a petition for review on certiorari under Rule 45 seeking to set aside the July 1995 (ST-
27, 2005 Decision1 and October 26, 2005 Resolution2 of the Court of Tax Appeals En EWT-95-
Banc (CTA-En Banc) in C.T.A. E.B. No. 83 entitled "Rizal Commercial Banking 0203-
Corporation v. Commissioner of Internal Revenue." 2000) 64,365,610.12 58,757,866.78 25,000.00 123,148,477.15
1994 (ST-
THE FACTS EWT-94-
0204-
2000) 53,058,075.25 59,047,096.34 25,000.00 112,130,171.59
Petitioner Rizal Commercial Banking Corporation (RCBC) is a corporation engaged in Deficiency Final Tax on FCDU Onshore Income
general banking operations. It seasonably filed its Corporation Annual Income Tax
1995 (ST-
Returns for Foreign Currency Deposit Unit for the calendar years 1994 and 1995. 3
OT-95-
0205-
On August 15, 1996, RCBC received Letter of Authority No. 133959 issued by then 2000) 81,508,718.20 61,901,963,.52 25,000.00 143,435,681.72
Commissioner of Internal Revenue (CIR) Liwayway Vinzons-Chato, authorizing a 1994 (ST-
special audit team to examine the books of accounts and other accounting records OT-94-
for all internal revenue taxes from January 1, 1994 to December 31, 1995. 4 0206-
2000) 34,429,503.10 33,052,322.98 25,000.00 67,506,826.08
On January 23, 1997, RCBC executed two Waivers of the Defense of Prescription Deficiency Expanded Withholding Tax
Under the Statute of Limitations of the National Internal Revenue Code covering the 1995 (ST-
internal revenue taxes due for the years 1994 and 1995, effectively extending the EWT-95-
period of the Bureau of Internal Revenue (BIR) to assess up to December 31, 0207-
2000.5 2000) 5,051,415.22 4,583,640.33 113,000.00 9,748,055.55
1994 (ST-
Subsequently, on January 27, 2000, RCBC received a Formal Letter of Demand EWT-94-
together with Assessment Notices from the BIR for the following deficiency tax 0208-
assessments:6 2000) 4,482,740.35 4,067,626.31 78,200.00 8,628,566.66
Deficiency Documentary Stamp Tax
Compromise 1995 (ST-
Particulars Basic Tax Interest Penalties Total DST1-95-
0209-
2000) 351,900,539.39 315,804,946.26 250,000.00 667,955,485.65
Deficiency Income Tax 1995 (ST-
1995 (ST- DST2-95-
INC-95- 0210-
0199- 2000) 367,207,105.29 331,535,844.68 300,000.00 699,042,949.97
2000) ₱ 252,150,988.01 ₱ 191,496,585.96 ₱ 25,000.00 ₱ 443,672,573.97 1994 (ST-
1994 (ST- DST3-94-
INC-94- 0211-
0200- 2000) 460,370,640.05 512,193,460.02 300,000.00 972,864,100.07
2000) 216,478,397.90 207,819,261.99 25,000.00 424,322,659.89 1994 (ST-
Deficiency Gross Receipts Tax DST4-94-
1995 (ST- 0212-
GRT-95- 2000) 223,037,675.89 240,050,706.09 300,000.00 463,388,381.98
0201-
2000) 13,697,083.68 12,428,696.21 2,819,745.52 28,945,525.41
TOTALS ₱2,130,226,954.83 ₱2,035,495,733.8 ₱4,335,945.52 ₱4,170,058,634.49
1994 (ST- 2,488,462.38 2,755,716.42 25,000.00 5,269,178.80
47
9 000005)
1994 (DST2- 17,040,104.84   4,260,026.21 21,300,131.05
94-000001)

Disagreeing with the said deficiency tax assessment, RCBC filed a protest on
February 24, 2000 and later submitted the relevant documentary evidence to TOTALS ₱ ₱ ₱ 12,291,947.73 ₱
support it. Much later on November 20, 2000, it filed a petition for review before the 164,712,903.44 126,155,645.38 303,160,496.55
CTA, pursuant to Section 228 of the 1997 Tax Code.7

On December 6, 2000, RCBC received another Formal Letter of Demand with On the same day, RCBC paid the following deficiency taxes as assessed by the BIR: 9
Assessment Notices dated October 20, 2000, following the reinvestigation it
requested, which drastically reduced the original amount of deficiency taxes to the
following:8 Particulars 1994 1995 Total

Deficiency Income Tax ₱ ₱ 722,236.11 ₱ 3,687,785.55


Compromise
2,965,549.44
Particulars Basic Tax Interest Penalties Total
Deficiency Gross Receipts Tax 300,695.84 6,701,893.17 7,002,589.01
Deficiency Income Tax
Deficiency Final Withholding 410,174.44 714,682.02 1,124,856.46
1995 (INC-95- ₱ 374,348.45 ₱ 346,656.92   ₱ 721,005.37 Tax
000003)
1994 (INC-94- 1,392,366.28 1,568,605.52   2,960,971.80 Deficiency Expanded 672,490.14 1,052,753.48 1,725,243.62
000002) Withholding Tax
Deficiency Gross Receipts Tax
Deficiency Documentary 1,131,330.92 749,863.40 1,881,194.32
1995 (GRT-95- 2,000,926.96 3,322,589.63 ₱ 1,367,222.04 6,690,738.63 Stamp Tax
000004)
1994 (GRT-94- 138,368.61 161,872.32   300,240.93 TOTALS ₱ ₱ ₱
000003) 5,480,240.78 9,941,428.18 15,421,668.96
Deficiency Final Withholding Tax
1995 (FT-95- 362,203.47 351,287.75   713,491.22
RCBC, however, refused to pay the following assessments for deficiency onshore tax
000005)
and documentary stamp tax which remained to be the subjects of its petition for
1994 (FT-94- 188,746.43 220,807.47   409,553.90 review:10
000004)
Deficiency Final Tax on FCDU Onshore Income
1995 (OT-95- 81,508,718.20 79,052,291.08   160,561,009.28 Particulars 1994 1995 Total
000006)
Deficiency Final Tax on FCDU Onshore Income
1994 (OT-94- 34,429,503.10 40,277,802.26   74,707,305.36
000005) Basic ₱ 34,429,503.10 ₱ 81,508,718.20 ₱ 115,938,221.30
Deficiency Expanded Withholding Tax
1995 (EWT-95- 520,869.72 505,171.80 25,000.00 1,051,041.03 Interest 40,277,802.26 79,052,291.08 119,330,093.34
000004)
Sub Total ₱ 74,707,305.36 ₱ 160,561,009.28 ₱ 235,268,314.64
1994 (EWT-94- 297,949.95 348,560.63 25,000.00 671,510.58
000003) Deficiency Documentary Stamp Tax
Deficiency Documentary Stamp Tax
Basic ₱ 17,040,104.84 ₱ 24,953,842.46 ₱ 41,993,947.30
1995 (DST-95- 599,890.72   149,972.68 749,863.40
000006) Surcharge 4,260,026.21 6,238,460.62 10,498,486.83
1995 (DST2- 24,953,842.46   6,238,460.62 31,192,303.08
95-000002) Sub Total ₱ 21,300,131.05 ₱ 31,192,303.08 ₱ 52,492,434.13
1994 (DST-94- 905,064.74   226,266.18 1,131,330.92
48
TOTALS ₱ 96,007,436.41 ₱ 191,753,312.36 ₱ 287,760,748.77 deficiency taxes. As such, it modified its earlier decision and ordered RCBC to pay
the amount of ₱ 132,654,261.69 plus 20% delinquency tax. 18

RCBC argued that the waivers of the Statute of Limitations which it executed on RCBC elevated the case to the CTA-En Banc where it raised the following issues:
January 23, 1997 were not valid because the same were not signed or conformed to
by the respondent CIR as required under Section 222(b) of the Tax Code. 11 As
regards the deficiency FCDU onshore tax, RCBC contended that because the I. Whether or not the right of the respondent to assess deficiency
onshore tax was collected in the form of a final withholding tax, it was the borrower, onshore tax and documentary stamp tax for taxable year 1994 and
constituted by law as the withholding agent, that was primarily liable for the 1995 had already prescribed when it issued the formal letter of
remittance of the said tax.12 demand and assessment notices for the said taxable years.

On December 15, 2004, the First Division of the Court of Tax Appeals (CTA-First II. Whether or not petitioner is liable for deficiency onshore tax for
Division) promulgated its Decision13 which partially granted the petition for review. taxable year 1994 and 1995.
It considered as closed and terminated the assessments for deficiency income tax,
deficiency gross receipts tax, deficiency final withholding tax, deficiency expanded III. Whether or not petitioner’s special savings account is subject
withholding tax, and deficiency documentary stamp tax (not an industry issue) for to documentary stamp tax under then Section 180 of the 1993 Tax
1994 and 1995.14 It, however, upheld the assessment for deficiency final tax on Code.19
FCDU onshore income and deficiency documentary stamp tax for 1994 and 1995
and ordered RCBC to pay the following amounts plus 20% delinquency tax: 15
The CTA-En Banc, in its assailed Decision, denied the petition for lack of merit. It
ruled that by receiving, accepting and paying portions of the reduced assessment,
Particulars 1994 1995 Total RCBC bound itself to the new assessment, implying that it recognized the validity of
the waivers.20 RCBC could not assail the validity of the waivers after it had received
Deficiency Final Tax on FCDU Onshore Income and accepted certain benefits as a result of the execution of the said waivers. 21 As
to the deficiency onshore tax, it held that because the payor-borrower was merely
Basic ₱ 22,356,324.43 ₱ 16,067,952.86 ₱ 115,938, 221.30 designated by law to withhold and remit the said tax, it would then follow that the
tax should be imposed on RCBC as the payee-bank. 22 Finally, in relation to the
Interest 26,153,837.08 15,583,713.19 119,330,093.34 assessment of the deficiency documentary stamp tax on petitioner’s special savings
Sub Total 48,510,161.51 31,651,666.05 119,330,093.34 account, it held that petitioner’s special savings account was a certificate of deposit
and, as such, was subject to documentary stamp tax. 23
Deficiency Documentary Stamp Tax (Industry Issue)
Hence, this petition.
Basic ₱ 17,040,104.84 ₱ 24,953,842.46 ₱ 41,993,947.30

Surcharge 4,260,026.21 6,238,460.62 10,498,486.83 While awaiting the decision of this Court, RCBC filed its Manifestation dated July 22,
2009, informing the Court that this petition, relative to the DST deficiency
Sub Total 21,300,131.05 31,192,303.08 52,492,434.13
assessment, had been rendered moot and academic by its payment of the tax
TOTALS ₱ 69,810,292.56 ₱ 62,843,969.13 ₱ 171,822,527.47 deficiencies on Documentary Stamp Tax (DST) on Special Savings
Account (SSA) for taxable years 1994 and 1995 after the BIR approved its
applications for tax abatement.24
Unsatisfied, RCBC filed its Motion for Reconsideration on January 21, 2005, arguing
that: (1) the CTA erred in its addition of the total amount of deficiency taxes and In its November 17, 2009 Comment to the Manifestation, the CIR pointed out that
the correct amount should only be ₱ 132,654,261.69 and not ₱ 171,822,527.47; (2) the only remaining issues raised in the present petition were those pertaining to
the CTA erred in holding that RCBC was estopped from questioning the validity of RCBC’s deficiency tax on FCDU Onshore Income for taxable years 1994 and 1995 in
the waivers; (3) it was the payor-borrower as withholding tax agent, and not RCBC, the aggregate amount of ₱ 80,161,827.56 plus 20% delinquency interest per
who was liable to pay the final tax on FCDU, and (4) RCBC’s special savings account annum. The CIR prayed that RCBC be considered to have withdrawn its appeal with
was not subject to documentary stamp tax. 16 respect to the CTA-En Banc ruling on its DST on SSA deficiency for taxable years
1994 and 1995 and that the questioned CTA decision regarding RCBC’s deficiency
In its Resolution17 dated April 11, 2005, the CTA-First Division substantially upheld tax on FCDU Onshore Income for the same period be affirmed.25
its earlier ruling, except for its inadvertence in the addition of the total amount of
THE ISSUES

49
Thus, only the following issues remain to be resolved by this Court: Liability for Deficiency
Onshore Withholding Tax
Whether petitioner, by paying the other tax assessment covered by the
waivers of the statute of limitations, is rendered estopped from questioning RCBC is convinced that it is the payor-borrower, as withholding agent, who is
the validity of the said waivers with respect to the assessment of deficiency directly liable for the payment of onshore tax, citing Section 2.57(A) of Revenue
onshore tax.26 Regulations No. 2-98 which states:

and (A) Final Withholding Tax. — Under the final withholding tax system the amount of
income tax withheld by the withholding agent is constituted as a full and final
payment of the income tax due from the payee on the said income. The liability
Whether petitioner, as payee-bank, can be held liable for deficiency
for payment of the tax rests primarily on the payor as a withholding agent.
onshore tax, which is mandated by law to be collected at source in the form
Thus, in case of his failure to withhold the tax or in case of under
of a final withholding tax.27
withholding, the deficiency tax shall be collected from the
payor/withholding agent. The payee is not required to file an income tax return
THE COURT’S RULING for the particular income. (Emphasis supplied)

Petitioner is estopped from The petitioner is mistaken.


questioning the validity of the waivers
Before any further discussion, it should be pointed out that RCBC erred in citing the
RCBC assails the validity of the waivers of the statute of limitations on the ground abovementioned Revenue Regulations No. 2-98 because the same governs
that the said waivers were merely attested to by Sixto Esquivias, then Coordinator collection at source on income paid only on or after January 1, 1998. The deficiency
for the CIR, and that he failed to indicate acceptance or agreement of the CIR, as withholding tax subject of this petition was supposed to have been withheld on
required under Section 223 (b) of the 1977 Tax Code.28 RCBC further argues that income paid during the taxable years of 1994 and 1995. Hence, Revenue
the principle of estoppel cannot be applied against it because its payment of the Regulations No. 2-98 obviously does not apply in this case.
other tax assessments does not signify a clear intention on its part to give up its
right to question the validity of the waivers. 29
In Chamber of Real Estate and Builders’ Associations, Inc. v. The Executive
Secretary,32 the Court has explained that the purpose of the withholding tax system
The Court disagrees. is three-fold: (1) to provide the taxpayer with a convenient way of paying his tax
liability; (2) to ensure the collection of tax, and (3) to improve the government’s
Under Article 1431 of the Civil Code, the doctrine of estoppel is anchored on the rule cashflow. Under the withholding tax system, the payor is the taxpayer upon whom
that "an admission or representation is rendered conclusive upon the person making the tax is imposed, while the withholding agent simply acts as an agent or a
it, and cannot be denied or disproved as against the person relying thereon." A collector of the government to ensure the collection of taxes. 33 1avvphi1
party is precluded from denying his own acts, admissions or representations to the
prejudice of the other party in order to prevent fraud and falsehood. 30 It is, therefore, indisputable that the withholding agent is merely a tax collector and
not a taxpayer, as elucidated by this Court in the case of Commissioner of Internal
Estoppel is clearly applicable to the case at bench. RCBC, through its partial Revenue v. Court of Appeals,34 to wit:
payment of the revised assessments issued within the extended period as provided
for in the questioned waivers, impliedly admitted the validity of those waivers. Had In the operation of the withholding tax system, the withholding agent is the payor,
petitioner truly believed that the waivers were invalid and that the assessments a separate entity acting no more than an agent of the government for the collection
were issued beyond the prescriptive period, then it should not have paid the of the tax in order to ensure its payments; the payer is the taxpayer – he is the
reduced amount of taxes in the revised assessment. RCBC’s subsequent person subject to tax imposed by law; and the payee is the taxing authority. In
action effectively belies its insistence that the waivers are invalid. The records show other words, the withholding agent is merely a tax collector, not a taxpayer. Under
that on December 6, 2000, upon receipt of the revised assessment, RCBC the withholding system, however, the agent-payor becomes a payee by fiction of
immediately made payment on the uncontested taxes. Thus, RCBC is estopped from law. His (agent) liability is direct and independent from the taxpayer,
questioning the validity of the waivers. To hold otherwise and allow a party to because the income tax is still imposed on and due from the latter. The
gainsay its own act or deny rights which it had previously recognized would run agent is not liable for the tax as no wealth flowed into him – he earned no
counter to the principle of equity which this institution holds dear. 31 income. The Tax Code only makes the agent personally liable for the tax arising
from the breach of its legal duty to withhold as distinguished from its duty to pay
tax since:

50
"the government’s cause of action against the withholding agent is not for decision is not supported by substantial evidence or there is a showing of abuse or
the collection of income tax, but for the enforcement of the withholding improvident exercise of authority on the part of the Tax Court. 38
provision of Section 53 of the Tax Code, compliance with which is imposed on
the withholding agent and not upon the taxpayer."35 (Emphases supplied)
WHEREFORE, the petition is DENIED.

Based on the foregoing, the liability of the withholding agent is independent from
G.R. No. 170257               September 7, 2011
that of the taxpayer.1âwphi1 The former cannot be made liable for the tax due
RIZAL COMMERCIAL BANKING CORPORATION, Petitioner,
because it is the latter who earned the income subject to withholding tax. The
vs. COMMISSIONER OF INTERNAL REVENUE, Respondent.
withholding agent is liable only insofar as he failed to perform his duty to withhold
the tax and remit the same to the government. The liability for the tax, however,
remains with the taxpayer because the gain was realized and received by him. This is a petition for review on certiorari under Rule 45 seeking to set aside the July
27, 2005 Decision1 and October 26, 2005 Resolution2 of the Court of Tax Appeals En
Banc (CTA-En Banc) in C.T.A. E.B. No. 83 entitled "Rizal Commercial Banking
While the payor-borrower can be held accountable for its negligence in performing
Corporation v. Commissioner of Internal Revenue."
its duty to withhold the amount of tax due on the transaction, RCBC, as the
taxpayer and the one which earned income on the transaction, remains liable for
the payment of tax as the taxpayer shares the responsibility of making certain that THE FACTS
the tax is properly withheld by the withholding agent, so as to avoid any penalty
that may arise from the non-payment of the withholding tax due. Petitioner Rizal Commercial Banking Corporation (RCBC) is a corporation engaged in
general banking operations. It seasonably filed its Corporation Annual Income Tax
RCBC cannot evade its liability for FCDU Onshore Tax by shifting the blame on the Returns for Foreign Currency Deposit Unit for the calendar years 1994 and 1995. 3
payor-borrower as the withholding agent. As such, it is liable for payment of
deficiency onshore tax on interest income derived from foreign currency loans, On August 15, 1996, RCBC received Letter of Authority No. 133959 issued by then
pursuant to Section 24(e)(3) of the National Internal Revenue Code of 1993: Commissioner of Internal Revenue (CIR) Liwayway Vinzons-Chato, authorizing a
special audit team to examine the books of accounts and other accounting records
Sec. 24. Rates of tax on domestic corporations. for all internal revenue taxes from January 1, 1994 to December 31, 1995. 4

(e) Tax on certain incomes derived by domestic corporations On January 23, 1997, RCBC executed two Waivers of the Defense of Prescription
Under the Statute of Limitations of the National Internal Revenue Code covering the
internal revenue taxes due for the years 1994 and 1995, effectively extending the
(3) Tax on income derived under the Expanded Foreign Currency Deposit System.
period of the Bureau of Internal Revenue (BIR) to assess up to December 31,
– Income derived by a depository bank under the expanded foreign currency
2000.5
deposit system from foreign currency transactions with nonresidents, offshore
banking units in the Philippines, local commercial banks including branches of
foreign banks that may be authorized by the Central Bank to transact business with Subsequently, on January 27, 2000, RCBC received a Formal Letter of Demand
foreign currency depository system units and other depository banks under the together with Assessment Notices from the BIR for the following deficiency tax
expanded foreign currency deposit system shall be exempt from all taxes, except assessments:6
taxable income from such transactions as may be specified by the Secretary of
Finance, upon recommendation of the Monetary Board to be subject to the usual Compromise
income tax payable by banks: Provided, That interest income from foreign Particulars Basic Tax Interest Penalties Total
currency loans granted by such depository banks under said expanded
system to residents (other than offshore banking units in the Philippines or
other depository banks under the expanded system) shall be subject to a Deficiency Income Tax
10% tax. (Emphasis supplied) 1995 (ST-
INC-95-
As a final note, this Court has consistently held that findings and conclusions of the 0199-
CTA shall be accorded the highest respect and shall be presumed valid, in the 2000) ₱ 252,150,988.01 ₱ 191,496,585.96 ₱ 25,000.00 ₱ 443,672,573.97
absence of any clear and convincing proof to the contrary.36 The CTA, as a 1994 (ST-
specialized court dedicated exclusively to the study and resolution of tax problems, INC-94-
has developed an expertise on the subject of taxation.37 As such, its decisions shall 0200-
not be lightly set aside on appeal, unless this Court finds that the questioned 2000) 216,478,397.90 207,819,261.99 25,000.00 424,322,659.89

51
Deficiency Gross Receipts Tax DST4-94-
1995 (ST- 0212-
GRT-95- 2000)
0201-
2000) 13,697,083.68 12,428,696.21 2,819,745.52 28,945,525.41
₱2,035,495,733.8
1994 (ST- TOTALS ₱2,130,226,954.83 ₱4,335,945.52 ₱4,170,058,634.49
9
GRT-94-
0202-
2000) 2,488,462.38 2,755,716.42 25,000.00 5,269,178.80
Deficiency Final Withholding Tax Disagreeing with the said deficiency tax assessment, RCBC filed a protest on
1995 (ST- February 24, 2000 and later submitted the relevant documentary evidence to
EWT-95- support it. Much later on November 20, 2000, it filed a petition for review before the
0203- CTA, pursuant to Section 228 of the 1997 Tax Code.7
2000) 64,365,610.12 58,757,866.78 25,000.00 123,148,477.15
1994 (ST- On December 6, 2000, RCBC received another Formal Letter of Demand with
EWT-94- Assessment Notices dated October 20, 2000, following the reinvestigation it
0204- requested, which drastically reduced the original amount of deficiency taxes to the
2000) 53,058,075.25 59,047,096.34 25,000.00 112,130,171.59 following:8
Deficiency Final Tax on FCDU Onshore Income
1995 (ST-
OT-95- Compromise
0205- Particulars Basic Tax Interest Penalties Total
2000) 81,508,718.20 61,901,963,.52 25,000.00 143,435,681.72
1994 (ST- Deficiency Income Tax
OT-94- 1995 (INC-95- ₱ 374,348.45 ₱ 346,656.92   ₱ 721,005.37
0206- 000003)
2000) 34,429,503.10 33,052,322.98 25,000.00 67,506,826.08 1994 (INC-94- 1,392,366.28 1,568,605.52   2,960,971.80
Deficiency Expanded Withholding Tax 000002)
1995 (ST- Deficiency Gross Receipts Tax
EWT-95- 1995 (GRT-95- 2,000,926.96 3,322,589.63 ₱ 1,367,222.04 6,690,738.63
0207- 000004)
2000) 5,051,415.22 4,583,640.33 113,000.00 9,748,055.55
1994 (GRT-94- 138,368.61 161,872.32   300,240.93
1994 (ST- 000003)
EWT-94-
Deficiency Final Withholding Tax
0208-
2000) 4,482,740.35 4,067,626.31 78,200.00 8,628,566.66 1995 (FT-95- 362,203.47 351,287.75   713,491.22
000005)
Deficiency Documentary Stamp Tax
1994 (FT-94- 188,746.43 220,807.47   409,553.90
1995 (ST-
000004)
DST1-95-
0209- Deficiency Final Tax on FCDU Onshore Income
2000) 351,900,539.39 315,804,946.26 250,000.00 667,955,485.65 1995 (OT-95- 81,508,718.20 79,052,291.08   160,561,009.28
1995 (ST- 000006)
DST2-95- 1994 (OT-94- 34,429,503.10 40,277,802.26   74,707,305.36
0210- 000005)
2000) 367,207,105.29 331,535,844.68 300,000.00 699,042,949.97 Deficiency Expanded Withholding Tax
1994 (ST- 1995 (EWT-95- 520,869.72 505,171.80 25,000.00 1,051,041.03
DST3-94- 000004)
0211- 1994 (EWT-94- 297,949.95 348,560.63 25,000.00 671,510.58
2000) 460,370,640.05 512,193,460.02 300,000.00 972,864,100.07 000003)
1994 (ST- 223,037,675.89 240,050,706.09 300,000.00 463,388,381.98
52
Deficiency Documentary Stamp Tax Deficiency Documentary Stamp Tax
1995 (DST-95- 599,890.72   149,972.68 749,863.40
000006) Basic ₱ 17,040,104.84 ₱ 24,953,842.46 ₱ 41,993,947.30
1995 (DST2- 24,953,842.46   6,238,460.62 31,192,303.08
Surcharge 4,260,026.21 6,238,460.62 10,498,486.83
95-000002)
1994 (DST-94- 905,064.74   226,266.18 1,131,330.92 Sub Total ₱ 21,300,131.05 ₱ 31,192,303.08 ₱ 52,492,434.13
000005)
1994 (DST2- 17,040,104.84   4,260,026.21 21,300,131.05 TOTALS ₱ 96,007,436.41 ₱ 191,753,312.36 ₱ 287,760,748.77
94-000001)
RCBC argued that the waivers of the Statute of Limitations which it executed on
TOTALS ₱ ₱ ₱ 12,291,947.73 ₱ January 23, 1997 were not valid because the same were not signed or conformed to
164,712,903.44 126,155,645.38 303,160,496.55 by the respondent CIR as required under Section 222(b) of the Tax Code. 11 As
regards the deficiency FCDU onshore tax, RCBC contended that because the
onshore tax was collected in the form of a final withholding tax, it was the borrower,
constituted by law as the withholding agent, that was primarily liable for the
On the same day, RCBC paid the following deficiency taxes as assessed by the BIR: 9 remittance of the said tax.12

On December 15, 2004, the First Division of the Court of Tax Appeals (CTA-First
Particulars 1994 1995 Total
Division) promulgated its Decision13 which partially granted the petition for review.
Deficiency Income Tax ₱ ₱ 722,236.11 ₱ 3,687,785.55 It considered as closed and terminated the assessments for deficiency income tax,
2,965,549.44 deficiency gross receipts tax, deficiency final withholding tax, deficiency expanded
withholding tax, and deficiency documentary stamp tax (not an industry issue) for
Deficiency Gross Receipts Tax 300,695.84 6,701,893.17 7,002,589.01 1994 and 1995.14 It, however, upheld the assessment for deficiency final tax on
FCDU onshore income and deficiency documentary stamp tax for 1994 and 1995
Deficiency Final Withholding 410,174.44 714,682.02 1,124,856.46 and ordered RCBC to pay the following amounts plus 20% delinquency tax: 15
Tax

Deficiency Expanded 672,490.14 1,052,753.48 1,725,243.62 Particulars 1994 1995 Total


Withholding Tax
Deficiency Final Tax on FCDU Onshore Income
Deficiency Documentary 1,131,330.92 749,863.40 1,881,194.32
Stamp Tax Basic ₱ 22,356,324.43 ₱ 16,067,952.86 ₱ 115,938, 221.30

TOTALS ₱ ₱ ₱ Interest 26,153,837.08 15,583,713.19 119,330,093.34


5,480,240.78 9,941,428.18 15,421,668.96
Sub Total 48,510,161.51 31,651,666.05 119,330,093.34

RCBC, however, refused to pay the following assessments for deficiency onshore tax Deficiency Documentary Stamp Tax (Industry Issue)
and documentary stamp tax which remained to be the subjects of its petition for
Basic ₱ 17,040,104.84 ₱ 24,953,842.46 ₱ 41,993,947.30
review:10
Surcharge 4,260,026.21 6,238,460.62 10,498,486.83
Particulars 1994 1995 Total Sub Total 21,300,131.05 31,192,303.08 52,492,434.13
Deficiency Final Tax on FCDU Onshore Income TOTALS ₱ 69,810,292.56 ₱ 62,843,969.13 ₱ 171,822,527.47
Basic ₱ 34,429,503.10 ₱ 81,508,718.20 ₱ 115,938,221.30
Unsatisfied, RCBC filed its Motion for Reconsideration on January 21, 2005, arguing
Interest 40,277,802.26 79,052,291.08 119,330,093.34
that: (1) the CTA erred in its addition of the total amount of deficiency taxes and
Sub Total ₱ 74,707,305.36 ₱ 160,561,009.28 ₱ 235,268,314.64 the correct amount should only be ₱ 132,654,261.69 and not ₱ 171,822,527.47; (2)
the CTA erred in holding that RCBC was estopped from questioning the validity of
the waivers; (3) it was the payor-borrower as withholding tax agent, and not RCBC,
53
who was liable to pay the final tax on FCDU, and (4) RCBC’s special savings account respect to the CTA-En Banc ruling on its DST on SSA deficiency for taxable years
was not subject to documentary stamp tax. 16 1994 and 1995 and that the questioned CTA decision regarding RCBC’s deficiency
tax on FCDU Onshore Income for the same period be affirmed.25
In its Resolution17 dated April 11, 2005, the CTA-First Division substantially upheld
its earlier ruling, except for its inadvertence in the addition of the total amount of THE ISSUES
deficiency taxes. As such, it modified its earlier decision and ordered RCBC to pay
the amount of ₱ 132,654,261.69 plus 20% delinquency tax. 18
Thus, only the following issues remain to be resolved by this Court:

RCBC elevated the case to the CTA-En Banc where it raised the following issues:
Whether petitioner, by paying the other tax assessment covered by the
waivers of the statute of limitations, is rendered estopped from questioning
I. Whether or not the right of the respondent to assess deficiency the validity of the said waivers with respect to the assessment of deficiency
onshore tax and documentary stamp tax for taxable year 1994 and onshore tax.26
1995 had already prescribed when it issued the formal letter of
demand and assessment notices for the said taxable years.
and

II. Whether or not petitioner is liable for deficiency onshore tax for
Whether petitioner, as payee-bank, can be held liable for deficiency
taxable year 1994 and 1995.
onshore tax, which is mandated by law to be collected at source in the form
of a final withholding tax.27
III. Whether or not petitioner’s special savings account is subject
to documentary stamp tax under then Section 180 of the 1993 Tax
THE COURT’S RULING
Code.19

Petitioner is estopped from


The CTA-En Banc, in its assailed Decision, denied the petition for lack of merit. It
questioning the validity of the waivers
ruled that by receiving, accepting and paying portions of the reduced assessment,
RCBC bound itself to the new assessment, implying that it recognized the validity of
the waivers.20 RCBC could not assail the validity of the waivers after it had received RCBC assails the validity of the waivers of the statute of limitations on the ground
and accepted certain benefits as a result of the execution of the said waivers. 21 As that the said waivers were merely attested to by Sixto Esquivias, then Coordinator
to the deficiency onshore tax, it held that because the payor-borrower was merely for the CIR, and that he failed to indicate acceptance or agreement of the CIR, as
designated by law to withhold and remit the said tax, it would then follow that the required under Section 223 (b) of the 1977 Tax Code.28 RCBC further argues that
tax should be imposed on RCBC as the payee-bank. 22 Finally, in relation to the the principle of estoppel cannot be applied against it because its payment of the
assessment of the deficiency documentary stamp tax on petitioner’s special savings other tax assessments does not signify a clear intention on its part to give up its
account, it held that petitioner’s special savings account was a certificate of deposit right to question the validity of the waivers. 29
and, as such, was subject to documentary stamp tax. 23
The Court disagrees.
Hence, this petition.
Under Article 1431 of the Civil Code, the doctrine of estoppel is anchored on the rule
While awaiting the decision of this Court, RCBC filed its Manifestation dated July 22, that "an admission or representation is rendered conclusive upon the person making
2009, informing the Court that this petition, relative to the DST deficiency it, and cannot be denied or disproved as against the person relying thereon." A
assessment, had been rendered moot and academic by its payment of the tax party is precluded from denying his own acts, admissions or representations to the
deficiencies on Documentary Stamp Tax (DST) on Special Savings prejudice of the other party in order to prevent fraud and falsehood. 30
Account (SSA) for taxable years 1994 and 1995 after the BIR approved its
applications for tax abatement.24 Estoppel is clearly applicable to the case at bench. RCBC, through its partial
payment of the revised assessments issued within the extended period as provided
In its November 17, 2009 Comment to the Manifestation, the CIR pointed out that for in the questioned waivers, impliedly admitted the validity of those waivers. Had
the only remaining issues raised in the present petition were those pertaining to petitioner truly believed that the waivers were invalid and that the assessments
RCBC’s deficiency tax on FCDU Onshore Income for taxable years 1994 and 1995 in were issued beyond the prescriptive period, then it should not have paid the
the aggregate amount of ₱ 80,161,827.56 plus 20% delinquency interest per reduced amount of taxes in the revised assessment. RCBC’s subsequent
annum. The CIR prayed that RCBC be considered to have withdrawn its appeal with action effectively belies its insistence that the waivers are invalid. The records show

54
that on December 6, 2000, upon receipt of the revised assessment, RCBC law. His (agent) liability is direct and independent from the taxpayer,
immediately made payment on the uncontested taxes. Thus, RCBC is estopped from because the income tax is still imposed on and due from the latter. The
questioning the validity of the waivers. To hold otherwise and allow a party to agent is not liable for the tax as no wealth flowed into him – he earned no
gainsay its own act or deny rights which it had previously recognized would run income. The Tax Code only makes the agent personally liable for the tax arising
counter to the principle of equity which this institution holds dear. 31 from the breach of its legal duty to withhold as distinguished from its duty to pay
tax since:
Liability for Deficiency
Onshore Withholding Tax "the government’s cause of action against the withholding agent is not for
the collection of income tax, but for the enforcement of the withholding
provision of Section 53 of the Tax Code, compliance with which is imposed on
RCBC is convinced that it is the payor-borrower, as withholding agent, who is
the withholding agent and not upon the taxpayer."35 (Emphases supplied)
directly liable for the payment of onshore tax, citing Section 2.57(A) of Revenue
Regulations No. 2-98 which states:
Based on the foregoing, the liability of the withholding agent is independent from
that of the taxpayer.1âwphi1 The former cannot be made liable for the tax due
(A) Final Withholding Tax. — Under the final withholding tax system the amount of
because it is the latter who earned the income subject to withholding tax. The
income tax withheld by the withholding agent is constituted as a full and final
withholding agent is liable only insofar as he failed to perform his duty to withhold
payment of the income tax due from the payee on the said income. The liability
the tax and remit the same to the government. The liability for the tax, however,
for payment of the tax rests primarily on the payor as a withholding agent.
remains with the taxpayer because the gain was realized and received by him.
Thus, in case of his failure to withhold the tax or in case of under
withholding, the deficiency tax shall be collected from the
payor/withholding agent. The payee is not required to file an income tax return While the payor-borrower can be held accountable for its negligence in performing
for the particular income. (Emphasis supplied) its duty to withhold the amount of tax due on the transaction, RCBC, as the
taxpayer and the one which earned income on the transaction, remains liable for
the payment of tax as the taxpayer shares the responsibility of making certain that
The petitioner is mistaken.
the tax is properly withheld by the withholding agent, so as to avoid any penalty
that may arise from the non-payment of the withholding tax due.
Before any further discussion, it should be pointed out that RCBC erred in citing the
abovementioned Revenue Regulations No. 2-98 because the same governs
RCBC cannot evade its liability for FCDU Onshore Tax by shifting the blame on the
collection at source on income paid only on or after January 1, 1998. The deficiency
payor-borrower as the withholding agent. As such, it is liable for payment of
withholding tax subject of this petition was supposed to have been withheld on
deficiency onshore tax on interest income derived from foreign currency loans,
income paid during the taxable years of 1994 and 1995. Hence, Revenue
pursuant to Section 24(e)(3) of the National Internal Revenue Code of 1993:
Regulations No. 2-98 obviously does not apply in this case.

Sec. 24. Rates of tax on domestic corporations.


In Chamber of Real Estate and Builders’ Associations, Inc. v. The Executive
Secretary,32 the Court has explained that the purpose of the withholding tax system
is three-fold: (1) to provide the taxpayer with a convenient way of paying his tax (e) Tax on certain incomes derived by domestic corporations
liability; (2) to ensure the collection of tax, and (3) to improve the government’s
cashflow. Under the withholding tax system, the payor is the taxpayer upon whom
(3) Tax on income derived under the Expanded Foreign Currency Deposit System.
the tax is imposed, while the withholding agent simply acts as an agent or a
– Income derived by a depository bank under the expanded foreign currency
collector of the government to ensure the collection of taxes. 33 1avvphi1
deposit system from foreign currency transactions with nonresidents, offshore
banking units in the Philippines, local commercial banks including branches of
It is, therefore, indisputable that the withholding agent is merely a tax collector and foreign banks that may be authorized by the Central Bank to transact business with
not a taxpayer, as elucidated by this Court in the case of Commissioner of Internal foreign currency depository system units and other depository banks under the
Revenue v. Court of Appeals,34 to wit: expanded foreign currency deposit system shall be exempt from all taxes, except
taxable income from such transactions as may be specified by the Secretary of
Finance, upon recommendation of the Monetary Board to be subject to the usual
In the operation of the withholding tax system, the withholding agent is the payor,
income tax payable by banks: Provided, That interest income from foreign
a separate entity acting no more than an agent of the government for the collection
currency loans granted by such depository banks under said expanded
of the tax in order to ensure its payments; the payer is the taxpayer – he is the
system to residents (other than offshore banking units in the Philippines or
person subject to tax imposed by law; and the payee is the taxing authority. In
other depository banks under the expanded system) shall be subject to a
other words, the withholding agent is merely a tax collector, not a taxpayer. Under
10% tax. (Emphasis supplied)
the withholding system, however, the agent-payor becomes a payee by fiction of
55
As a final note, this Court has consistently held that findings and conclusions of the disallowance, the examiner alleging, among others, that petitioner failed to withhold
CTA shall be accorded the highest respect and shall be presumed valid, in the the appropriate tax thereon. This is also the same basis for the imposition of the
absence of any clear and convincing proof to the contrary.36 The CTA, as a deficiency withholding tax assessment on the management fees. Revenue
specialized court dedicated exclusively to the study and resolution of tax problems, Regulations No. 6-85 (EWT Regulations) does not impose or prescribe EWT on
has developed an expertise on the subject of taxation.37 As such, its decisions shall management fees paid to a non-resident.
not be lightly set aside on appeal, unless this Court finds that the questioned
decision is not supported by substantial evidence or there is a showing of abuse or
On November 7, 2001, nearly eight (8) years later, respondent’s external auditors
improvident exercise of authority on the part of the Tax Court. 38
received a letter from herein petitioner Commissioner of Internal Revenue dated
October 27, 2001. The letter advised the respondent that petitioner had rendered a
WHEREFORE, the petition is DENIED. final decision denying its protest on the ground that the protest against the disputed
tax assessment was allegedly filed beyond the 30-day reglementary period
prescribed in then Section 229 of the National Internal Revenue Code.
G.R. No. 169225               November 17, 2010
COMMISSIONER OF INTERNAL REVENUE, Petitioner,
vs. HAMBRECHT & QUIST PHILIPPINES, INC., Respondent. On December 6, 2001, respondent filed a Petition for Review docketed as CTA Case
No. 6362 before the then Court of Tax Appeals, pursuant to Section 7 of Republic
Act No. 1125, otherwise known as an ‘Act Creating the Court of Tax Appeals’ and
This is a Petition for Review on Certiorari under Rule 45 of the Rules of Court
Section 228 of the NIRC, to appeal the final decision of the Commissioner of
seeking to set aside the Decision1 dated August 12, 2005 of the Court of Tax
Internal Revenue denying its protest against the deficiency income and withholding
Appeals (CTA) En Banc in C.T.A. E.B. No. 73 (C.T.A. Case No. 6362), entitled
tax assessments issued for taxable year 1989.3
"Commissioner of Internal Revenue vs. Hambrecht & Quist Philippines, Inc.," which
affirmed the Decision2 dated September 24, 2004 of the CTA Original Division in
C.T.A. Case No. 6362 canceling the assessment issued against respondent for In a Decision dated September 24, 2004, the CTA Original Division held that the
deficiency income and expanded withholding tax for the year 1989 for failure of subject assessment notice sent by registered mail on January 8, 1993 to
petitioner Commissioner of Internal Revenue (CIR) to enforce collection within the respondent’s former place of business was valid and binding since respondent only
period allowed by law. gave formal notice of its change of address on February 18, 1993. Thus, the
assessment had become final and unappealable for failure of respondent to file a
protest within the 30-day period provided by law. However, the CTA (a) held that
The CTA summarized the pertinent facts of this case, as follows:
the CIR failed to collect the assessed taxes within the prescriptive period; and (b)
directed the cancellation and withdrawal of Assessment Notice No. 001543-89-
In a letter dated February 15, 1993, respondent informed the Bureau of Internal 5668. Petitioner’s Motion for Reconsideration and Supplemental Motion for
Revenue (BIR), through its West-Makati District Office of its change of business Reconsideration of said Decision filed on October 14, 2004 and November 22, 2004,
address from the 2nd Floor Corinthian Plaza, Paseo de Roxas, Makati City to the respectively, were denied for lack of merit.
22nd Floor PCIB Tower II, Makati Avenue corner H.V. De la Costa Streets, Makati
City. Said letter was duly received by the BIR-West Makati on February 18, 1993.
Undaunted, the CIR filed a Petition for Review with the CTA En Banc but this was
denied in a Decision dated August 12, 2005, the dispositive portion reads:
On November 4, 1993, respondent received a tracer letter or follow-up letter dated
October 11, 1993 issued by the Accounts Receivable/Billing Division of the BIR’s
WHEREFORE, the Petition for Review is DENIED DUE COURSE and the case is
National Office and signed by then Assistant Chief Mr. Manuel B. Mina, demanding
accordingly DISMISSED for lack of merit.4
for payment of alleged deficiency income and expanded withholding taxes for the
taxable year 1989 amounting to ₱2,936,560.87.
Hence, the instant Petition wherein the following issues are raised:
On December 3, 1993, respondent, through its external auditors, filed with the
same Accounts Receivable/Billing Division of the BIR’s National Office, its protest I
letter against the alleged deficiency tax assessments for 1989 as indicated in the
said tracer letter dated October 11, 1993.
WHETHER OR NOT THE COURT OF TAX APPEALS HAS JURISDICTION TO
RULE THAT THE GOVERNMENT’S RIGHT TO COLLECT THE TAX HAS
The alleged deficiency income tax assessment apparently resulted from an PRESCRIBED.
adjustment made to respondent’s taxable income for the year 1989, on account of
the disallowance of certain items of expense, namely, professional fees paid,
II
donations, repairs and maintenance, salaries and wages, and management fees.
The latter item of expense, the management fees, made up the bulk of the
56
WHETHER OR NOT THE PERIOD TO COLLECT THE ASSESSMENT HAS Any internal revenue tax which has been assessed within the period of limitation
PRESCRIBED.5 above-prescribed may be collected by distraint or levy or by a proceeding in
court within three years following the assessment of the tax. (Emphases
supplied.)
The petition is without merit.

In connection therewith, Section 3 of the 1986 NIRC states that the collection of
Anent the first issue, petitioner argues that the CTA had no jurisdiction over the
taxes is one of the duties of the BIR, to wit:
case since the CTA itself had ruled that the assessment had become final and
unappealable. Citing Protector’s Services, Inc. v. Court of Appeals, 6 the CIR argued
that, after the lapse of the 30-day period to protest, respondent may no longer Sec. 3. Powers and duties of Bureau. - The powers and duties of the Bureau of
dispute the correctness of the assessment and its appeal to the CTA should be Internal Revenue shall comprehend the assessment and collection of all national
dismissed. The CIR took issue with the CTA’s pronouncement that it had jurisdiction internal revenue taxes, fees, and charges and the enforcement of all
to decide "other matters" related to the tax assessment such as the issue on the forfeitures, penalties, and fines connected therewith including the execution of
right to collect the same since the CIR maintains that when the law says that the judgments in all cases decided in its favor by the Court of Tax Appeals and the
CTA has jurisdiction over "other matters," it presupposes that the tax assessment ordinary courts. Said Bureau shall also give effect to and administer the supervisory
has not become final and unappealable. and police power conferred to it by this Code or other laws. (Emphasis supplied.)

We cannot countenance the CIR’s assertion with regard to this point. The Thus, from the foregoing, the issue of prescription of the BIR’s right to collect taxes
jurisdiction of the CTA is governed by Section 7 of Republic Act No. 1125, as may be considered as covered by the term "other matters" over which the CTA has
amended, and the term "other matters" referred to by the CIR in its argument can appellate jurisdiction.
be found in number (1) of the aforementioned provision, to wit:
Furthermore, the phraseology of Section 7, number (1), denotes an intent to view
Section 7. Jurisdiction. - The Court of Tax Appeals shall exercise exclusive appellate the CTA’s jurisdiction over disputed assessments and over "other matters" arising
jurisdiction to review by appeal, as herein provided – under the NIRC or other laws administered by the BIR as separate and independent
of each other. This runs counter to petitioner’s theory that the latter is qualified by
the status of the former, i.e., an "other matter" must not be a final and
1. Decisions of the Commissioner of Internal Revenue in cases involving disputed
unappealable tax assessment or, alternatively, must be a disputed assessment.
assessments, refunds of internal revenue taxes, fees or other charges, penalties
imposed in relation thereto, or other matters arising under the National
Internal Revenue Code or other law as part of law administered by the Likewise, the first paragraph of Section 11 of Republic Act No. 1125,
Bureau of Internal Revenue. (Emphasis supplied.)
as amended by Republic Act No. 9282,8 belies petitioner’s assertion as the provision
Plainly, the assailed CTA En Banc Decision was correct in declaring that there was is explicit that, for as long as a party is adversely affected by any decision, ruling or
nothing in the foregoing provision upon which petitioner’s theory with regard to the inaction of petitioner, said party may file an appeal with the CTA within 30 days
parameters of the term "other matters" can be supported or even deduced. What is from receipt of such decision or ruling. The wording of the provision does not take
rather clearly apparent, however, is that the term "other matters" is limited only by into account the CIR’s restrictive interpretation as it clearly provides that the mere
the qualifying phrase that follows it. existence of an adverse decision, ruling or inaction along with the timely filing of an
appeal operates to validate the exercise of jurisdiction by the CTA.
Thus, on the strength of such observation, we have previously ruled that the
appellate jurisdiction of the CTA is not limited to cases which involve decisions of To be sure, the fact that an assessment has become final for failure of the taxpayer
the CIR on matters relating to assessments or refunds. The second part of the to file a protest within the time allowed only means that the validity or correctness
provision covers other cases that arise out of the National Internal Revenue Code of the assessment may no longer be questioned on appeal. However, the validity of
(NIRC) or related laws administered by the Bureau of Internal Revenue (BIR). 7 the assessment itself is a separate and distinct issue from the issue of whether the
right of the CIR to collect the validly assessed tax has prescribed. This issue of
prescription, being a matter provided for by the NIRC, is well within the jurisdiction
In the case at bar, the issue at hand is whether or not the BIR’s right to collect
of the CTA to decide.
taxes had already prescribed and that is a subject matter falling under Section
223(c) of the 1986 NIRC, the law applicable at the time the disputed assessment
was made. To quote Section 223(c): With respect to the second issue, the CIR insists that its right to collect the tax
deficiency it assessed on respondent is not barred by prescription since the
prescriptive period thereof was allegedly suspended by respondent’s request for
reinvestigation.
57
Based on the facts of this case, we find that the CIR’s contention is without which, by the very nature of its functions, is dedicated exclusively to the resolution
basis.1avvphi1 The pertinent provision of the 1986 NIRC is Section 224, to wit: of tax problems and has accordingly developed an expertise on the subject unless
there has been an abuse or improvident exercise of authority. 11
Section 224. Suspension of running of statute. – The running of the statute of
limitations provided in Sections 203 and 223 on the making of assessment and the Indeed, it is contradictory for the CIR to argue that respondent’s December 3, 1993
beginning of distraint or levy or a proceeding in court for collection, in respect of protest which contained a request for reinvestigation was filed beyond the
any deficiency, shall be suspended for the period during which the Commissioner is reglementary period but still claim that the same request for reinvestigation was
prohibited from making the assessment or beginning distraint or levy or a implicitly granted by virtue of its October 27, 2001 letter. We find no cogent reason
proceeding in court and for sixty days thereafter; when the taxpayer requests to reverse the CTA when it ruled that the prescriptive period for the CIR’s right to
for a re-investigation which is granted by the Commissioner; when the collect was not suspended under the circumstances of this case.
taxpayer cannot be located in the address given by him in the return filed upon
which a tax is being assessed or collected: Provided, That, if the taxpayer informs
WHEREFORE, the petition is DENIED. The assailed Decision of the Court of Tax
the Commissioner of any change in address, the statute will not be suspended;
Appeals (CTA) En Banc dated August 12, 2005 is AFFIRMED. No costs.
when the warrant of distraint and levy is duly served upon the taxpayer, his
authorized representative, or a member of his household with sufficient discretion,
and no property could be located; and when the taxpayer is out of the Philippines. G.R. No. L-11527           November 25, 1958
(Emphasis supplied.) THE COLLECTOR OF INTERNAL REVENUE, petitioner,
vs. SUYOC CONSOLIDATED MINING COMPANY, ET AL., respondents.
The plain and unambiguous wording of the said provision dictates that two
requisites must concur before the period to enforce collection may be suspended: Suyoc Consolidated Mining Company, a mining corporation operating before the
(a) that the taxpayer requests for reinvestigation, and (b) that petitioner grants war, was unable to file in 1942 its income tax return for the year 1941 due to the
such request. last war. After liberation, Congress enacted Commonwealth Act No. 722 which
extended the filing of tax returns for 1941 up to December 31, 1945. Its records
having been lost or destroyed, the company requested the Collector of Internal
On this point, we have previously held that:
Revenue to grant it an extension of time to file its return, which was granted until
February 15, 1946, and the company was authorized to file its return for 1941 on
The above section is plainly worded. In order to suspend the running of the the basis of the best evidence obtainable.
prescriptive periods for assessment and collection, the request for
reinvestigation must be granted by the CIR.9 (Emphasis supplied.)
The company filed three income tax returns for the calendar year ending December
31, 1941. On February 12, 1946, it filed a tentative return as it had not yet
Consequently, the mere filing of a protest letter which is not granted does not completely reconstructed its records. On November 28, 1946, it filed a second final
operate to suspend the running of the period to collect taxes. In the case at bar, the return on the basis of the records it has been able to reconstruct at that time. On
records show that respondent filed a request for reinvestigation on December 3, February 6, 1947, it filed its third amended final return on the basis of the available
1993, however, there is no indication that petitioner acted upon respondent’s records which to that date it had been able to reconstruct.
protest. As the CTA Original Division in C.T.A. Case No. 6362 succinctly pointed out
in its Decision, to wit:
On the basis of the second final return filed by the company on November 28, 1946,
the Collector assessed against it the sum of P28,289.96 as income tax for 1941,
It is evident that the respondent did not conduct a reinvestigation, the protest plus P1,414.50 as 5 per cent surcharge and P3,894.80 as 1 per cent monthly
having been dismissed on the ground that the assessment has become final and interest from March 1, 1946 to February 28, 1947, or a total of P33,099.26. The
executory. There is nothing in the record that would show what action was taken in assessment was made on February 11, 1947. On February 21, 1947, the company
connection with the protest of the petitioner. In fact, petitioner did not hear asked for an extension of at least one year from February 28, 1947 within which to
anything from the respondent nor received any communication from the respondent pay the amount assessed, reserving its right to question the correctness of the
relative to its protest, not until eight years later when the final decision of the assessment. The Collector granted an extension of only three months from March
Commissioner was issued (TSN, March 7, 2002, p. 24). In other words, the 20, 1947.
request for reinvestigation was not granted. x x x.10 (Emphasis supplied.)
The company failed to pay the tax within the period granted to it and so the
Since the CIR failed to disprove the aforementioned findings of fact of the CTA Collector sent to it a letter on November 28, 1950 demanding payment of the tax
which are borne by substantial evidence on record, this Court is constrained to due as assessed, plus surcharge and interest up to December 31, 1950. On April 6,
uphold them as binding and true. This is in consonance with our oft-cited ruling that 1951, the company asked for a reconsideration and reinvestigation of the
instructs this Court to not lightly set aside the conclusions reached by the CTA, assessment, which was granted, the case being assigned to another examiner, but

58
the Collector made another assessment against the company in the sum of reinvestigations were made and a hearing was even held by the Conference Staff
P33,829.66. This new assessment was made on March 7, 1952. On April 18, 1952, organized in the collection office to consider claims of such nature which, as the
the Collector revised this last assessment and required the company to pay the sum record shows, lasted for several months. After inducing petitioner to delay collection
of P28,289.96 as income tax, P1,414.50 as surcharge, P20,934.57 as interest up to as he in fact did, it is most unfair for respondent to now take advantage of such
April 30, 1952 and P40 as compromise. desistance to elude his deficiency income, tax liability to the prejudice of the
Government invoking the technical ground of prescription.
After several other negotiations conducted at the request of respondent, including
an appeal to the Conference Staff created to act on such matters in the Bureau of While we may agree with the Court of Tax Appeals that a mere request for
Internal Revenue, the assessment was finally reduced by the Collector to reexamination or reinvestigation may not have the effect of suspending the running
P24,438.96, without surcharge and interest, and of this new assessment the of the period of limitation for in such case there is need of a written agreement to
company was notified on July 28, 1955. Within the reglementary period, the extend the period between the Collector and the taxpayer, there are cases however
company filed with the Court of Tax Appeals a petition for review of this assessment where a taxpayer may be prevented from setting up the defense of prescription
made on July 26, 1955 on the main ground that the right of the Government to even if he has not previously waived it in writing as when by his repeated requests
collect the tax has already prescribed. After the case was heard, the court rendered or positive acts the Government has been, for good reasons, persuaded to postpone
its decision upholding this defense and, accordingly, it set aside the ruling of the collection to make him feel that the demand was not unreasonable or that no
Collector of Internal Revenue. The Collector interposed the present petition for harassment or injustice is meant by the Government. And when such situation
review. comes to pass there are authorities that hold, based on weighty reasons, that such
an attitude or behavior should not be countenanced if only to protect the interest of
the Government.
Under the law, an internal revenue tax shall be assessed within five years after the
return is filed by the taxpayer and no proceeding in court for its collection shall be
begun after the expiration of such period (Section 331, National Internal Revenue This case has no precedent in this jurisdiction for it is the first time that such has
Code). The law also provides that where an assessment of internal revenue tax is risen, but there are several precedents that may be invoked in American
made within the above period, such tax may be collected by distraint or levy or by a jurisprudence. As Mr. Justice Cardozo has said: "The applicable principle is
proceeding in court but only if the same is begun (1) within five years after fundamental and unquestioned. 'He who prevents a thing from being done may not
assessment or (2) within the period that may be agreed upon in writing between avail himself of the nonperformance which he has himself occasioned, for the law
the Collector and the taxpayer before the expiration of the 5-year period [Section says to him in effect "this is your own act, and therefore you are not damnified." '
332 (c), Idem.]. "(R. H. Stearns Co. vs. U.S., 78 L. ed., 647). Or, as was aptly said, "The tax could
have been collected, but the government withheld action at the specific request of
the plaintiff. The plaintiff is now estopped and should not be permitted to raise the
It appears that the first assessment made against respondent based on its second
defense of the Statute of Limitations." [Newport Co. vs. U.S., (DC-WIS), 34 F.
final return filed on November 28, 1946 was made on February 11, 1947. Upon
Supp. 588].
receipt of this assessment respondent requested for at least one year within which
to pay the amount assessed although it reserved its right to question the
correctness of the assessment before actual payment. Petitioner granted an The following authorities cited in the brief of the Solicitor General are in point:
extension of only three months. When it failed to pay the tax within the period
extended, petitioner sent respondent a letter on November 28, 1950 demanding
The petitioner makes the point that by the Revenue Act of May 29, 1928 (chap.
payment of the tax as assessed, and upon receipt of the letter respondent asked for
852, 45 Stat. at L. 791, 875, sec. 609, U.S.C. title 26, sec. 2609), a credit against a
a reinvestigation and reconsideration of the assessment. When this request was
liability in respect of any taxable year shall be "void" if it has been made against a
denied, respondent again requested for a reconsideration on April 25, 1952, which
liability barred by limitation. The aim of that provision, as we view it, was to
was denied on May 6, 1953, which denial was appealed to the Conference Staff. The
invalidate such a credit if made by the Commissioner of his own motion without the
appeal was heard by the Conference Staff from September 2, 1953 to July 16,
taxpayer's approval or with approval failing short of inducement or request. Cf.
1955, and as a result of these various negotiations, the assessment was finally
Stange vs. United States, 282 U. S. 270, 75 L. ed. 335, 51 S. Ct. 145, supra;
reduced on July 26, 1955. This is the ruling which is now being questioned after a
Revenue Act of 1928, sec. 506 (b) (c), chap. 852, 45 Stat. at L. 791, 870, 871,
protracted negotiation on the ground that the collection of the tax has already
U.S.C. title 26, see. 1062a. If nothing more than this appeared, there was to be no
prescribed.
exercise in invitum of governmental power. But the aim of the statute suggests a
restraint upon its meaning. To know whether liability has been barred by limitation
It is obvious from the foregoing that petitioner refrained from collecting the tax by it will not do to refer to the flight of time alone. The limitation may have been
distraint or levy or by proceeding in court within the 5-year period from the filing of postponed by force of a simple waiver, which must then be made in adherence to
the second amended final return due to the several requests of respondent for the statutory forms, or so we now assume. It may have been postponed by
extension to which petitioner yielded to give it every opportunity to prove its claim deliberate persuasion to withhold official action. We think it an unreasonable
regarding the correctness of the assessment. Because of such requests, several construction that would view the prohibition of the statute as over-riding the

59
doctrine of estoppel (Randon vs. Tobey, 11 How. 493, 519, 13 L. ed. 784, 795) and G.R. No. L-21609             September 29, 1966
invalidating a credit made at the taxpayer's request. Here at the time of the REPUBLIC OF THE PHILIPPINES, plaintiff-appellant,
request, the liability was still alive, unaffected as yet by any statutory bar. The vs. KER & COMPANY, LTD., defendant-appellant.
request in its fair meaning reached forward into the future and prayed for the
postponement of collection till the audits for later years had been completed in the
Ker & Co., Ltd., a domestic corporation, filed its income tax returns for the years
usual course. This having been done, the suspended collection might be effected by
1947, 1948, 1949 and 1950 on the following dates:
credit or by distraint or by other methods prescribed by law. Congress surely did not
mean that a credit was to be void if made by the Government in response to such
prayer. Year Date Filed
1947 April 12, 1948
The applicable principle is fundamental and unquestioned. "He who prevents a thing 1948 April 30, 1949
from being done may not avail himself of the nonperformance which he has himself 1949 May 15, 1950
occasioned, for the law says to him in effect "this is your own act, and therefore you 1950 May 9, 1951
are not damnified," ' " Dolan vs. Rogers, 149 N. Y. 489, 491, 44 N.E. 167, and
Imperator Realty Co. vs. Tull, 228 N. Y. 447, 457, 127 N.E. 263, quoting West vs.
Blakeway, 2 Mann. & G. 729, 751, 133 Eng. Reprint, 940, 949. Sometimes the It amended its income tax returns for 1948 and 1949 on May 11, 1949 and June 30,
resulting disability has been characterized as an estoppel, sometimes as a waiver. 1950, respectively.
The label counts for little. Enough for present purposes that the disability has its
roots in a principle more nearly ultimate than either waiver or estoppel, the principle In 1953 the Bureau of Internal Revenue examined and audited Ker & Co., Ltd.'s
that no one shall be permitted to found any claim upon his own inequity or take returns and books of accounts and subsequently issued the following assessments
advantage of his own wrong. Imperator Realty Co. vs. Tull, 228 N.Y. 447, 127 N.E. for deficiency income tax:
263, supra. A suit may not be built on an omission induced by him who sues. Swain
vs. Seamens, 9 Wall. 254, 274, 19 L. ed. 554, 560; United States vs. Peck, 102
Year Amount Date Assessed
U.S. 64, 26 L. ed. 46; Thomson vs. Poor, 147 N.Y. 402, 42 N.E. 13; New Zealand
Shipping Co. vs. Societe des Ateliers (1919) A. C. 1, 6-H. L.; 2 Williston, Contr. sec. 1947 P42,342.30 July 25, 1953
689. (R. H. Stearns Co. vs. U.S., supra; Emphasis supplied.) 1948 18,651.87 Feb. 16, 1953
1949 139.67 Feb. 16, 1953
. . . It is admitted that these assessments were timely made in August 1923. Upon 1950 12,813.00 Feb. 16, 1953
the making of the assessment the Commissioner sought to make collection, which
likewise was at a time when the statute had not ran on collection, but the due and payable on dates indicated in the accompanying notices of assessment. The
authorized representative of the Lattimores strenuously objected to the collection assessments for 1948 and 1950 carried the surcharge of 50% authorized under
and urged the Commissioner to withhold collection, pending adjustment of the Section 72 of the Tax Code for the filing of fraudulent returns.
controversy between them and the Commissioner. The Commissioner yielded to
their request and postponed collection until August 19, 1926, which was after the
statute had run on collection. In the meantime, further claims for refund and Upon request of Ker & Co., Ltd., through Atty. Jose Leido, its counsel, the Bureau of
protests were filed, conferences were held and consideration was given to the Internal Revenue reduced the assessments for the year 1947 from P42,342.30 to
settlement of the controversy, and the matter was not finally disposed of until 1926, P27,026.28 and for the year 1950 from P12,813.00 to P8,542.00, imposed the 50%
when the statute had run on collection. The procedure carried out was that surcharge for the year 1947 and eliminated the same surcharge from the
requested by plaintiffs, and they cannot now be heard to say that the collection was assessment for the year 1950. The assessments for years 1948 and 1949 remained
not timely. R. H. Stearns Company vs. United States, 291 U.S. 54, 54 S. Ct. 325, the same.
78 L. Ed. 647. (Lattimore vs. U.S., 12 F. Supp. 895, 91.)
On March 1, 1956 Ker & Co., Ltd. filed with the Court of Tax Appeals a petition for
Wherefore, the decision appealed from is reversed. review with preliminary injunction. No preliminary injunction was issued, for said
court dismissed the appeal for having been instituted beyond the 30-day period
provided for in Section 11 of Republic Act 1125. We affirmed the order of dismissal
The decision of the Collector of Internal Revenue rendered on July 26, 1955 is of L-12396. 1
hereby affirmed. No costs.

On March 15, 1962, the Bureau of Internal Revenue demanded payment of the
Paras, C. J., Bengzon, Labrador, Concepcion, Reyes, J. B. L. and Endencia, aforesaid assessments together with a surcharge of 5% for late payment and
JJ., concur. interest at the rate of 1% monthly. Ker & Co., Ltd. refused to pay, instead in its
letters dated March 28, 1962 and April 10, 1962 it set up the defense of prescription
60
of the Commissioner's right to collect the tax. Subsequently, the Republic of the person, and maintaining that since the complaint was filed nine years, one month
Philippines filed on March 27, 1962 a complaint with the Court of First Instance of and eleven days after the deficiency assessments for 1948, 1949 and 1950 were
Manila seeking collection of the aforesaid deficiency income tax for the years 1947, made and since the filing of its petition for review in the Court of Tax Appeals did
1948, 1949 and 1950. The complaint did not allege fraud in the filing of any of the not stop the running of the period of limitations, the right of the Commissioner of
income tax returns for the years involved, nor did it pray for the payment of the Internal Revenue to collect the tax in question has prescribed.
corresponding 50% surcharge, but it prayed for the payment of 5% surcharge for
late payment and interest of 1% per month without however specifying from what
The two motions for reconsideration having been denied, both parties appealed
date interest started to accrue.
directly to this Court.

Summons was served not on the defendant taxpayer but upon Messrs. Leido and
The issues in this case are:
Associates, its counsel in the proceedings before the Bureau of Internal Revenue
and the Court of Tax Appeals.
1. Did the Court of First Instance acquire jurisdiction over the person of
defendant Ker & Co., Ltd.? .
On April 14, 1962 Ker & Co., Ltd. through its counsel, Leido, Andrada, Perez &
Associates, moved for the dismissal of the complaint on the ground that the court
did not acquire jurisdiction over the person of the defendant and that plaintiff's 2. Did the right of the Commissioner of Internal Revenue to assess
cause of action has prescribed. This motion was denied and defendant filed a motion deficiency income tax for the year 1947 prescribe? .
for reconsideration. Resolution on said motion, however, was deferred until trial of
the case on the merits. 3. Did the filing of a petition for review by the taxpayer in the Court of Tax
Appeals suspend the running of the statute of limitations to collect the
On May 18, 1962, Ker & Co., Ltd. filed its answer to the complaint interposing deficiency income for the years 1948, 1949 and 1950?
therein the defense set up in its motion to dismiss of April 14, 1962.
4. When did the delinquency interest on the deficiency income tax for the
On September 18, 1962 the Republic of the Philippines amended its complaint, in years 1948, 1949 and 1950 accrue?
answer to which Ker & Co., Ltd. adopted the same answer which it had filed on May
18, 1962. First Issue

On January 30, 1963 the Court of First Instance rendered judgment, the dispositive Ker & Co., Ltd. maintains that the court a quo did not acquire jurisdiction over its
portion of which states: person inasmuch as summons was not served upon it but upon Messrs. Leido and
Associates who do not come under any of the class of persons upon whom
WHEREFORE, this Court dismisses the claim for the collection of deficiency summons should be served as enumerated in Section 13, Rule 7 of the Rules of
income taxes for 1947, but orders defendant taxpayer to pay the deficiency Court, 2 which reads:
income taxes for 1948, 1949 and 1950, in the amounts of P18,651.87,
P139.67 and P8,542.00, respectively, plus 5% surcharge thereon on each SEC. 13. Service upon private domestic corporation or partnership.—If the
amount and interest of 1% a month computed from March 27, 1962 and defendant is a corporation formed under the laws of the Philippines or a
until full payment thereof is made, plus the costs of suit. partnership duly registered, service may be made on the president,
manager, secretary, cashier, agent, or any of its directors.
On February 20, 1963 the Republic of the Philippines filed a motion for
reconsideration contending that the right of the Commissioner of Internal Revenue Messrs. Leido and Associates acted as counsel for Ker Co., Ltd. when this tax case
to collect the deficiency assessment for 1947 has not prescribed by a lapse of was in its administrative stage. The same counsel represented Ker & Co., Ltd., when
merely five years and three months, because the taxpayer's income tax return was it appealed said case to the Court of Tax Appeals and later to this Court.
fraudulent in which case prescription sets in ten years from October 31, 1951, the Subsequently, when the Deputy Commissioner of Internal Revenue, by letter dated
date of discovery of the fraud, pursuant to Section 332 (a) of the Tax Codes and March 15, 1962, demanded the payment of the deficiency income tax in question, it
that the payment of delinquency interest of 1% per month should commence from was Messrs. Leido, Andrada, Perez & Associates who replied in behalf of Ker & Co.,
the date it fell due as indicated in the assessment notices instead of on the date the Ltd. in two letters, dated March 28, 1962 and April 10, 1962, both after the
complaint was filed. complaint in this case was filed. At least therefore on April 2, 1962 when Messrs.
Leido and Associates received the summons, they were still acting for and in behalf
On March 6, 1963 Ker & Co., Ltd. also filed a motion for reconsideration reiterating of Ker & Co., Ltd. in connection with its tax liability involved in this case. Perforce,
its assertion that the Court of First Instance did not acquire jurisdiction over its
61
they were the taxpayer's agent when summons was served. Under Section 13 of the provisions of mandatory or prohibitory laws shall be void (Art. 5, New
Rule 7, aforequoted, service upon the agent of a corporation is sufficient. Civil Code). . . .

We observe that the motion to dismiss filed on April 14, 1962, aside from disputing Said court resolved the issue without touching upon fraudulence of the return. The
the lower court's jurisdiction over defendant's person, prayed for dismissal of the reason is that the complaint alleged no fraud, nor did the plaintiff present evidence
complaint on the ground that plaintiff's cause of action has prescribed. By to prove fraud.
interposing such second ground in its motion to dismiss, Ker & Co., Ltd. availed of
an affirmative defense on the basis of which it prayed the court to resolve
In reply to the lower court's conclusion, the Republic of the Philippines maintains in
controversy in its favor. For the court to validly decide the said plea of defendant
its brief that Ker & Co., Ltd. filed a false return and since the fraud penalty of 50%
Ker & Co., Ltd., it necessarily had to acquire jurisdiction upon the latter's person,
surcharge was imposed in the deficiency income tax assessment, which has become
who, being the proponent of the affirmative defense, should be deemed to have
final and executory, the finding of the Commissioner of Internal Revenue as to the
abandoned its special appearance and voluntarily submitted itself to the jurisdiction
existence of the fraud has also become final and need not be proved. This
of the court.3
contention suffers from a flaw in that it fails to consider the well-settled principle
that fraud is a question of fact6 which must be alleged and proved.7 Fraud is a
Voluntary appearance cures defects of summons, if any. 4 Such defect, if any, was serious charge and, to be sustained, it must be supported by clear and convincing
further cured when defendant filed its answer to the complaint. 5 A defendant can proof.8 Accordingly, fraud should have been alleged and proved in the lower court.
not be permitted to speculate upon the judgment of the court by objecting to the On these premises We therefore sustain the ruling of the lower court upon the point
court's jurisdiction over its person if the judgment is adverse to it, and acceding to of prescription.
jurisdiction over its person if and when the judgment sustains its defenses.
It would be worth mentioning that since the assessment for deficiency income tax
Second Issue for 1947 has become final and executory, Ker & Co., Ltd. may not anymore raise
defenses which go into the merits of the assessment, i.e., prescription of the
Commissioner's right to assess the tax. Such was our ruling in previous cases. 9 In
Ker & Co., Ltd. contends that under Section 331 of the Tax Code the right of the
this case however, Ker & Co., Ltd. raised the defense of prescription in the
Commissioner of Internal Revenue to assess against it a deficiency income tax for
proceedings below and the Republic of the Philippines, instead of questioning the
the year 1947 has prescribed because the assessment was issued on July 25, 1953
right of the defendant to raise such defense, litigated on it and submitted the issue
after a lapse of five years, three months and thirteen days from the date (April 12,
for resolution of the court. By its actuation, the Republic of the Philippines should be
1948) it filed its income tax return. On the other hand, the Republic of the
considered to have waived its right to object to the setting up of such defense.
Philippines insists that the taxpayer's income tax return was fraudulent, therefore
the Commissioner of Internal Revenue may assess the tax within ten years from
discovery of the fraud on October 31, 1951 pursuant to Section 322(a) of the Tax Third Issue
Code.
Ker & Co., Ltd. impresses upon Us that since the Republic of the Philippines filed the
The stand of the Republic of the Philippines hinges on whether or not taxpayer's complaint for the collection of the deficiency income tax for the years 1948, 1949
income tax return for 1947 was fraudulent. and 1950 only on March 27, 1962, or nine years, one month and eleven days from
February 16, 1953, the date the tax was assessed, the right to collect the same has
prescribed pursuant to Section 332 (c) of the Tax Code. The Republic of the
The court a quo, confining itself to determining whether or not the assessment of
Philippines however contends that the running of the prescriptive period was
the tax for 1947 was issued within the five-year period provided for in Section 331
interrupted by the filing of the taxpayer's petition for review in the Court of Tax
of the Tax Code, ruled that the right of the Commissioner of Internal Revenue to
Appeals on March 1, 1956.
assess the tax has prescribed. Said the lower court:

If the period during which the case was pending in the Court of Tax Appeals and in
The Court resolves the second issue in the negative, because Section 331
the Supreme Court were not counted in reckoning the prescriptive period, less than
of the Revenue Code explicitly provides, in mandatory terms, that "Internal
five years would have elapsed, hence, the right to collect the tax has not
Revenue taxes shall be assessed within 5 years after the return was filed,
prescribed.
and no proceedings in court without assessment, for the collection of such
taxes, shall be begun after expiration of such period. The attempt by the
Commissioner of Internal Revenue to make an assessment on July 25, The taxpayer counters that the filing of the petition for review in the Court of Tax
1953, on the basis of a return filed on April 12, 1948, is an exercise of Appeals could not have stopped the running of the prescriptive period to collect
authority against the aforequoted explicit and mandatory limitations of because said court did not have jurisdiction over the case, the appeal having been
statutory law. Settled in our system is the rule that acts committed against interposed beyond the 30-day period set forth in Section 11 of Republic Act 1125.

62
Precisely, it adds, the Tax Court dismissed the appeal for lack of jurisdiction and Ltd. appealed to the Supreme Court. By the time the Supreme Court affirmed the
said dismissal was affirmed by the Supreme Court in L-12396 aforementioned. order of dismissal of the Court of Tax Appeals in L-12396 on January 31, 1962 more
than five years had elapsed since the final assessments were made on January 5,
1954. Thereafter, the Commissioner of Internal Revenue demanded extra-judicially
Under Section 333 of the Tax Code, quoted hereunder:
the payment of the deficiency tax in question and in reply the taxpayer, by its letter
dated March 28, 1962, advised the Commissioner of Internal Revenue that the right
SEC. 333. Suspension of running of statute.—The running of the statute of to collect the tax has prescribed pursuant to Section 332 (c) of the Tax
limitations provided in Section 331 or three hundred thirty-two on the Code.1awphîl.nèt
making of assessments and the beginning, of distraint or levy or a
proceeding in court for collection, in respect of any deficiency, shall be
Thus, did the taxpayer produce the effect of temporarily staying the hands of the
suspended for the period during which the Collector of Internal Revenue is
Commissioner of Internal Revenue simply through a choice of remedy. And, if We
prohibited from making the assessment or beginning distraint or levy or a
were to sustain the taxpayer's stand, We would be encouraging taxpayers to delay
proceeding in court, and for sixty days thereafter.
the payment of taxes in the hope of ultimately avoiding the same.

the running of the prescriptive period to collect the tax shall be suspended for the
Under the circumstances, the Commissioner of Internal Revenue was in effect
period during which the Commissioner of Internal Revenue is prohibited from
prohibited from collecting the tax in question. This being so, the provisions of
beginning a distraint and levy or instituting a proceeding in court, and for sixty days
Section 333 of the Tax Code will apply.
thereafter.

Fourth Issue
Did the pendency of the taxpayer's appeal in the Court of Tax Appeals and in the
Supreme Court have the effect of legally preventing the Commissioner of Internal
Revenue from instituting an action in the Court of First Instance for the collection of The Republic of the Philippines maintains that the delinquency interest on the
the tax? Our view is that it did. deficiency income tax for 1948, 1949 and 1950 accrued and should commence from
the date of the assessments as shown in the assessment notices, pursuant to
Section 51(e) of the Tax Code, instead of from the date the complaint was filed as
From March 1, 1956 when Ker & Co., Ltd. filed a petition for review in the Court of
determined in the decision appealed from.
Tax Appeals contesting the legality of the assessments in question, until the
termination of its appeal in the Supreme Court, the Commissioner of Internal
Revenue was prevented, as recognized in this Court's ruling in Ledesma, et al. v. Section 51 (e) of the Tax Code states:
Court of Tax Appeals, 10 from filing an ordinary action in the Court of First Instance
to collect the tax. Besides, to do so would be to violate the judicial policy of avoiding
SEC. 51(e). Surcharge and interest in case of delinquency.—To any sum or
multiplicity of suits and the rule on lis pendens. 11
sums due and unpaid after the dates prescribed in subsections (b), (c) and
(d) for the payment of the same, there shall be added the sum of five per
It would be interesting to note that when the Commissioner of Internal Revenue centum on the amount of tax unpaid and interest at the rate of one per
issued the final deficiency assessments on January 5, 1954, he had already lost, by centum a month upon said tax from the time the same became due, except
prescription, the right to collect the tax (except that for 1950) by the summary from the estates of insane, deceased, or insolvent persons. (emphasis
method of warrant of distraint and levy. Ker & Co., Ltd. immediately thereafter supplied)
requested suspension of the collection of the tax without penalty incident to late
payment pending the filing of a memorandum in support of its views. As requested,
Exhibit "F" — the letter of assessment — shows that the deficiency income tax for
no tax was collected. On May 22, 1954 the projected memorandum was filed, but as
1948 and 1949 became due on March 15, 1953 and that for 1950 accrued on
of that date the Commissioner's right to collect by warrant of distraint and levy the
February 15, 1954 in accordance with Section 51(d) of the Tax Code. Since the tax
deficiency tax for 1950 had already prescribed. So much so, that on March 1, 1956
in question remained unpaid, delinquency interest accrued and became due starting
when Ker & Co., Ltd. filed a petition for review in the Court of Tax Appeals, the
from said due dates. The decision appealed from should therefore be modified
Commissioner of Internal Revenue had but one remedy left to collect the tax, that
accordingly.
is, by judicial action. 12 However, as stated, an independent ordinary action in the
Court of First Instance was not available to the Commissioner pursuant to Our ruling
in Ledesma, et al. v. Court of Tax Appeals, supra, in view of the pendency of the WHEREFORE, the decision appealed from is affirmed with the modification that the
taxpayer's petition for review in the Court of Tax Appeals. Precisely he urgently filed delinquency interest at the rate of 1% per month shall be computed from March 15,
a motion to dismiss the taxpayer's petition for review with a view to terminating 1953 for the deficiency income tax for 1948 and 1949 and from February 15, 1954
therein the proceedings in the shortest possible time in order that he could file a for the deficiency income tax for 1950. With costs against Ker & Co., Ltd. So
collection case in the Court of First Instance before his right to do so is cut off by ordered.
the passage of time. As moved, the Tax Court dismissed the case and Ker & Co.,
63
G.R. No. L-20477             March 29, 1968           ... There are cases however where a taxpayer may be prevented
REPUBLIC OF THE PHILIPPINES, plaintiff-appellant, from setting up the defense of prescription even if he has not previously
vs. FELIX B. ACEBEDO, defendant-appellee. waived it in writing as when by his repeated requests or positive acts the
Government has been, for good reasons, persuaded to postpone collection
to make him feel that the demand was not unreasonable or that no
          This is a suit for collection of deficiency income tax for the year 1948 in the
harassment or injustice is meant by the Government. (Emphasis supplied.)
amount of P5,962.83. The corresponding notice of assessment was issued on
September 24, 1949. The complaint was filed on December 27, 1961. After the
defendant filed his answer but before trial started he moved to dismiss on the           Likewise, when a taxpayer asks for a reinvestigation of the tax assessment
ground of prescription. The court received evidence on the motion, and on issued to him and such reinvestigation is made, on the basis of which the
September 1, 1962 issued an order finding the same meritorious and hence Government makes another assessment, the five-year period with which an action
dismissing the complaint. The case is before us on appeal by the plaintiff from the for collection may be commenced should be counted from this last assessment.
order of dismissal. (Republic vs. Lopez, L-18007, March 30, 1963; Commissioner v. Sison, et al., L-
13739, April 30, 1963.)
          The statute of limitations which governs this case is Section 332, subsection
(c), of the National Internal Revenue Code, which reads:           In the case at bar, the defendant, after receiving the assessment notice of
September 24, 1949, asked for a reinvestigation thereof on October 11, 1949 (Exh.
A). There is no evidence that this request was considered or acted upon. In fact, on
          SEC. 332. Exemptions as to period of limitation of assessment and
October 23, 1950 the then Collector of Internal Revenue issued a warrant of
collection of taxes. —
distraint and levy for the full amount of the assessment at (Exh. D), but there was
no follow up of this warrant. Consequently, the request for reinvestigation did not
          (c) Where the assessment of any internal-revenue tax has been suspend the running of the period for filing an action for collection.
made with the period of limitation above prescribed such tax may be
collected by distraint or levy or by a proceeding in court, but only if begun
          The next communication of record is a letter signed for the defendant by one
(1) within five years after the assessment of the tax, or (2) prior to the
Troadio Concha and dated October 6, 1951, again requesting a reinvestigation of his
expiration of any period for collection agreed upon in writing by the
tax liability (Exh. B). Nothing came of this request either. Then on February 9,
Collector of Internal Revenue and the taxpayer before the expiration of
1954, the defendant's lawyers wrote the Collector of Internal Revenue informing
such five-year period. The period so agreed upon may be extended by
him that the books of their client were ready at their office for examination (Exh.
subsequent agreements in writing made before the expiration of the period
C). The reply was dated more than a year later, or on October 4, 1955, when the
previously agreed upon.
Collector bestirred himself for the first time in connection with the reinvestigation
sought, and required that the defendants specify his objections to the assessment
          The present suit was not begun within five years after the assessment of the and execute "the enclosed forms for waiver, of the statute of limitations." (Exh. E).
tax, which was in 1949. Was it, however, begun prior to the expiration of any period The last part of the letter was a warning that unless the waiver "was accomplished
for collection agreed upon in writing by the Commissioner of Internal Revenue and and submitted within 10 days the collection of the deficiency taxes would be
the defendant before the expiration of such five-year period? The only evidence of enforced by means of the remedies provided for by law."
such written agreement, in the form of a "waiver of the statute of limitations"
signed by the defendant, is Exhibit U (also Exh. 4), dated December 17, 1959. But
          It will be noted that up to October 4, 1955 the delay in collection could not
this waiver was ineffective because it was executed beyond the original five-year
be attributed to the defendant at all. His requests in fact had been unheeded until
limitation.
then, and there was nothing to impede enforcement of the tax liability by any of the
means provided by law. By October 4, 1955, more than five years had elapsed since
          The plaintiff contends that the period of prescription was suspended by the assessment in question was made, and hence prescription had already set in,
defendant's various requests for reinvestigation or reconsideration of the tax making subsequent events in connection with the said assessment entirely
assessment. The trial court rejected this contention, saying that a mere request for immaterial. Even the written waiver of the statute signed by the defendant on
reinvestigation or reconsideration of an assessment does not have the effect of such December 17, 1959 could no longer revive the right of action, for under the law
suspension. The ruling is logical, otherwise there would be no point to the legal such waiver must be executed within the original five-year period within which suit
requirement that the extension of the original period be agreed upon in writing. could be commenced.

          To be sure, this legal provision, according to some, decisions of this Court,           The order appealed from is affirmed, without pronouncement as to
does not rule out a situation where the taxpayer may be in estoppel to claim costs.1äwphï1.ñët
prescription. Thus we said in Commissioner of Internal Revenue, vs. Consolidated
Mining Co., L-11527, Nov. 25, 1958:
G.R. No. 76281 September 30, 1991
64
COMMISSIONER OF INTERNAL REVENUE, petitioner, abroad. It claimed that it was not able to remit the balance of fifty percent (50%) of
vs. WYETH SUACO LABORATORIES, INC. and THE COURT OF TAX the accrued royalties to its foreign licensors because of Central Bank Circular No.
APPEALS, respondents. 289 allowing remittance of royalties up to fifty percent (50%) only. With regard to
what the Bureau of Internal Revenue claimed as the amount of P2,952,391.00
forming part of the cash dividends declared in 1973, Wyeth Suaco alleged that the
The sole issue in this petition for review on certiorari is whether or not petitioner's
same was due its foreign stockholders. Again, Wyeth Suaco was not able to remit
right to collect deficiency withholding tax at source and sales tax liabilities from
these dividends because of the restriction of the Central Bank in a memorandum
private respondent is barred by prescription.
implementing CB Circular No. 289 dated February 21, 1970. Thus, Wyeth Suaco's
contention was that a withholding tax at source on royalties and dividends becomes
The antecedent facts are as follows: due and payable only upon their actual payment or remittance.

Private respondent Wyeth Suaco Laboratories, Inc. (Wyeth Suaco for brevity) is a On the matter of the withholding tax at source on remuneration for technical
domestic corporation engaged in the manufacture and sale of assorted services, Wyeth Suaco insisted that it was up-to-date in remitting the corresponding
pharmaceutical and nutritional products. Its accounting period is on a fiscal year withholding tax on this income to the Bureau of Internal Revenue.
basis ending October 31 of every year.
As to the assessed deficiency sales tax, Wyeth Suaco maintained that the difference
By virtue of Letter of Authority No. 52415 dated June 17, 1974 issued by then between its landed cost figure (which is the basis for computing the advancesales
Commissioner of Internal Revenue Misael P. Vera, Revenue Examiner Dante tax) and that of the revenue examiner, was due to the use of estimated amounts by
Kabigting conducted an investigation and examination of the books of accounts of the Bureau of Customs and to foreign exchange differential.
Wyeth Suaco.1 On October 15, 1974, he submitted a report containing the result of
his investigation. The report disclosed that Wyeth Suaco was paying royalties to its
Wyeth Suaco however, admitted liability with respect to the short payment of
foreign licensors as well as remuneration for technical services to Wyeth
advance sales tax in the amount of P1,000.00 on its importation of "Mega Polymycin
International Laboratories of London. Wyeth Suaco was also found to have declared
D."5
cash dividends on September 27, 1973 and these were paid on October 31, 1973.
However, it allegedly failed to remit withholding tax at source for the fourth (4th)
quarter of 1973 on accrued royalties, remuneration for technical services and cash On September 12, 1975, the Commissioner of Internal Revenue asked Wyeth Suaco
dividends, resulting in a deficiency withholding tax at source in the aggregate to avail itself of the compromise settlement under LOI 308. In its answer, Wyeth
amount of P3,178,994.15.2 Suaco manifested its conformity to a 10% compromise provided it be applied only
to the basic sales tax, excluding surcharge and interest. As to the deficiency
withholding tax at source, Wyeth took exception on the ground that it involves
Moreover, it was reported that during the periods from November 1, 1972 to
purely a legal question and some of the amounts included in the assessment have
December 31, 1972 and January 1, 1973 to October 31, 1973, Wyeth Suaco
already bee paid.
deducted the cost of non-deductible raw materials, resulting in its alleged failure to
pay the correct amount of advance sales tax. There was reportedly also a short
payment of advance sales tax in its importation of "Mega Polymycin D" on October On December 10, 1979, petitioner, thru then acting Commissioner of Internal
3, 1972. All these resulted in a deficiency sales tax in the amount of P60,855.21 Revenue Ruben B. Ancheta, rendered a decision reducing the assessment of the
and compromise penalty in the amount of P300.00 or a total amount of withholding tax at source for 1973 to P1,973,112.86. However, the amount of
P61,155.21.3 P61,155.21 as deficiency sales tax remained the same.6

Consequently, the Bureau of Internal Revenue assessed Wyeth Suaco on the Thereafter, Wyeth Suaco filed a petition for review in Court of Tax Appeals on
aforesaid tax liabilities in two (2) notices dated December 16, 1974 and December January 18, 1980, praying that lpeti tioner be enjoined from enforcing the
17, 1974. These assessment notices were both received by Wyeth Suaco on assessments by reason of prescription and that the assessments be declared null
December 19, 1974.4 and void for lack of legal and factual basis.7

Thereafter, Wyeth Suaco through its tax consultant SGV &Co., sent the Bureau of On February 7, 1980, petitioner issued a warrant of distrain of personal property
Intemal Revenue two (2) letters dated January 17, 1975 and February 8, 1975, and warrant of levy of real property again private respondent to enforce collection
protesting the assessments and requesting their cancellation or withdrawal on the of the deficiency taxes. These were served on private respondent on March 12,
ground that said assessments lacked factual or legal basis. 1980.8 However, collection of the deficiency taxes by virtue of warrants of distraint
and levy was enjoined by respondent court upon motion of Wyeth Suaco in a
resolution dated May 22, 1980.9
Wyeth Suaco argued that it was not liable to pay withholding tax at source on the
accrued royalties and dividends because they have yet to be remitted or paid
65
On May 30, 1980, petitioner filed his answer to Wyeth Suaco's petition for review before the expiration of the period previously agreed upon. (emphasis
praying, among others, that private respondent be declared liable to pay the supplied)
amount of P61,155.21 as deficiency sales tax for the periods November 1, 1972 to
December 31, 1972 and January 1, 1973 to October 31, 1973, plus 14% annual
The main thrust of petitioner for the allowance of this petition is that the five-year
interest thereon from December 17, 1974 until payment thereof pursuant to Section
prescriptive period provided by law to mak a collection by distraint or levy or by a
183 (now Section 193) of the Tax Code, and the amount of P1,973,112.86 as deficie
proceeding in court has not yet prescribed. Although he admits that more than five
withholding tax at source for the 4th quarter of 1973 plus 5% surcharge and 14%
(5) years have already lapsed from the time the assessment notices were received
per annum interest thereon from December 16, 1974 to December 16, 1977,
by private respondent on December 19, 1974 up to the time the warrants of
pursuant to Section 51 (e) of the Tax Code of 1977, as amended.10
distraint and levy were served on March 12, 1980, he avers that the running of the
prescriptive period was stayed or interrupted when Wyeth Suaco protested the
On August 29, 1986, the Court of Tax Appeals rendered a decision enjoining the assessments. Petitioner argues that the protest letters sent by SGV & Co. in behalf
Commissioner of Internal Revenue from collecting the deficiency taxes, the of Wyeth Suaco dated January 17, 1975 and February 8, 1975, requesting for
dispositive portion of which reads as follows: withdrawal and cancellation of the assessments were actually requests for
reinvestigation or reconsideration, which could interrupt the running of the five-year
prescriptive period.
WHEREFORE, the decision appealed from is hereby reversed and
respondent Commissioner of Internal Revenue is hereby enjoined from
collecting the deficiency withholding tax at source for the fourth quarter of Wyeth Suaco, on the other hand, maintains the position that it never asked for a
1973 as well as the deficiency sales tax assessed against petitioner (Wyeth reinvestigation nor reconsideration of th assessments. What it requested was the
Suaco). Without pronouncement as to costs.11 cancellation and with drawal of the assessments for lack of legal and factual basis.
Thus, its protest letters dated January 17, 1975 and February 8, 1975 did not
suspend or interrupt the running of the five-year prescriptive period.
The basis of the above decision was the finding of the Tax Court that while the
assessments for the deficiency taxes were made within the five-year period of
limitation, the right of petitioner to collect the same has already prescribed, in Settled is the rule that the prescriptive period provided by law to make a collection
accordance with Section 319 (c) of the Tax Code of 1977. The said law provides that by distraint or levy or by a proceeding in court is interrupted once a taxpayer
an assessment of any internal revenue tax within the five-year period of limitation requests for reinvestigation or reconsideration of the assessment. In the case
may be collected by distraint or levy or by a proceeding in court, but only if begun of Commissioner of Internal Revenue vs. Capitol Subdivision, Inc.,12 this Court held:
within five (5) years after the assessment of the tax.
The period of prescription of action to collect a taxpayer's deficiency
Hence, this recourse by petitioner. income tax assessment is interrupted when the taxpayer request for a
review or reconsideration of said assessment, and starts to run again when
said request is denied.
The applicable laws in the instant case are Sections 318 and 319 (c) of the National
Internal Revenue Code of 1977 (now Sections 203 and 224 of the National Internal
Revenue Code of 1986), to wit: In another case, this Court stated that the statutory period of limitation for
collection may be interrupted if by the taxpayer's repeated requests or positive acts
the Government has been, for good reasons, persuaded to postpone collection to
SEC. 318. Period of limitation upon assessment and collection — Except as provided
make him feel that the demand was not unreasonable or that no harassment or
in the succeeding section, internal revenue taxes shall be assessed within five years
injustice is meant by the Goverrument.13 Also in the case of Cordero vs. Gonda,14 we
after the return was filed, and no proceeding in court without assessment for the
held:
collection of such taxes shall be begun after the expiration of such period. ...

Partial payment would not prevent the government from suing the
SEC. 319. Exceptions as to period of limitations of assessment and collection of
taxpayer. Because, by such act of payment, the government is not thereby
taxes. —
"persuaded to postpone collection to make him feel that the demand was
not unreasonable or that no harassment or injustice is meant." This is the
(c) Where the assessment of any internal revenue tax has been made underlying reason behind the rule that the prescriptive period is arrested
within the period of limitation above-prescribed such tax may be collected by the taxpayer's request for re-examination or reinvestigation — even if
by distraint or levy by a proceeding in court, but only if begun (1) within he "has not previously waived it (prescription in writing)". ... (emphasis
five years after the assessment of the tax, or (2) prior the expiration of supplied)
any period for collection agreed upon in writing by the Commissioner and
the taxpayer before the expiration of such five-year period. The period so
agreed upon may be extended by subsequent agreements in writing made
66
Thus, the pivotal issue in this case is whether or not Wyeth Suaco sought Acting Commissioner Ruben B. Ancheta was date December 10, 1979 and received
reinvestigation or reconsideration of the deficiency tax assessments issued by the by private respondent on January 2, 1980, fixed its tax liability at P1,973,112.86 as
Bureau of Internal Revenue. deficiency withholding tax at source and P61,155.21 as deficiency sales tax. It was
only upon receipt by Wyeth Suaco of this final assessment that the five-year
prescriptive period started to run again.
After carefully examining the records of the case, we find that Wyeth Suaco
admitted that it was seeking reconsideration of the tax assessments as shown in a
letter of James A. Gump, its President and General Manager, dated April 28, 1975, Verily, the original assessments dated December 16 and 17, 1974 were both
the relevant portion of which is quoted hereunder, to wit: received by Wyeth Suaco on December 19, 1974. However, when Wyeth Suaco
protested the assessments and sought its reconsideration in two (2) letters received
by the Bureau of Internal Revenue on January 20 and February 10, 1975, the
We submit this letter as a follow-up to our protest filed with your office,
prescriptive period was interrupted. This period started to run again when the
through our tax advisers, Sycip, Gorres, Velayo & Co., on January 20 and
Bureau of Internal Revenue served the final assessment to Wyeth Suaco on January
February 10, 1975 regarding alleged deficiency on withholding tax at
2, 1980. Since the warrants of distraint and levy were served on Wyeth Suaco on
source of P3,178,994.15 and on percentage tax of P60,855.21, including
March 12, 1980, then, only about four (4) months of the five-year prescriptive
interest and surcharges, on which we are seeking
period was used.
reconsideration.15 (emphasis supplied)

Having resolved the issue of prescription, we now come to the merits of the case.
Furthermore, when Wyeth Suaco thru its tax consultant SGV & Co. sent the letters
protesting the assessments, the Bureau of Internal Revenue, Manufacturing Audit
Division, conducted a review and reinvestigation of the assessments. This fact was Wyeth Suaco questions the legality of the regulation imposed by the Bureau of
admitted by Wyeth Suaco thru its Finance Manager in a letter dated July 1, 1975 Intemal Revenue of requiring a withholding agent or taxpayer to remit the taxes
addressed to the Chief, Tax Accounts Division. The pertinent portion of said letter deducted and withheld at source on incomes which have not yet been paid. It
reads as follows: maintains the stand that withholding tax at source should only be remitted to the
Bureau of Internal Revenue once the incomes subject to withholding tax at source
have actually been paid. Thus, private respondent avers that it was not liable to
This will acknowledge receipt of your letter dated May 22, 1975 regarding
remit the taxes withheld at source on royalties and dividends unless these incomes
our alleged income and business tax deficiencies for fiscal year 1972/73.
have been actually paid to its foreign licensors and stockholders.

Nevertheless, please be advised that the deficiency tax stated in your letter
It is said that taxes are what we pay for civilized society. Without taxes, the
is what we are protesting on pursuant to the letters we filed with the
government would be paralyzed for lack of the motive power to activate and
Bureau of Internal Revenue on January 20, 1975 and on February 10,
operate it. ... It is the lifeblood of the government and so should be collected
1975.
without unnecessary hindrance ...17

As we understand, the matter is now undergoing review and consideration


In line with this principle, the Tax Code, particularly Section 54 (a) [now Section 51
by your Manufacturing Audit Division. Pending the outcome of their
(a)] provides that "the Commissioner of Internal Revenue may, with the approval of
decision, we regret our inability to make settlement. ... 16 (Emphasis
the Secretary of Finance, require the withholding agents to pay or deposit the taxes
supplied)
deducted and withheld at more frequent intervals when necessary to protect the
interest of the government. The return shall be filed and the payment made within
Although the protest letters prepared by SGV & Co. in behalf of private respondent 25 days from the close of each calendar quarter". Presently, Revenue Regulation
did not categorically state or use th words "reinvestigation" and "reconsideration," No. 6-85 effective July 1, 1985, requires the filing of monthly return and payment of
the same are to be treated as letters of reinvestigation and reconsideration. By taxes withheld at source within (10) days after the end of each month.
virtue of these letters, the Bureau of Internal Revenue ordered its Manufacturing
Audit Division to review the assessment made. Furthermore, private respondent's
Moreover, the records show that Wyeth Suaco adopted the accrual method of
claim that it did not seek reinvestigation or reconsideration of the assessments is
accounting wherein the effect of transactions and other events on assets and
belied by the subsequent correspondence or letters written by its officers, as shown
liabilities are recognized and reported in the time periods to which they relate rather
above.
than only when cash is received or paid. The "Report of Investigation" submitted by
the tax examiner indicated that accrual was the basis of the taxpayer's
These letters of Wyeth Suaco interrupted the running of the five-year prescriptive return.18 Thus, private respondent recorded accrued royalties and dividends payable
period to collect the deficiency taxes. The Bureau of Internal Revenue, after having as well as the withholding tax at source payable on these incomes. Having deducted
reviewed the record of Wyeth Suaco, in accordance with its request for and withheld the tax at source and having recorded the withholding tax at source
reinvestigation, rendered a final assessment. This final assessment issue by then
67
payable in its books of accounts, private respondent was obligated to remit the On February 28, 1948, petitioner filed his income tax return for the year 1947.
same to the Bureau of Internal Revenue. Subsequently, he also filed income tax returns for the years 1948 to 1950.
Sometime in 1951, respondent Collector issued an income tax assessment notice
charging petitioner an income tax on the amount of P9,004.22, representing
With regard to the accuracy of the assessment on deficiency sales tax, we rule that
expenses incurred by petitioner for the repair of his house, and which petitioner
the examiner's assessment should be given full weight and credit, in the absence of
claimed as deductible expense in his 1947 returns. On December 14, 1951, in a
proof submitted by Wyeth Suaco to the contrary. This is in line with our ruling in
letter addressed to respondent Collector, petitioner requested reconsideration of this
several cases wherein we said that tax assessments by tax examiners are presumed
opinion of the Collector on the ground that there was nothing added to the house to
correct and made in good faith. The taxpayer has the duty to prove otherwise. In
increase its original value, and, therefore, said amount cannot be made part of the
the absence of proof of any irregularities in the performance of duties, an
asset. On May 25, 1953, without having received a reply to his first request,
assessment duly made by a Bureau of Internal Revenue examiner and approved by
petitioner reiterated his request for exemption from the income tax mentioned in
his superior officers will not be disturbed. All presumptions are in favor of the
respondent's assessment notice in another letter addressed to the Municipal
correctness of tax assessments.19 The case of Commissioner of Internal Revenue vs.
Treasurer of San Fernando, La Union. On July 10, 1954, the Examiner assigned to
Construction Resources of Asia, Inc.,20 where this Court cited 51 Am. Jur. pp. 620-
investigate the case submitted his report recommending denial of petitioner's
621, states the principle in detail, thus:
request, and charging as disallowances the costs of repairs but recommending
instead that said repairs be capitalized and depreciated yearly. On February 9,
All presumptions are in favor of the correctness of tax assessments. The 1955, respondent Collector issued a tax assessment notice of P753.51 for 1947. He
good faith of tax assessors and the validity of their actions are presumed. also issued assessment tax notices for the years 1948 to 1950. On February 19,
They will be presumed to have taken into consideration all the facts to 1955, petitioner returned assessment notices from 1947 to 1950, with a request for
which their attention was called. No presumption can be indulged that all of information how the computation of his tax liability had been arrived at.
the public officials of the State in the various counties who have to do with
the assessment of property for taxation will knowingly violate the duties
On September 14, 1956, respondent Collector issued a warrant of distraint and levy
imposed upon them by law.
against petitioner's properties to satisfy the amount of P1,808.10, petitioner's total
income tax liability for the years 1947 to 1950, plus its legal increments. On
The final assessment issued by the Bureau of Internal Revenue declared the September 19, 1956, the municipal treasurer distrained and levied on petitioner's
issuance of deficiency sales tax assessments to be legal and valid. It was residential house. On September 29, 1956, petitioner filed a petition for review,
ascertained that during the investigation, Wyeth Suaco deducted non-deductible seeking to declare the warrant of distraint and levy null and void, and to enjoin
raw materials which were not subjected to advance sales tax thereby resulting in its respondent from executing the warrant. On appeal to the Tax Court, that Court held
failure to pay the correct amount of sales tax under Section 183, in relation to that the right to collect summarily had already prescribed, and declared the warrant
Section 186 and 186-B of the Tax Code, prior to and after amendment by null and void. Appeal by the respondent Collector to this Court was later dismissed.
Presidential Decree No. 69. Wyeth Suaco was not able to refute this by submitting
supporting documents.21
On March 18, 1959, petitioner filed an amended petition for review to the end that
the real matter in dispute may be completely determined in a single proceeding;
WHEREFORE, the petition is GRANTED. Wyeth Suaco Laboratories, Inc, is hereby and on October 7, 1959, the Tax Court rendered the decision, modifying respondent
ordered to pay the Bureau of Internal Revenue the amount of P1,973,112.86 as Collector's decision in the sense stated above, and from which petitioner is now
deficiency withholding tax at source, with interest and surcharge in accordance with appealing.
law, without prejudice to any reduction brought about by payments or remittance
made. Wyeth Suaco Laboratories, Inc. is also ordered to pay the Bureau of Internal
Petitioner argues that the Tax Court erred in holding that an initial assessment
Revenue the amount of P60,855.21 as deficiency sales tax with interest and
notice for the year 1947 was sent to petitioner sometime in 1951 and in finding that
surcharge in accordance with law. Costs against private respondent.
his requests for reconsideration suspended the running of the prescriptive period,
thus upholding the non-prescriptibility of the right of the Collector to collect from
SO ORDERED. petitioner a deficiency income tax for 1947; the respondent claims, in turn, that the
evidence on record regarding the initial assessment is clear, as admitted in
G.R. No. L-16705            October 30, 1962 petitioner's requests for reconsideration (letters of December 14, 1951 and May 25,
ANTONIO E. QUEROL, petitioner, 1953), which evidence the Tax Court can take cognizance of since evidence not
vs. COLLECTOR OF INTERNAL REVENUE, respondent. formally offered may be taken into account in deciding a case; that since the period
from the first request for reconsideration to the date of the respondent's receipt of
the result of the investigation suspended the running of the period for the running
Review of a decision of the Court of Tax Appeals sentencing petitioner to pay a of prescription, the tax assessment notices issued on February 9, 1955 were on
deficiency income tax of P753.51 for the year 1947, plus 5% surcharge for late time, and so was the judicial action for collection, which was considered filed when
payment and interest at the rate of 1% per month from March 16, 1955.

68
respondent Collector filed his answer to the amended petition for review on April 8, Tax Appeals, asked the Court, in April of 1959, to order payments thereof, less than
1959. 5 years after the revised assessment was made.

The issue is here concentrated by the appellant taxpayer on his defense of Much is made by appellant of the fact that no clear evidence exists on the date
prescription of the action to collect the tax due on his income during 1947. when the original tax assessment was issued by the Collector or when it was
received by the taxpayer. But it must be remembered that prescription is a matter
of defense; hence, the burden is on the taxpayer to prove that the full period
Starting from the ruling of the Tax Court that the irregularities in his return do not
limitation has expired, and this requires him to positively establish the date when
make it false or fraudulent, the appellant first contends that the Collector's 1955
the period started running, and when the same was fully accomplished.
revised assessment is void, because it was not made within the five years
prescribed by section 331 after the Internal Revenue Code, from and after the filing
of his income tax return (Exhibit A) on February 28, 1948. We agree with the court Finding no cogent reason to vary the ruling of the Tax Court, its decision is hereby
below that this contention is not tenable, because there had been a preceding affirmed, with costs against appellant.
assessment in 1951. In his own letter of December 14, 1951 (Exhibit O or 11A),
Querol stated:

With reference to the income tax assessment notice No. 24-A-36- 51/47, I have the
honor to request reconsideration of your opinion charging me an income tax for the
amount of expenses I incurred for the repair of my house.

These words necessarily import that the taxpayer had received a tax assessment
notice before the date of the letter. That such assessment referred to his 1947
income tax return is shown by his reference to the value of the repairs made to his
house, that he had claimed as deduction in his 1947 return (Exhibit A). No similar
claim appears in any other tax return made by him. Now, from February, 1948
(when the return was filed) to December, 1951 is less than four years; hence, the
first assessment was made on time.

It is true that the Collector revised the original assessment on February 9, 1955;
and appellant avers that this revision was invalid in that it was not made within five-
year prescriptive period fixed by law (Collector vs. Pineda L-14522, 31 May 1961).
But the fact is that the revised assessment was merely a result of petitioner
Querol's requests for reconsideration of the original assessment, contained in his
letters of December 14, 1951 and May 25, 1953. The records of the Bureau of
Internal Revenue show that after receiving the letters, the Bureau conducted a
reinvestigation of petitioner's tax liabilities, and, in fact, sent a tax examiner to San
Fernando, La Union, for that purpose; that because of the examiner's report, the
Bureau revised the original assessment, and that while it still refused to allow full
deduction of the repairs to the taxpayer's residence as a business expense, it
allowed him to capitalize the amount, and permitted him to deduct a reasonable
depreciation for 1947. In other words, the reconsideration was granted in part, and
the original assessment was altered. Consequently, the period between the petition
for reconsideration and the revised assessment should be subtracted from the total
prescriptive period (Republic vs. Ablaza, L-14519, 26 July 1960).

We have also ruled that once the assessment has been reconsidered at the
taxpayer's instance, the five-year period for filing of the court action for collection
should begin to run from the date of the reconsidered or modified assessment.
(Collector vs. Pineda, supra.) The judicial action to recover the taxes in the present
case was made when the Collector, countering the taxpayer's suit in the Court of

69

You might also like